X



トップページ数学
1002コメント398KB
数理論理学(数学基礎論) その14
レス数が1000を超えています。これ以上書き込みはできません。
0001132人目の素数さん
2019/03/22(金) 11:55:47.59ID:oVa5RLRo
数学基礎論は、数学の基礎づけを目的として誕生したが
現在では、数理論理学として、証明論、再帰的関数論、
構成的数学、モデル理論、公理的集合論など、 多くの分野
に分かれ、極めて高度な純粋数学として発展を続けています。
(「数学基礎論」という言葉の使い方には、専門家でも
若干の個人差があるようです。)
応用、ないし交流のある分野は、計算機科学の諸分野や、
代数幾何学、英米系哲学の一部などを含み、多岐にわたります。
(数学セミナー98年6月号、「数学基礎論の学び方」
ttp://www.math.tohoku.ac.jp/~tanaka/intro.html
或いは 岩波文庫「不完全性定理」 6.4 数学基礎論の数学化などを参照)

前スレ
数学基礎論・数理論理学 その13
https://rio2016.5ch.net/test/read.cgi/math/1532721493/
0002132人目の素数さん
2019/03/22(金) 13:13:35.41ID:64YyWaX6
数学基礎論は数学の基礎論ではない。まったく数学には関係ない論理のお遊び
0004132人目の素数さん
2019/03/22(金) 21:40:24.58ID:EWzYRJ4u
計算機科学はチューリングマシンをイデア的に捉える
論理学は一階述語論理を人の論理の形式化とする
数学はチューリングマシンも一階述語論理も集合とする
0005132人目の素数さん
2019/03/24(日) 07:17:00.84ID:N4FNRVCO
Yコンビネータ

型無しラムダ計算においてよく知られた(そしておそらく最もシンプルな)
不動点コンビネータはYコンビネータと呼ばれる。これはハスケル・カリーによって
発見されたもので、次のように定義される。

Y = (λf . (λx . f (x x)) (λx . f (x x)))

実際に関数gを適用することによって、この関数が不動点コンビネータとして動作する
のが分かる。

Y g = (λf . (λx . f (x x)) (λx . f (x x))) g (Yの定義より)
= (λx . g (x x)) (λx . g (x x)) (λfのβ簡約、主関数をgに適用)
= (λy . g (y y)) (λx . g (x x)) (α変換、束縛変数の名前を変える)
= g ((λx . g (x x)) (λx . g (x x))) (λyのβ簡約、左の関数を右の関数に適用)
= g (Y g) (第2式より)

これをそのままラムダ計算で使うと、評価戦略が値渡しだった場合には (Y g) が
(g (Y g)) と展開された後も、引数の値を先に求めようとして
(g (g (Y g))) →...→ (g ... (g (Y g))...) のように無限に展開され続けて
止まらなくなってしまうので、次節で示すZコンビネータのように修正する。

評価戦略が名前渡しの場合はこのまま使える。 このカリーによるコンビネータのみを
Yコンビネータとすることもあるが、実装などでは不動点コンビネータを指す名前として
他の形であってもYという名前を使っていることもある。

SKIコンビネータ計算では次のようになる。
Y = S (K (S I I)) (S (S (K S) K) (K (S I I)))
0006132人目の素数さん
2019/03/24(日) 07:19:23.05ID:N4FNRVCO
>>4
計算機科学はチューリングマシンをハードウェア、機械として捉える、が正解
0007132人目の素数さん
2019/03/24(日) 21:34:57.51ID:FOTRZJL+
>>5
> Yコンビネータ
・・・
> Y = (λf . (λx . f (x x)) (λx . f (x x)))
> 評価戦略が名前渡しの場合はこのまま使える。 ・・・

これは間違い。評価戦略が名前渡しであっても Y は使えない。それを示すために
君が書き下してくれたY gの式変形をどちら向きにも進める変換 “=” による等式でなく
簡約 “→” による評価として→で向き付けられた形に書き直してみよう。

(“≡” は構文的に同一の項であることを示す。なお、「α変換」は
束縛変数に対する名前の付け替えというメタ言語的な操作なので
ここではα変換の前後は構文的に同一の項として扱い“≡”で示した)

  Y g
≡ (λf . (λx . f (x x)) (λx . f (x x))) g (Yの定義より)
→ (λx . g (x x)) (λx . g (x x)) (λfのβ簡約、主関数をgに適用)
≡ (λy . g (y y)) (λx . g (x x)) (α変換、束縛変数の名前を変える)
→ g ((λx . g (x x)) (λx . g (x x))) (λyのβ簡約、左の関数を右の関数に適用)
← g (Y g) (第2式より)

最後のステップが逆向きの簡約 “←” であることに注意。
このステップは絶対に本来の簡約 “→” の向きには書き直せないので
Y コンビネータは名前呼びでも使えないことが分かる。

名前呼びの簡約戦略の場合に正しく動く不動点コンビネータで最も有名な例は
Turing のΘコンビネータで、λ項として書き下すと以下のようになる:

Θ ≡ (λx y, y(x x y))(λx y, y(x x y))

最後に、これを用いた Θg という項が簡約によって確かに g(Θg) になることを確かめておく:

  Θ g
≡ ((λx y, y(x x y))(λx y, y(x x y))) g
≡ ((λz w, w(z z w))(λx y, y(x x y))) g ・・・・・・・・・・・・ α変換
→ (λw. w((λx y, y(x x y))(λx y, y(x x y)) w)) g ・・・  β簡約
≡ (λw. w(Θ w)) g ・・・・・・・・・・・・・・・・・・・・・・・・・・・・・ Θの定義で置き換え
→ g(Θ g)

なお、値呼びの簡約戦略で正しく動く不動点コンビネータも勿論ある。
具体的な定義は、例えば次にある:
Daniel P. Friedman & Matthias Felleisen “The Little Schemer” 4th ed., p. 172 (The MIT Press, 1996).
0008132人目の素数さん
2019/03/25(月) 18:38:51.28ID:ZEwsypVC
>>7訂正

失礼、チューリングのΘコンビネータのλ項で、束縛変数とλ項本体とを区切るピリオド “.” が間違ってコンマ “,” になってました
0009132人目の素数さん
2019/03/25(月) 22:13:02.02ID:MhMPsIFV
Bertrand Russellはその著書「Principia Mathematica」の序で次のように述べている。
「……数学の原理に関するいかなる理論も、それを支持する主たる論拠は、
常に帰納的なものでなければならない。換言すれば、それはその問題の理論によって、
我々が通常の数学を演繹できるという事実の中に存在するものでなければならない。
数学においては、最高度の自明性は、演繹のはじめの段階で得られることはめずらしく、
解いていくにつれてある段階で得られるというのが普通である。

したがって、その段階に到達するまでの初期の演繹においては、前提から真の結論が
引き出されるという理由でその前提が信じられるのであり、前提からその結論が
引き出されるという理由でその結論が信じられるのではない」
0011132人目の素数さん
2019/03/31(日) 10:07:14.17ID:pd4YzCEG
age
0012132人目の素数さん
2019/04/11(木) 21:03:28.51ID:ZT7Ri93V
まだまだ死に絶えてはいません
0013132人目の素数さん
2019/05/06(月) 10:11:07.83ID:X59eIqI1
∪Aに関する疑問を突き詰めてみると
P(x)≡∃y∈A(x∈y)
の真偽って確定するとしていいのかな?
与えられたxについてyを選ばなくて良いって言うのはなんだか胡散臭いし
選び出すことができるってなんだか選択公理っぽい
公理にしてる根拠が薄くないかな
0014132人目の素数さん
2019/05/06(月) 15:38:16.17ID:ffT6Gv3d
選び出すって何おかしなこと言うてんねん
存在するかどうかを問題にしてるだけや
0015132人目の素数さん
2019/05/06(月) 19:27:43.69ID:FaiecvaY
何が存在してるか言えなくて
何か存在してるだけではやだなぁ
0016132人目の素数さん
2019/05/06(月) 19:38:32.02ID:X9tK7TPF
xに対して
∃y∈A(x∈y)
が成り立っているとき、そのyをブルバキ流に書けば
τy(y∈A∧x∈y)

これは∃の意味をどう決めるかによる
0017132人目の素数さん
2019/05/07(火) 18:27:34.24ID:yJ06pJHo
ブルバキだと
{(x,τy(y∈A∧x∈y))}
は集合なの?
これが集合なら
F={(x,τy(y∈Bx))}
も集合?Fって選択関数のことだけど
0018132人目の素数さん
2019/05/07(火) 19:17:25.36ID:+08/LJkT
xが集合で、
∃y(y∈A∧x∈y)
が成り立ってるなら
τy(y∈A∧x∈y)
も集合。だから
(x,τy(y∈A∧x∈y))
は集合。よって
{(x,τy(y∈A∧x∈y))}
は集合。

Bxが何のことかわからないけど、
何か集合aがあって、Bをa上の写像と見做すことにする。
∀x∊a(Bx≠φ)
が成り立ってるなら、
F={(x,τy(y∈Bx))| x∊a}
はブルバキでは選択関数となる。
でもそれはτに強い規則を課していて
s=t⇒τy(y∈Bs)=τy(y∈Bt)
が満たされるから。
ブルバキでは選択公理は定理となる。
でもブルバキ読んでないしあまり詳しいことはよく知らない。
0019132人目の素数さん
2019/05/07(火) 23:24:21.19ID:yJ06pJHo
>>18
>ブルバキでは選択公理は定理となる
ありがとう
やはりブルバキだとそうなるのね
ちょっと強すぎな感じ
0020132人目の素数さん
2019/05/16(木) 14:40:02.69ID:+P5Ylnkx
数学基礎論=ZFC教
0022132人目の素数さん
2019/05/16(木) 22:56:30.67ID:1KZufyIl
>>20
> 数学基礎論=ZFC教

全くの間違い
数学基礎論は本来は今の数理論理学全般を指すのではなく数学の基礎付け=ヒルベルトのプログラムの精神を引き継いだ学問分野を表していた
そういう意味では数学基礎論の最もコアな部分は証明論それもGentzen流の純粋に構文的な手段のみに限定して還元主義的な証明論だが
このGentzen流証明論は構文的であるが故にZFCのような集合論とは全く無縁と言って良い

現在の日本のように「数学基礎論」という学問名を「数理論理学」つまり論理を数学的な手段で分析する学問一般を表すものとして使うのは
数学基礎論本来の意味からは邪道極まりない使い方

竹内先生や八杉さんらが共立現代数学講座から出した『数学基礎論』は実に正しい使い方だ、何しろあの本には証明論しか書いてないのだから
その点、新井さんの岩波からの『数学基礎論』は邪道なタイトルだなw

でも、竹内先生らの本が同講座の巻が軒並み品切れとなって久しいころに復刊された時には『証明論入門』というタイトルに変えられてしまった
その頃には「数学基礎論」という言葉が本来の意味でなく数理論理学一般を指す邪道な使われ方がメジャーになってしまっていたから
数理論理学の教科書としては証明論しか書いてないので羊頭狗肉だと(数学基礎論の本来の意味を知らない)無知な読者たちから
批判を浴びるのを出版社が恐れたからかも知れないね
0025132人目の素数さん
2019/05/16(木) 23:33:46.85ID:gL9bCd2q
「本来は」とか言い出したら現代数学のほぼ全てが邪道になってしまうよ
数学に限らず物理もそうかな
0026132人目の素数さん
2019/05/17(金) 07:36:28.10ID:xqLphdBG
>>24
wikipediaでもmathoverflowでもmathematical logicを数学の分野として扱ってるが
0027132人目の素数さん
2019/05/17(金) 16:46:16.68ID:xABauc9m
逆にマスマティクスじゃないロジックってなんなの?
0030132人目の素数さん
2019/05/18(土) 00:28:38.47ID:SjRfDMFu
>>23
“mathematical logic”という分野の意味で「数理論理学」でなく「数学基礎論」の語を使う日本だけが異常なんだよ
海外では“foundations of mathematics”と“mathematical logic”は適切に使い分けられているよ
学問分野の名称(前者FMは後者MLの一部分というか“philosophy of mathematics”とも共通部分を持つような学問領域を指す)でも
書籍のタイトルでもね
0031132人目の素数さん
2019/05/18(土) 00:40:09.67ID:psenGI9v
数学に威張り散らすより実学工学な計算機科学と密接に連携しろよ。

まあ文学部卒のバカだと実学な時点で臍が曲がるんだろうけど。
0032132人目の素数さん
2019/05/18(土) 06:15:18.02ID:HKFcHFTe
超巨大基数ってクラスぐらいに大きい集合があるってことにしたいことかな?
ZFに付け加えても矛盾起こらないんだろうけれども
0033132人目の素数さん
2019/05/18(土) 08:51:38.68ID:nop0lv60
文学部が計算機科学と連携とか面白そうだな
俺は数学科卒だけど
0034132人目の素数さん
2019/05/18(土) 23:23:35.48ID:nCdi+f/4
今日本の数理論理学、数学基礎論の日本を代表する研究者って誰?
0035132人目の素数さん
2019/05/18(土) 23:37:50.35ID:HKFcHFTe
>>34
新井先生
0037132人目の素数さん
2019/05/19(日) 11:14:04.85ID:Kf1QbH9H
test
0039132人目の素数さん
2019/06/05(水) 11:44:34.47ID:bMrghUP7
n変数部分帰納的関数f、gについて質問ですが、
f \simeq g の定義って、「f、gの定義域が一致して、その定義域における値も一致している」
つまり「f、gを(n+1)項関係関係と見たとき、集合としてf=g」という理解でいいですか?
0040132人目の素数さん
2019/06/05(水) 11:50:24.27ID:bMrghUP7
それとも、「f、gの定義域は異なっていても良いが、f、g両方の定義域に属する元に対しては値が一致する」ということですか?
0041132人目の素数さん
2019/06/05(水) 23:36:31.75ID:MBnOpo4E
さあ?
0044132人目の素数さん
2019/08/24(土) 19:02:54.25ID:F/68y/86
新井敏康『数学基礎論』の集合論の章で、
Godel operationとして書いてある
F4(x,y):={u∪{v}:u∈x, v∈y}から
直積が構成できると書いてあるけど、
具体的にはどうやれば良いですかね?

冪集合の公理も置換公理もないから
どうやれば良いのかちょっとよく分からないのですが。
0045132人目の素数さん
2019/09/05(木) 19:18:25.75ID:l3HLNP+x
このスレ死んだな
たまにチラホラ更新されるから見てたんだけど
0046132人目の素数さん
2019/09/05(木) 23:14:43.33ID:K6NA6nMr
新井さんの本のタイトルが『数学基礎論』ってなんか大袈裟というか羊頭狗肉というか

数学の基礎付けに関して特記すべきような内容は何も書いてなくて
数理論理学の教科書として標準的な話題を扱ってるだけなんだから
素直に『数理論理学』ってタイトルにしとけばよかったのに
0047132人目の素数さん
2019/09/11(水) 19:20:58.37ID:5ZZj5QzZ
数理論理学も複素関数化すると簡単になるのか?
解析接続とか数理論理学で聞いたことないが…
0048132人目の素数さん
2019/09/12(木) 15:01:13.79ID:H5VlAQfR
>>47
頑張ってね
0049132人目の素数さん
2019/09/12(木) 15:01:40.62ID:H5VlAQfR
>>46
英文タイトルは?
0050132人目の素数さん
2019/09/12(木) 15:11:03.24ID:H5VlAQfR
>>44
F4って順序対には成らないの?
0051132人目の素数さん
2019/09/12(木) 18:41:57.39ID:/GDlAK12
>>47-48
解析接続を一般化した概念の「層」を使った強制法という手法で連続体仮説が解決されてる。
0052132人目の素数さん
2019/09/12(木) 19:24:26.30ID:DAPNlTut
強制法に層は普通出てこないと思うが
そういうのが載ってる資料とかあるなら興味あるから知りたい
0053132人目の素数さん
2019/09/12(木) 19:25:55.71ID:V8gkaG1N
>>51
> 解析接続を一般化した概念の「層」

本の拾い読みばっかりしてる感じ
0054132人目の素数さん
2019/09/12(木) 21:18:20.14ID:/GDlAK12
>>53
コピペで数物の話題をIUTのスレに書き込んでる奴にストーカーされても困るしな。
0056132人目の素数さん
2019/09/13(金) 00:57:42.94ID:bz4llzC/
>>52 トポスのことを層と同一視してしまっているのだろうと思う。
ある種のトポスと一階述語論理に対応がついて,その上で強制法が展開できるのはマックレーンのSGLとかに載っているが,層論的に整理した複素解析とトポスとは直接的な関係はあんまりないと思う。知らんけど
0057132人目の素数さん
2019/09/13(金) 01:05:08.44ID:bz4llzC/
>>52 トポスのことを層と同一視してしまっているのだろうと思う。
ある種のトポスと一階述語論理に対応がついて,その上で強制法が展開できるのはマックレーンのSGLとかに載っているが,層論的に整理した複素解析とトポスとは直接的な関係はあんまりないと思う。知らんけど
0058132人目の素数さん
2019/09/14(土) 17:43:50.37ID:VYIPOabR
>>49
Mathematical Logic だね
0059132人目の素数さん
2019/09/14(土) 18:20:05.50ID:n4OZq3Li
京都賞が如何にも基礎論っぽい方向だしそう名付ける方が本人にはしっくりくるんだろう
0060132人目の素数さん
2019/09/15(日) 16:47:46.79ID:g2F0dADR
集合論の初歩の指導に関する質問

現代数学の系譜 工学物理雑談 古典ガロア理論も読む77
http://rio2016.5ch.net/test/read.cgi/math/1568026331/

ここの>>1
「{}∈{{}} {{}}∈{{{}}} だから {}∈{{{}}}だ」 とか
「{{}}∈{{{}}} で {{}}は集合 だから {{}}⊂{{{}}}だ」 とか
トンチンカンなことばっかりいうんですよ

どういう指導が効果的でしょうか?
また どういうテキストがおすすめでしょうか?
0061132人目の素数さん
2019/09/15(日) 17:12:31.27ID:feoQolBD
>>58
> Mathematical Logic だね

だよねー
だったら新井さんも日本語タイトルは「数理論理学」にしておけよって話

日本だけだよ、大昔ならいざ知らず現代になっても
“foundations of mathematics”(に相当する言葉)を“mathematical logic”(に相当する言葉)と全くの同義語かの如く使うのは

新井さんが英語タイトルを『数学基礎論』に相当する“Foundations of Mathematics”でなく“Mathamatical Logic”にしてるってことは
彼は日本だけの非常識さを知ってるってことだ
にも拘わらず日本語タイトルでは相変わらず羊頭狗肉な『数学基礎論』を使うって実に質が悪いとしか言い様がない
0062132人目の素数さん
2019/09/15(日) 17:19:21.85ID:g2F0dADR
>>61
>羊頭狗肉な『数学基礎論』

私は逆の認識ですけどね

「数学基礎論」って数理哲学みたいじゃないですか
でも中身は完全に数学 むしろ狗頭羊肉

ということで、あなたとは全く逆の考えから
日本語タイトルは「数理論理学」にしておいたほうがよかった
と思っています
0063132人目の素数さん
2019/09/15(日) 17:39:28.35ID:p066SErf
三段論法みたいに人間が直感的に信じてる論理って誰が用意したんだろう
0064132人目の素数さん
2019/09/15(日) 23:01:09.17ID:950Jq7/Q
>>59
文意を把握できない
京都賞と数理論理学の関係をどうとらえているの?
0065132人目の素数さん
2019/09/15(日) 23:43:37.10ID:8v10KzTl
>64
失礼。思い切り思い込みでてきとー書いちまった。
日本数学会賞。
0066132人目の素数さん
2019/09/16(月) 08:35:18.65ID:4OYL0rf4
あげ
0067132人目の素数さん
2019/09/16(月) 21:50:23.73ID:r30lSyGN
>>62
貴方がどちらを高級と考えているかはこの議論に全く関係ない
日本のその分野の研究者たちの間では「数学基礎論」のほうが「数理論理学」より高級だと思われている、という事実だけが重要

彼ら自身の間ではそう思われていることの証拠は、彼らの研究分野の英語名がmathematical logicであるにもかかわらず
日本語では自分達の分野を今でも数学基礎論と呼ぶケースが現在でも極めて多いからだ
つまり数学的であること以上に「(数理)哲学」的であることを高級だと思っているということだ

これは私の個人的な推測であり根拠も証拠もないが、彼らは自分達の分野が恐らく「数学とは何かの本質に迫っている」とでも思いたいから
「数理論理学」という単純に技術的な手段と研究対象とから成る分野名でなく「基礎論」という言葉を有難がって今も使いたがるのだと思われる

だから新井さん本人を含め彼ら自身にとっては「数学基礎論」という呼び名のほうが「数理論理学」よりも高級な名前なのですよ
貴方が逆だと思おうともね
0069132人目の素数さん
2019/09/16(月) 23:17:21.38ID:vMn/CWib
>>62
数理論理学は羊頭狗肉
0070132人目の素数さん
2019/09/17(火) 00:14:34.06ID:yGySqh6n
論理主義者は数理論理学=基礎論って考えになるんじゃないの?
0071132人目の素数さん
2019/09/17(火) 07:20:24.31ID:bNUmN2st
論理主義者ならそもそも数学より前に論理学があることになるから、
数学を前提とする数理論理学は基礎にはならないな

正直俺は数学基礎論を日本にしかない別名程度にしか思ってなかったけど
0072132人目の素数さん
2019/09/17(火) 13:41:42.79ID:xUSjznUd
>>71
> 正直俺は数学基礎論を日本にしかない別名程度にしか思ってなかったけど

「数学基礎論」に相当する言葉は海外にもあるぞ、例えば英語ならば“foundations of mathematics”だ
そして、英語圏ではこの言葉はこの言葉に相応しい内容(数学の基礎付けという数理哲学的な色彩の濃い内容)を指したり
そういう内容の書籍のタイトルとして使用されるのが普通だ

“foundations of mathematics”(に相当する言葉)を“mathematical logic”(に相当する言葉)の別名あるいは同義語として
平気で広く使っているのは私の知る限り日本だけだ
0073132人目の素数さん
2019/09/19(木) 19:59:21.20ID:7GQwcv+X
>>72
>“foundations of mathematics”を
>“mathematical logic”の別名あるいは同義語として
>平気で広く使っているのは私の知る限り日本だけだ

竹内外史が悪いw
0075132人目の素数さん
2019/09/20(金) 13:22:10.35ID:KyAOfC1j
2215
かずきち@dy_dt_dt_dx 8月28日
学コン8月号Sコース1等賞1位とれました!
マジで嬉しいです!
来月からも理系に負けず頑張りたいと思います!
https://twitter.com/dy_dt_dt_dx
https://twitter.com/5chan_nel (5ch newer account)
0076132人目の素数さん
2019/09/26(木) 03:10:04.18ID:exvNnjtl
Sierpinskiの“Cardinal and Ordinal Numbers”について質問です。
第1版と第2版とで内容はどの様に違っているのでしょうか?
(ページ数に関しては487pp.と491pp.なので4ページしか増えていないようなのですが)

御存知でしたら教えて頂けると助かります。宜しくお願い致します。
0077132人目の素数さん
2019/11/13(水) 16:43:06.33ID:kY12vs/x
(P∨Q)∧(P∨R)├P∨(Q∧R)
この証明ってレモン方式の自然演繹でどう記述すればいいんでしょうか?

初歩的なことなんでしょうけど、どうしてもわからないのでどなたかお教えください。
よろしくお願いします。
0078132人目の素数さん
2019/11/13(水) 19:20:42.56ID:m5WqW6pw
>>77
長いよ
0079132人目の素数さん
2019/11/13(水) 23:27:43.34ID:m5WqW6pw
(P∨Q)∧(P∨R)|-P∨Q
(P∨Q)∧(P∨R)|-P∨R
P|-P∨(Q∧R)
¬P,P∨Q|-Q
¬P,P∨R|-R
Q,R|-Q∧R
Q∧R|-P∨(Q∧R)
を組み合わせてね
爆発律と排中律を使うし
0080132人目の素数さん
2019/11/14(木) 07:29:12.76ID:KCoPBPRp
(P + Q)*(P + R)
=P + P*R + Q*P + Q*R
=(P + P*Q) + P*R + Q*R
=P + P*R + Q*R
=(P + P*R)+ Q*R
=P + Q*R
0081132人目の素数さん
2019/11/14(木) 22:04:41.76ID:UXeyDyw9
>>77
(P∨Q)∧(P∨R)|-P∨Q
(P∨Q)∧(P∨R)|-P∨R
P|-P∨(Q∧R)
Q,R|-Q∧R
Q∧R|-P∨(Q∧R)
0082132人目の素数さん
2019/11/19(火) 13:24:15.65ID:ngZlj00P
P→QはあるのにP←Qはないの?P↔QはあるよねP←Qを見かけないんだけど?
0083132人目の素数さん
2019/11/19(火) 13:28:20.38ID:ngZlj00P
P←Qがあれば(P→Q)←R=P→(Q←R)とか分かりやすいのに
0084132人目の素数さん
2019/11/19(火) 20:33:08.62ID:5VBxKFYH
>>83
←があれば便利だと思うのならば自分で好きに定義して使えば良いじゃない、例えば

 P←Q def= Q→P

とね

ついでに言えば、自然言語の文法を型理論的に分析するLambek Calculusだと
上の→と←との各々に対応する型構成子がある
“Lambek Calculus”とか“Categorial Grammar”とかで色々と調べてみてごらん
0085132人目の素数さん
2019/11/20(水) 11:54:20.91ID:JRZAHV8l
>>84
>P←Q def= Q→P
読み方は
「なんとなれば」
でいいかな?チョット長い
「なしか」
でどうかな
0086132人目の素数さん
2019/11/20(水) 12:02:45.35ID:s2bhFDG0
英語ではすでにifとonly ifでわけてるんじゃないですか?
0087132人目の素数さん
2019/11/20(水) 12:15:54.84ID:JRZAHV8l
P if QはP←Q
P only if Qは?if and only ifで成語となると思うんだけどP only if Q単独で¬P←¬Qの意味になる?
0088132人目の素数さん
2019/11/20(水) 12:23:33.43ID:s2bhFDG0
P only if Q
Pが成り立つのはQが成り立つ時に限る

P→Qてことですよね
0089132人目の素数さん
2019/11/20(水) 13:23:26.20ID:JRZAHV8l
自分はif and only if (iff)以外の単独でonly ifが使われたのみたときないけど
それはそれとして
P only if QがP→Qであるとしたら
only if=ならば
if=?
0090132人目の素数さん
2019/11/20(水) 13:28:47.76ID:s2bhFDG0
屁理屈こねる前に素直に英語の意味考えてみればああそうだなってなりますよね

if← and only if→=⇆

これだけですよね
0091132人目の素数さん
2019/11/20(水) 15:03:54.46ID:JRZAHV8l
>>90
屁理屈って・・・
if and only ifが⇔の意味であることは良いんだって
only ifが単独で使われたのみたときないのと
それはそれとして
only if=ならば
となるから
if=?
と聞いてみただけ
0092132人目の素数さん
2019/11/20(水) 15:44:28.18ID:BZyGNV5o
The logical relation is, as before, expressed as "if P, then Q" or "P ⇒ Q".
This can also be expressed as "P only if Q", "P implies Q" or several other variants.

Necessity and sufficiencyのwikipediaの記事で上の通り言ってるぞ、“only if”
0093132人目の素数さん
2019/11/28(木) 00:56:03.70ID:P3RwT7Zb
ゲーデルの不完全性定理と複素平面上のガウス驚異の定理は関係ありますか?
ヒルベルトはどっちも理解していたはすですが、関連させて研究したひとは
カレーが好きですか。
0094132人目の素数さん
2019/12/06(金) 07:22:52.64ID:a5FaM1Ty
>>93
>ゲーデルの不完全性定理と複素平面上のガウス驚異の定理は関係ありますか?
ないですね
0095132人目の素数さん
2019/12/19(木) 17:53:26.82ID:QR5dZAZ5
¬(P⋀R)を口頭で言うとき、どういえばP⋀¬Rと区別した言い方になりますか?
0096132人目の素数さん
2019/12/19(木) 18:09:49.47ID:Z0fx1z4F
のっとかっこぴかつきゅかっことじ

日本語で言いたい?

ぴかつきゅということではない

とかでは?
0098132人目の素数さん
2019/12/19(木) 21:54:55.49ID:V+OT4hGF
>>94 95 96 97
RをQに読替とは、知能が宇宙人LEV.だ

【予想される超模範怪答】
口頭 ⇔ 論理式
Pで、Q以外 ⇔ P∧¬Q
PとQ 以外 ⇔ ¬(P∧¬Q)
PやQ 以外 ⇔ ¬(P∨Q)
Pや Q以外 ⇔ P∨¬Q

【予想されるトンデモ反論】
スペースは、はずせ。
日本語がワカッテない。グダグダ
0099132人目の素数さん
2019/12/19(木) 22:33:21.05ID:Z0fx1z4F
>>98
¬はノットであって以外では無いのは意外だろうか
0101132人目の素数さん
2019/12/21(土) 06:15:38.35ID:EWAlKOof
【あるコーヒー店での注文時の話】

店員「お飲みの場所、お席ですか?」
ポク「店外でないです 店内です!」
店員さんはニコリとし、会計を終えた
コーヒーの注文に今日も成功したゼェ

ちょっと待てよ。変な日本語だよな。
普通の日本語は、普通は、
「店内ですか」とヒアリングだ。でも
「お席ですか」とヒアリングしてきた。

30分程で、コーヒー飲み終え、
カフェインで脳内が活性化だ。閃いた
店内∨お庭 ⇔ お席
¬(店内∨お庭) ⇔ ¬お席
お庭 ⇔ 店内なのに店外 あらっ

さて、店内∧立呑なら安くなるか
任意の時に、お質問しようと思う
0102132人目の素数さん
2019/12/21(土) 06:29:21.63ID:VHHrP6Aa
お庭 ⇔ (店内∨お庭)∧¬店内 ⇔ お席∧¬店内 ⇔ お席∧店外
0103132人目の素数さん
2019/12/22(日) 01:32:37.21ID:j6fusz3e
馬鹿な質問∨馬鹿な小咄 ⇔ 実に馬鹿
0105132人目の素数さん
2020/01/02(木) 13:12:37.47ID:6x9LHOCh
日本語の「証明図」って英語にすると何?
proof diagramで検索すると明らかに違うんだが
0107132人目の素数さん
2020/01/25(土) 22:50:55.03ID:Ua59xAdh
sage
0109132人目の素数さん
2020/01/26(日) 11:51:01.86ID:SMdiCgm3
> どうも、M_SHIRAISHI氏(つーか、EURMS)の理論」のほうが正しいようだな。

> 例えば、対偶律は、従来は、 (P⊃Q)⊃(¬Q⊃¬P) で表わされるもののこと
> と考えられていたのだっただが、これは、どうやら、誤りだったようだ。

> そして、M_SHIRAISHI氏の言う[P(x)⇒/x/Q(x)]⇒/p,q/[¬Q(x)⇒/x/¬P(x)]
> こそが【対偶律】を正しく捉えてたものと考えられる。

> M_SHIRAISHI氏(たち?)の主張する Logical Reformation は、おそらく、世界を
> 席巻することとなろう。
0110132人目の素数さん
2020/01/26(日) 13:41:42.04ID:4rUhrACw
>>109
革命w
0112識者
2020/02/05(水) 04:04:57.42ID:/9NeCVlz
M_SHIRAISHI氏の恐るべきは、様相論理、時間論理、等々のすべてを熟知した上で、あの理論(=RL)を提起していることである。
0113132人目の素数さん
2020/02/05(水) 21:19:42.16ID:U6Mz6cYg
久しぶりに見に来たらちょっとレスが進んでたな

つい最近数理論理学の書籍をアマゾン検索したら、割と新しいのにこんなのが見つかった

数学基礎論序説: 数の体系への論理的アプローチ
逆数学:定理から公理を「証明」する
コンピュータは数学者になれるのか?
確かさを求めて―数学の基礎についての哲学論考

他になんかつい最近出た本や、知られてないけど良い本あったら教えてくれ
(ただし和書で。和訳OK)
0114132人目の素数さん
2020/02/05(水) 22:10:24.26ID:3+KA+4Zb
『コンピューターは数学者になれるか』は矢田部俊介?(と思われるツイッターアカウント)が推薦してたな
0115Eukie M SHIRAISHI
2020/02/06(木) 04:53:03.34ID:/BMtiW8q
論理学は、諸学の究極の基礎である。

Logic (RL) is the utomost foundation of all of the sciences including Theology. w
0116132人目の素数さん
2020/02/06(木) 07:14:24.50ID:Z/9oCOx7
SAT成果はなぜ公表されないのだろう。
0118132人目の素数さん
2020/03/05(木) 18:48:18.29ID:ermcA2ad
http://www.cs.tsukuba.ac.jp/~mizutani/grad_school/logicSS/2013/2013-06.pdf
演習問題
以下の様相論理の式を証明せよ
1. |-K □p∧□q⊃□(p∧q)
これの解答どなたかたのむ
0119132人目の素数さん
2020/03/05(木) 19:55:04.05ID:DgjKazw0
方針は以下かな
p⊃(q⊃p∧q)
□(p⊃(q⊃p∧q))
□p⊃□(q⊃p∧q)
□p⊃(□q⊃□(p∧q))
□p∧□q⊃□(p∧q)
0122sage
2020/03/05(木) 22:46:28.71ID:ermcA2ad
>119
助かった。ありがとう
0123132人目の素数さん
2020/03/05(木) 23:15:09.95ID:5A6NAdOC
>>118
Qって証明されている命題の全体みたいなもの?
◇って¬□なの?
0124132人目の素数さん
2020/03/13(金) 06:01:48.68ID:cKjRDu8I
MIP*=RE ?
0125132人目の素数さん
2020/03/15(日) 17:25:20.36ID:GHmGgnL/
哲学科の公開セミナーだとあんな適当なこと言っても怒られないのか
勉強になった
0126132人目の素数さん
2020/03/15(日) 18:26:40.03ID:v+yfiMnW
>>125
くわしく
0127132人目の素数さん
2020/03/16(月) 14:11:10.39ID:gQed3o6a
>>126
シーケント計算が(複数の)前提と複数の帰結からなっていることを「主張すること」と「否認できないこと」の組と読み替えて
古典論理のゲンツェン流のシーケント計算の推論規則(右→と左→)が主張と否認の間で命題を移動させることからそれを複数人での議論に見立てた
で古典論理について研究したPeirceがプラグマティズムを提唱したことから、その「複数人間での議論」というところで古典論理からプラグマティズムを見出したんじゃないかって
でもジャンプがありすぎるしシーケント計算って歴史的にプラグマティズムよりかなり最近だよね
0128132人目の素数さん
2020/03/16(月) 15:43:51.91ID:xw7qN3/R
>>127
プラグマティズムって知らんけど(知りたくもないけど)
その人が提唱したことがシーケント計算から由来すると推測しているということ?
例え類似のものがあったとしても関連があるかどうか特にこれが元になってそれができているのかどうかは
そうとう精密な調査検証が必要なことかも知れないのに
そういうことって言わないんでしょ?
現代思想とか全部そう
まるで古代宗教と変わらない
まるで厳密性のないなんらかの主張をしたいがために
確固とした基盤のある事柄を木に竹を接ぐように引用する
引用でも何でも無いのにあたかもそれが正しいかのように
怒られないのはみんなそうやってるからだよ
0129132人目の素数さん
2020/03/16(月) 15:55:40.96ID:3d8xU5zI
俺も素朴にイデアの実在信じてるピタゴラス教団の末裔だからよくわかるわその気持ち
0130132人目の素数さん
2020/03/16(月) 16:19:50.87ID:gQed3o6a
更に言うと直観主義論理との対比で古典論理とプラグマティズムの関係を述べてた訳だけど、ブラウワーが直観主義論理を提唱したのは1920年代であってPeirceが亡くなった後なのでやっぱりズレてる
古典論理しか研究されてなかった時代なんだから古典論理からプラグマティズムが出てきたと見なされるのはある意味自明で、
それはシーケント計算云々は関係なくて、当然その時なかった直観主義論理から何かが出てくるはずもないのでその時点で古典対直観主義の対立があったかのように言ってたのも欺瞞
0131132人目の素数さん
2020/03/16(月) 16:34:00.97ID:gQed3o6a
適当に書いたらPeirceだけアルファベットで他はカタカナになってしまった
パースな
0132132人目の素数さん
2020/03/16(月) 17:23:53.73ID:gQed3o6a
数学寄りのロジシャンが解釈とか思想史をやりたがらないし哲学畑の人でロジックをやりたがる人も少ないから中途半端な分野になってツッコむ人が少ない
プラグマティズムなんて知りたくもないっていう態度も哲学寄りのロジックが無法地帯になってる原因の一つだよ
0133132人目の素数さん
2020/03/28(土) 22:35:11.10ID:fPhLCIQv
http://www.asakura.co.jp/books/isbn/978-4-254-11723-3/
シリーズ: 基礎数学シリーズ 23
数学基礎論入門 (復刊)
B5/216ページ/2006年03月20日
ISBN978-4-254-11723-3 C3341
定価3,740円(本体3,400円+税)
前原昭二 著

これって良書ですか?
0136132人目の素数さん
2020/03/28(土) 23:45:06.74ID:uArgyPDH
教科書の中にお気持ちを書いちゃいけないってルールだと竹内外史もアウトになりそう
0137132人目の素数さん
2020/03/29(日) 09:27:28.88ID:462mPqWO
>>135

その本ですが、非常に分かりづらい本ですよね。
やっぱり著者が悪かったんですね。
0138132人目の素数さん
2020/03/29(日) 16:24:37.67ID:mVS6e59j
>>135
なぜそう定義したのかを説明するのに「そのように定義されているから」じゃ的外れ

>∀導入規則において F(x) が a を含んではいけない理由は
>∀導入規則がそのように定義されているからに過ぎず、
>そうすることが正しいと思われる主観的な理由を書き連ねることに
>証明論としての意味は無い。それは、単なる感想に過ぎない。
0139132人目の素数さん
2020/03/30(月) 21:37:13.27ID:L2N5YItY
もしかすると
無限集合の場合のみならず
一般の集合でも全称量化された集合の元は取れない
という仮説が正しいかも知れん
まだ検討中だ
0140132人目の素数さん
2020/03/30(月) 21:40:27.03ID:L2N5YItY
全称量化された集合の元を取れると定義した

という公理主義はいただけないが
そう書く他はないというのが現状だろう
さてどうするかは考え中だ

有限集合の場合でも全称命題を書くことができない場合を考えている

すべてのカラスは黒い

これは有限集合の話だとすると
やはり一般の集合でも全称量化したものは記述できない
というのが正当であろう
0141132人目の素数さん
2020/03/30(月) 21:53:41.86ID:L2N5YItY
写像の場合を考えてみよう

現状は

∀x∈X, ∃1y∈Y; f(x)=y, f:X→Y

と書くがこれに全射を仮定して

∀y∈Y,∃x∈X; f(x)=y

と書けば既知の集合Yの元とxを対応させればYの元をすべて書くことはできる
がそもそも論理学的には全称命題の不存在性よりYの元に存在性はない
あくまでも数学的にはYの元を書けるということと存在することは等値である
という立場に立つことも可能ではあるが
存在が保証されていないものを「在る」というのはいささか不合理である

ここでたとえば

∃x∈X; ∃y∈Y; f(x)=y

とすればこれは実質的に有限集合を扱っていることになる
さて上記の写像はyの唯一性より変数xの範囲を制限するものであると
考えられてきたがXの元が変数ではなく定数であると考えると
そもそも写像(関数)とは何だったかの問題になってしまう
他方

人間は考えられる範囲しかわからない

という立場からはこれでも十分であるとも言える

存在が担保されていない場合でもそれを言い尽くさなければならない

というものが必要なのかどうかを検討されたい

俺は高校数学をやってる
これはおそらく位相の問題だと思う
0142132人目の素数さん
2020/03/31(火) 01:17:07.66ID:Oo9Qpl7F
これから先東京をロックダウンする
ただしロックダウンは
誰も予想しないときに突然実施する
ロックダウンはいつ起こるか論ぜよ
0143132人目の素数さん
2020/03/31(火) 01:34:53.90ID:nZVVEKlM
誰も予想しない、つまりいつ起こるか論じていないとき
0144132人目の素数さん
2020/03/31(火) 02:41:56.25ID:+LMTnMxG
経験則
・体は3ヶ月で慣れる@
・人の噂も75日A
を用いる

今年の3月からマスメディアで
連日コロナの情報が伝えられたとし
@よりそれに慣れるまで3ヶ月掛かるとする
さらにコロナの情報に慣れてからAより2ヶ月半で
コロナの情報に無頓着になるので合計6ヶ月で
コロナパニックは起きにくくなると推測される

以上より今から約6ヶ月後にロックダウンが施行される
但しロックダウンが今から約半年後に起こることを予測していた者は
パニックになる惧れがあるので注意を要する
0145132人目の素数さん
2020/03/31(火) 07:53:39.19ID:+LMTnMxG
全称命題について
たとえば不等式の三者択一の法則において
すべてのa,b∈R(実数)に対して

a < b または a=b または a > b

の何れか一つが成立する
が在るがこのとき全称命題の不存在性から
aやbに具体的な数値を代入することはできない
それゆえこの具体例として
2≦3や5≦5を挙げることはできない

ではこの全称命題の意味は何かというと

∀x∈Rに対してある定数nが存在して

nx > 1 ⇒ x > 1/n

というようにxの一次不等式を解くときに用いる
このときxの値は実質的に無いのでxに数値を代入することはできない
これに対していまnは定数なのでたとえばn=1のとき
すべてのx∈Rはn=1のとき1より大きいということが保証されている
をいう
0146132人目の素数さん
2020/03/31(火) 07:58:36.22ID:+LMTnMxG
また大小比較をする場合に
すべての実数a,bについて

a≦bを示したいときに三者択一の法則から

@a < b
Aa = b

の何れか一方を示せば十分である
但しa,bに具体的な数値を代入することはできない
0147132人目の素数さん
2020/03/31(火) 18:42:30.53ID:+LMTnMxG
関数f(x)に数値を代入するときは全射の前提が必要である

関数f(x):=xとする

x=1のとき f(1)=1
x=2のとき f(
0148132人目の素数さん
2020/03/31(火) 18:45:46.97ID:+LMTnMxG
>>147
x=2のとき f(2)=2
x=3のとき f(3)=3
……
0149132人目の素数さん
2020/03/31(火) 18:51:38.99ID:+LMTnMxG
このように

すべての数値に対してあるxが存在する

の翻訳は

xがfで,ある値を取るときに,fによって出力される値のすべてが,fの値域である

と読む

つまり左から書くが右から読むのである
これは伝統的に写像fを

f:Y ← X

と書いてきたことと関連する

このときのXは定義域でありYは値域である
昔は右から書いて右から読んだのだそうだ
つまり数学書は左から書いてあるが右から読まなければならない
0150132人目の素数さん
2020/03/31(火) 18:53:45.05ID:+LMTnMxG
前に何処かに書いたかも知れないが
この右から書くというのが改められたのが1960年頃であり
数学史的にはごく最近の出来事である
詳しくは

成田正雄『初等代数学』共立出版 1966

を読まれたい
0151132人目の素数さん
2020/04/01(水) 00:07:59.23ID:vf0RBxx6
同じことだが書き直してみた

■写像

f:Y ← X 

すべてのx∈Xに対して あるy∈Yが唯一つ f(x)=y をみたすように存在する

翻訳

あるy∈Yが f(x)=y をみたすように 唯一つ存在し それがfによって すべてのx∈Xに対応すると書ける



Y:={1,2,3}

f(x):=x

とする.このとき

f(x_1)=1
f(x_2)=2
f(x_3)=3

となるすべてのx_1,x_2,x_3∈Xを書ける(全射).

また

f(x_1)=1
f(x_2)=1
f(x_3)=2

などでもよい(全射でない).


■全射

すべてのy∈Yに対して あるx∈Xが f(x)=y をみたすように存在する

翻訳

あるx∈Xが f(x)=y をみたすように存在し それがfによって すべてのy∈Yの値として書ける



X:={1,2,3}
f(x):=x

とする.このとき

f(1)=1
f(2)=2
f(3)=3

である(とくに全単射).これより値域は

Y:={1,2,3}

であることがわかる.
0152132人目の素数さん
2020/04/01(水) 00:09:25.67ID:vf0RBxx6
現代に合わせれば写像は

f:X → Y

と書いた方が望ましいだろう
0153132人目の素数さん
2020/04/01(水) 00:11:30.41ID:vf0RBxx6
>>151
>となるすべてのx_1,x_2,x_3∈Xを書ける(全射)

訂正 全単射
0154132人目の素数さん
2020/04/01(水) 00:17:37.44ID:vf0RBxx6
訂正
写像の翻訳
あるy∈Yが y=f(x) をみたすように 唯一つ存在し それがfによって すべてのx∈Xに対応すると書ける

全射の翻訳
あるx∈Xが y=f(x) をみたすように存在し それがfによって すべてのy∈Yの値として書ける
0155132人目の素数さん
2020/04/01(水) 08:39:27.76ID:ibjGMPr7
写像f:X→Yとはf={(x,y)∈X×Y | 2つの条件}のことだぞ
それ以上でも以下でもない
0156132人目の素数さん
2020/04/01(水) 09:20:16.38ID:vf0RBxx6
>写像の定義はそれ以上でも以下でもない

これじゃあ話になりませんな
まあ少しでも議論できたらよいなと思って書き込んだのだが
意味がなかったようだ
俺はこれで立ち去るわ
じゃあな
0157132人目の素数さん
2020/05/11(月) 21:42:44.61ID:FABjWISR
ここの人たちはこのスレで議論されている逆接、「しかし」、「であっても」を論理学として扱うというのはどう思う?

数学記号を考案・改良するスレ
https://rio2016.5ch.net/test/read.cgi/math/1582284855/l50
0158132人目の素数さん
2020/05/13(水) 10:31:13.41ID:amMseTLl
神戸大学で数理論理学を専攻して博士を取りたい場合、学部から神戸大学に行くのがいいですか?
それとも学部は京大など旧帝や東京工業大に行ったほうがいいですか?
お勧め大学があれば教えてください。
0159132人目の素数さん
2020/05/13(水) 10:53:07.73ID:EIyANIud
あくまで個人的な意見ですが、
東大や京大に行くとレベルが高い人がいて気圧されることもありますね
(数理論理学では就職先が研究者くらいしかないでしょうし、そういう意味では神戸大学の方が心が折れにくいでしょう)

そんな性格でなければ、例えば東大には新井敏康先生とかいらっしゃいますし、感情抜きでは東大京大の方が研究職に就きやすいのではないでしょうか
0160132人目の素数さん
2020/05/13(水) 11:03:37.41ID:YxiDM0Si
>>158
入れるところに行けば?
0161132人目の素数さん
2020/05/13(水) 11:08:06.29ID:YxiDM0Si
>>159
周りがどうこういう以前に自分の意欲と努力の問題だけどね

よく「まだ本気だしてないだけ」っていう人いるけど
そもそも本気だせない時点で意欲が欠如してるわけで
そういう時は自分の内なる声に耳を傾けて
別の道に行ったほうがいいよね
0162132人目の素数さん
2020/05/14(木) 13:39:19.92ID:laTGt7ex
数理論理も、数論幾何とか関数解析とか位相幾何みたいな
「ふつうの」数学の分野も、多分就職は
そう変わらないのであまりそう言うことは考えなくて良い。

2012年以降はこれまでは就職はそれなりに良かったけど
今後は企業の倒産とかが続いて就職先が減ってくる
可能性があるので、それは留意した方が良いけど
0163132人目の素数さん
2020/05/14(木) 17:11:27.06ID:yUsAr7Ai
そもそも就職考えるんなら数学科に行かないほうがいいけどね
0164132人目の素数さん
2020/05/14(木) 19:16:10.94ID:yMurV5iC
産業界からは世捨て人と呼ばれている。
0165132人目の素数さん
2020/05/24(日) 23:03:11.18ID:OpZXQBtX
wikipediaの型理論のページに
「型理論の詳細はホワイトヘッドとラッセルの 『プリンキピア・マテマティカ』にある。」
って書いてあるの古すぎじゃないですか?

外国ではHoTTとかやっているというのに
0166132人目の素数さん
2020/05/24(日) 23:11:42.06ID:p1hbh81S
プリンキピアマテマティカ読んで数学を勉強してるやつなんて今どきいないっしょ
0167132人目の素数さん
2020/06/29(月) 22:13:56.24ID:A6HnHqWU
いけねえ
知らぬとはいえ
俺はフェイトから知識を盗むところだった
だからあいつは強情だったんだな
謎の上から目線で人の話は聞かないという姿勢
それは情報を守るためだったんだな
よくわかったよ
今までありがとうな
またどこかで会おう
0168132人目の素数さん
2020/09/10(木) 21:25:33.88ID:09TJq5ZY
背理法不要論は数学基礎論の範疇ですか
0169132人目の素数さん
2020/09/11(金) 10:15:35.74ID:ymVLBOEu
背理法不要論という立場自体は、信念の問題で、いわゆる基礎論の範疇ではありません。
ですが、一般に背理法が成立しない証明体系はあり(直感主義論理の証明体系など)、その体系の数理論理学・証明論的な研究は範疇になりますね。
そういった研究は、もはや化石のようなものでしょう。
0170132人目の素数さん
2020/10/27(火) 02:07:09.31ID:Qpi1JQGL
別に化石ではないと思うけど。
数学の中心的な潮流かとか言われたらアレだけど
0171132人目の素数さん
2020/11/07(土) 01:10:49.70ID:5UHV1VuJ
幾何学的モデル理論のamazonレビュー、
なかなか的外れでアレだな
0178132人目の素数さん
2020/12/21(月) 23:05:58.17ID:slb93lrJ
Pを仮定して矛盾が出たら¬Pを結論するのは背理法じゃないんだよなあ
これはチョ環主義論理でも問題なく使える
0179132人目の素数さん
2020/12/21(月) 23:28:12.69ID:eRMpoYlk
素朴集合論
0180132人目の素数さん
2020/12/22(火) 11:21:28.41ID:l21dM6jc
点集合論
0181132人目の素数さん
2020/12/22(火) 17:26:41.73ID:4zoIFivU
>>171
氏のレビューは鵜呑みはできない部分もあるね
カリフォルニア州立工芸大学(全米屈指のエリート校)の現数学科名誉教授の本を読み「あまり理解してる人ではなさそう」と評してる(当然、理解してないなどと思われるような部分はない)
ただ圏論についてなんかのレビューとかは参考になる部分も多い
0182132人目の素数さん
2020/12/22(火) 18:38:59.85ID:SzaK3Ywi
>>181
評価に値するところなどどこにもない。本文を読んでいたらあんな出鱈目なレビューは書けない
0183132人目の素数さん
2020/12/22(火) 22:17:13.50ID:RfMIjE2z
論理学やり始めたばかりなんですけど、真理表とタブローの木と自然演繹って何が違うんですか?同じことの別表現?
0184132人目の素数さん
2020/12/22(火) 22:58:56.96ID:l21dM6jc
かわいそうに
0185132人目の素数さん
2020/12/22(火) 23:00:17.20ID:l21dM6jc
ここ笑うとこ?
>カリフォルニア州立工芸大学(全米屈指のエリート校)
0186132人目の素数さん
2020/12/22(火) 23:07:46.62ID:l21dM6jc
まさかcatechの間違い?
0187132人目の素数さん
2020/12/22(火) 23:08:50.83ID:l21dM6jc
caltechだったw
0188132人目の素数さん
2020/12/22(火) 23:10:21.75ID:8IPH7YoJ
カルテックのことか?

ゴメン、おれカリフォルニアなら
UCバークレーしかしらんわw
0189132人目の素数さん
2020/12/22(火) 23:11:55.86ID:8IPH7YoJ
>>183
>真理表とタブローの木と自然演繹って何が違うんですか?

戸田山和久の「論理学をつくる」でも読んでみて いい本だから デカいけどw
0190132人目の素数さん
2020/12/22(火) 23:16:40.87ID:l21dM6jc
科学哲学の冒険はいいね
0191132人目の素数さん
2020/12/22(火) 23:17:23.16ID:l21dM6jc
論文の教室もいいぞ
0192132人目の素数さん
2020/12/22(火) 23:46:50.11ID:l29QQJeL
>>189
真理値表は命題論理の論理式を真理値関数として表したもの
タブローの木は命題論理の論理式が真(または偽)であることの同値性を表したもの
自然演繹は仮定と結論の連鎖を表現したもの
0193132人目の素数さん
2020/12/23(水) 00:15:58.43ID:QLH9bnSQ
>>189
読んでみるわ

>192
何となくわかったわ
0194132人目の素数さん
2020/12/23(水) 00:51:33.97ID:+TUB+VXn
>>193
真理値表は{T,F}だけでなくN値やファジーや他の付値を付けることも可能
タブローの木は真偽だけ
自然演繹は推論規則や公理を調節可能
0195132人目の素数さん
2020/12/24(木) 23:49:41.55ID:b5EOE98c
自然演繹やった。シークエント計算とかいうのもやった方がいいん?
0196132人目の素数さん
2020/12/24(木) 23:52:46.09ID:b5EOE98c
あ、もうやってた
0197132人目の素数さん
2020/12/25(金) 00:37:46.41ID:gtZA6OGh
>>195
本質的に同じ
0198132人目の素数さん
2021/01/05(火) 12:09:14.49ID:lt2IPKYB
無限論理では
¬∀a1∃b1∀a2∃b3∀a3∃b3…P(a1,b1,a1,b2,…)

∃a1∀b1∃a2∀b2∃a3∀b3…¬P(a1,b1,a1,b2,…)
てことにならないの?(変数はすべて集合Xの元)
これを普通の有限の論理式に書くコトってできないのかな
0199132人目の素数さん
2021/01/12(火) 02:55:33.16ID:4JWb2sP4
論理を語るメタ論理を論じる地の文が精々プレーンな一階論理なのが笑えるところ
0200132人目の素数さん
2021/01/12(火) 20:30:14.11ID:SZEETdnM
iZ-Cのサンプルプログラム書き始めたので見てね。
https://sunasunax.hatenablog.com
0201132人目の素数さん
2021/01/18(月) 22:40:42.19ID:Xvy2vIAP
>>199
意味不明
0202132人目の素数さん
2021/01/19(火) 08:36:59.74ID:OT1YzgW0
>>201
でもないが
0203132人目の素数さん
2021/01/19(火) 09:59:05.91ID:7ty+V44F
>>202
じゃあ意味を教えて

「論理を語るメタ論理を論じる地の文が精々プレーンな一階論理なのが笑えるところ」
何が言いたいんだろう?
0204132人目の素数さん
2021/01/19(火) 12:09:49.65ID:42ng3Osp
横からだけど意味わからないなら少し勉強が足りないと思う
0205132人目の素数さん
2021/03/28(日) 03:27:22.77ID:mKvp33II
丸善出版のスマリヤン三部作と>>133の本で迷ってるんだけどどっちが良書?
0206132人目の素数さん
2021/03/28(日) 08:35:31.73ID:U1XBPI6N
いい加減さが許容できない人はスムリアンの本はやめた方がよい
0207132人目の素数さん
2021/04/01(木) 01:29:03.44ID:sTOYz/XZ
前原本とかやめとけ
お前らほんと凡人が小綺麗にまとめた本好きだな
あんなの読んだら数学のセンス悪くなるって
何歳か知らないけど若いならスマリヤンみたいな天才が書いた本から吸収した方がいい
0209132人目の素数さん
2021/04/01(木) 06:48:20.45ID:ucoTEeNO
すごくたくさんの内容が無理に
詰められた本って、「わざわざ一冊に纏めなくても」
という感じはどうしても出てきちゃうよね

新井「数学基礎論」とかJechとか
Schindlerとかそんな感じ
0210132人目の素数さん
2021/04/01(木) 15:25:44.66ID:PGfVosn9
>>207
センスのある人の本ばかり読んでると高瀬正仁になってしまう
0211132人目の素数さん
2021/04/02(金) 22:38:23.21ID:5xtBVH2m
丸善のスマリヤン不完全性定理(改訳版)は完全に頭おかしい
0213132人目の素数さん
2021/04/03(土) 01:50:17.24ID:+5+47BBd
その本、原著でよんだけどめっちゃよかった
昔の高橋訳がいけてなさすぎたけど改訳でもだめなのか
0214132人目の素数さん
2021/04/03(土) 01:53:57.23ID:C9qOHU9B
スマリヤンは某一般向け書だけ読んだけど頭おかしいのは合ってるよ、とても面白い読み物だった
0215132人目の素数さん
2021/04/03(土) 01:57:00.01ID:RQfig9CE
>>212
問題の解説がない
その上問題の難易度が高い
そりゃ独学で全問解けたら天才だと思うよ
到底人にすすめる本ではない
0216132人目の素数さん
2021/04/03(土) 02:18:31.47ID:+5+47BBd
確かに解説ないのはおかしい難易度だよね
スマリヤン先生は代入なしの方法で簡単に算術化した方法を解説してんのに
演習問題では代入ありの煩雑なやり方を導かせるという
あり得ない投げっぷりだと思った記憶がある
0217132人目の素数さん
2021/05/08(土) 23:17:49.51ID:r1Klbeuf
このスレで質問するのが適切かどうかわからないのですが。

射影幾何の公理に
 公理3 任意の異なる2点に対し,これと結びつくただ1つの直線が存在する
 公理4 任意の異なる2直線に対し,これと結びつくただ1つの点が存在する
があって、この二つは双対とのことですが、
この二つの双対関係はなにか深い理由がある必然的なことなのでしょうか、それとも
単なる偶然のことなのでしょうか?

ちなみに、双対というのも数理論理学の対象なのでしょうか?
0218132人目の素数さん
2021/05/09(日) 17:57:02.53ID:zwxorq/q
>>217
数理論理学で射影幾何の双対の話が出るのは
数理論理学をドライブしていたのが形式主義であることを
強調するときが多いんじゃないかな。
意味抜きで公理だけが重要という意味で。
点や直線が未定義用語という意味で。

>この二つの双対関係はなにか深い理由がある必然的なことなのでしょうか
論理学的な観点「のみ」で話をするならば
射影幾何の公理系が「たまたま」その変換(点を直線と読み替えるetc)によって
不変であるという性質だからだと思います。

>ちなみに、双対というのも数理論理学の対象なのでしょうか?
数理論理学は、極論すれば論理式という記号列の代数的な操作なので
双対という概念はしょっちゅう出てきますが、それは数理論理学が
双対を扱うというよりも数理論理がそのような構造のもとで
展開されているからだと思います。
0219132人目の素数さん
2021/05/09(日) 19:52:46.46ID:AKgWyr84
>>218
丁寧な回答ありがとうございます。

先の2つの公理の双対性はまあ偶然的と思ってもよいのですね。
なお、公理3と公理4は双対という以前に論理的に同値ですね。あとで気づきました。
双対と論理的同値とは微妙に関係しているのかな?

双対は数理論理の対象ではない(数理論理自体が双対的構造になっている)
ということですね。そのあたり数理論理と圏論なんかとは少し違うのですね。
0220132人目の素数さん
2021/05/09(日) 20:11:01.35ID:YS3vxiDw
>>217
>ちなみに、双対というのも数理論理学の対象なのでしょうか?
射影幾何関係無しならandとorの双対性定理とかかな
0221132人目の素数さん
2021/05/09(日) 20:13:19.10ID:YS3vxiDw
>>219
鵜呑みにせず理解深めるよう勉強したらいいよ
0222132人目の素数さん
2021/05/11(火) 15:24:29.33ID:xW0NFNhy
>公理3 任意の異なる2点に対し,これと結びつくただ1つの直線が存在する
>公理4 任意の異なる2直線に対し,これと結びつくただ1つの点が存在する
>なお、公理3と公理4は双対という以前に論理的に同値ですね。あとで気づきました。

趣旨とは違うので蛇足ですが論理的に同値ではないとおもいます
意味・モデルに引きずられてないでしょうか?
公理3は ∀xyPx⊃Py⊃∃!z Lz⊃Rxyz で
公理4は ∀xyLx⊃Ly⊃∃!z Pz⊃R*xyz ですよね
公理3と公理4を同値とするような論理的な導出はないと思います
0224132人目の素数さん
2021/05/12(水) 10:29:14.02ID:fswPqcwM
微妙にまちがえてました。こうですね
公理3は ∀xyPx⊃Py⊃∃!z Lz∧Rxyz
公理4は ∀xyLx⊃Ly⊃∃!z Pz∧R*xyz
0225132人目の素数さん
2021/05/12(水) 12:08:33.22ID:noCchEa2
随分一つの円にどれだけ180°が組まれてるかによるよな
0226132人目の素数さん
2021/05/12(水) 12:49:44.86ID:jFoqblLO
細かいけど"全ての異なる"のニュアンスが抜けてね
∀xy:[(x/=y)(Px→Py)...]

三項関係R*は反射的推移閉包でいいのかな、双対性を初めから組み込んだ公理だね
(ところで三項だとどの組について言及しているのか曖昧じゃない?)

任意の異なる2林檎に対しただ一つの相性の良いビールが存在
任意の異なる2ビールに対しただ一つの相性の良いリンゴが存在

主語zを固定して
リンゴと合う(x, y)、ビールと合う(x,y)は推移的なのでリンゴとビールの相性の連鎖で複雑な幾何学ができる
x,yに対して反射的なので、王林と紅玉がクリアドライに合うなら紅玉と王林もそう、どっちから食べても問題ない
0227132人目の素数さん
2021/05/12(水) 12:55:11.13ID:jFoqblLO
あ、"全ての異なる"を入れると同じリンゴ2つの食い合わせが保証されないのか
飽きるという意味論が自然に導かれた
0229132人目の素数さん
2021/05/12(水) 13:17:36.68ID:fswPqcwM
同一性の話はご指摘のとおり。R*は閉包ではなくてRとは別の三項関係ということ。修正すると
公理3は ∀xyPx⊃Py⊃x≠y⊃∃!z Lz∧Rxyz
公理4は ∀xyLx⊃Ly⊃x≠y⊃∃!z Pz∧Txyz
Px,Lx,Rxyz,Txyzを
M1: 「点である」「直線である」「線上に二点がある」「点は直線の交点である」
M2: 「直線である」「点である」「点は直線の交点である」「線上に二点がある」
と読む二つのモデルM1,M2において公理3と公理4は意味が互い違いになる。
そのような公理系だからこそ、双対性を持つ
0230132人目の素数さん
2021/05/12(水) 13:29:04.07ID:jFoqblLO
>>229
M1とM2が置換(12)(34)で移るからRとTが双対になるのね、納得

*が付いてたからRは多分幾何学に必要そうな推移性と反射性を備えてるんだろうなと推測しました
実際これで必要十分なんでしょうかね?
0231132人目の素数さん
2021/05/12(水) 13:45:20.62ID:jFoqblLO
基礎論はあんまり興味ないんだけど、数理論理スレってないのな
0階1階ファジーあたりはむしろ応用の方が熱いのでは
最も古典的かつ単純なbooleanSATだってよく後援付いてコンペやってるくらいだし
分けちゃダメかな?
0232132人目の素数さん
2021/05/12(水) 13:52:45.20ID:jFoqblLO
同じ違和感抱いてる人居ないかと基礎でレス検索したら>>22で腑に落ちた

仮に日本語として基礎論=数理論理としても、スレの見る限り入り乱れてるので、言葉でなく内容で割るべきかと
0233132人目の素数さん
2021/05/12(水) 13:54:22.13ID:v4bRloCn
>>222
> 公理3と公理4を同値とするような論理的な導出はないと思います

記号式でやると結構ごちゃごちゃするので論理的導出を日本語でやってみます。

(Ax3) あらゆる2点A,B上を通る直線は唯一に限る (「異なる」は省略;以下同じ)
このAx3の否定を仮定する。すなわち、
(¬Ax3) ある2点A,B上を2つの直線L,Mが通る
とする。すると、
(*) ある2つの直線L,Mの上に2つの点A,Bがある
ことになる。これは、
(Ax4) あらゆる2直線L,Mの上には唯一の点がある
の否定である。

以上から、¬Ax3 -> ¬Ax4
同様にして、¬Ax4 -> ¬Ax3
結局、 Ax3 <-> Ax4

なお、お気づきと思いますが、
Ax3の否定(¬Ax3)のときに、
「どの2点についてもそれらを通る直線が無い、ということはない」
という前提をおいていますから、厳密には、公理3と公理4は「論理的に"ほぼ"同値」
と言っておくべきだったかもしれません。

以上どうでしょうか?
0234132人目の素数さん
2021/05/12(水) 16:35:31.76ID:fswPqcwM
具体的な論証をするためにはAx3,Ax4以外の公理を共有しないと
議論できないと思います。ただ、一般的にAx3,Ax4をともに公理と呼ぶなら
それらは互いに独立(少なくとも独立と信じられている)のではないでしょうか?

その上であえて指摘させてもらいます

>(*) ある2つの直線L,Mの上に2つの点A,Bがある
これを Line(L,A,B)かつLine(M,A,B) と書きましょう。そして
>(Ax4) あらゆる2直線L,Mの上には唯一の点がある の否定
これを Cross(L,M,A)かつ Cross(L,M,B) と書きましょう
(アルファベットが異なるものは違うものという規約は維持します)
貴方はこの二つを同じとおっしゃっていますがそこを埋める
公理なり推論規則なりはあるのでしょうか?

直観的には明らかのような気もしますが
Line関係とCross関係を取り持つ公理なりがないと
なんともいえないかなあというきがします。
0235132人目の素数さん
2021/05/12(水) 17:00:31.23ID:jFoqblLO
(同じようなことを(拙く)書いてたら適切なレスが来てた)

俺も>>230で指摘してもらったように、通る(>>219の表現では結び付く)の意味が不明瞭
記号を借りると、RとR*(後にT)をオーバーロードしてしまっている
0236132人目の素数さん
2021/05/12(水) 17:41:30.90ID:jFoqblLO
記号を入れ替えただけの公理があるからこそ、その一方から従った定理と記号を入れ替えた双対定理が他方から自動的に従う

点である-線である (P-L)
cross-at - on-line(R-T)

あといくつか点と線の性質が公理1,2にあるはずで、これも必要
少なくとも>>226で述べたもの
cross-at(a,b,c)=cross(c,b,a) 反射性
…略
0237132人目の素数さん
2021/05/12(水) 17:50:32.99ID:jFoqblLO
補足
公理3,4だけで不十分な理由

cross-at(a,b,c)=cross(c,b,a)
であるのに
on-line(A, B, C)/=on-line(C, B, A)
なら、双対定理は明らかに成り立たない

双対のある体系を指定したいなら、点と線は無定義語であってはならない
少なくともその性質は述語を介して間接的に(かつ整合的に)定義することが要請される
0238132人目の素数さん
2021/05/12(水) 17:54:57.35ID:jFoqblLO
無定義語という言葉は違うニュアンスで使われたりするな、誤解を招きそう、ごめん
点と線自体はリンゴでもビールでもいい、しかし上述のような性質は定義する必要はある
0239132人目の素数さん
2021/05/12(水) 18:16:29.98ID:fswPqcwM
定義とか未定義用語とか意味とかその辺りの言葉遣いやニュアンスが
数理論理の人と数学一般の人とでだいぶ違うきがする
気がするだけだけかもしれないけど
0240132人目の素数さん
2021/05/12(水) 22:23:10.14ID:v4bRloCn
>>234
>具体的な論証をするためにはAx3,Ax4以外の公理を共有しないと
>議論できないと思います。

「どの2点についてもそれらを通る直線がある」
「どの2直線も少なくとも同じ1点を通る」
という2つの公理があればAx3とAx4は同値になると思います。
ただそうすると、公理の個数がむしろ1つ増えてしまいますw
こうして、公理の個数という観点では、元のAx3とAx4の2つの公理の設定が
最もシンプルだったというところに落ち着くのかもしれません。

>ただ、一般的にAx3,Ax4をともに公理と呼ぶなら
>それらは互いに独立(少なくとも独立と信じられている)のではないでしょうか?

一般に、公理群は必ずしも独立である必要はないでしょう。
ただ、上に述べたようにして、元のAx3とAx4の組が最もシンプルなのかもしれません。
それでも、Ax3とAx4の重複感あるいは非独立感がまだぬぐえないのですがw

> Line(L,A,B)かつLine(M,A,B) と〜〜
> Cross(L,M,A)かつ Cross(L,M,B)
> この二つを同じ〜〜を埋める公理なり推論規則なりはあるのでしょうか?

この二つが同じであるためには、他に何の公理も推論規則も必要としません。
Line(L,A,B) <-> on(A,L) and on(B,L)
Line(M,A,B) <-> on(A,M) and on(B,M)
Cross(L,M,A) <-> on(A,L) and on(A,M)
Cross(L,M,B) <-> on(B,L) and on(B,M)
でしょうから。
0241132人目の素数さん
2021/05/12(水) 22:50:08.62ID:fswPqcwM
あまり建設的な議論にならなそうなのでもうやめたいので
これを最後のレスにします

>2つの公理があればAx3とAx4は同値になると思います。
一つ目の公理には 点×点×直線 という三項関係しか登場せず
二つ目の公理には 直線×直線×点 という三項関係しか登場しません
なぜそれなのに、この2つの三項関係間の言明(つまり同値性)が
証明できるというのでしょう?できないですよね、普通に考えて。


>この二つが同じであるためには、他に何の公理も推論規則も必要としません。
>Line(L,A,B) <-> on(A,L) and on(B,L)
>Line(M,A,B) <-> on(A,M) and on(B,M)
>Cross(L,M,A) <-> on(A,L) and on(A,M)
>Cross(L,M,B) <-> on(B,L) and on(B,M)
>でしょうから。

Line関係とOn関係に同値性があるのはなぜですか?他に公理があるからなのではないですか?
Cross関係とOn関係い同値性があるのはなぜですか?他に公理があるからなのではないですか?
0242132人目の素数さん
2021/05/12(水) 23:17:46.98ID:N7QoAxjN
>>171 >>181-182
スティルウェル著、川辺治之訳、田中一之監修「逆数学」の書評がえらいことになってる・・・
0243132人目の素数さん
2021/05/13(木) 10:29:37.58ID:r2BnWzdK
>>241
>Line関係とOn関係に同値性があるのはなぜですか?他に公理があるからなのではないですか?
>Cross関係とOn関係い同値性があるのはなぜですか?他に公理があるからなのではないですか?
あなたのいうLineとCrossの「定義」がそうなのではなかったのですか?
それとも、LineもCrossも派生用語ではなく基本用語のつもりだったのですか?
0244132人目の素数さん
2021/05/13(木) 11:20:48.91ID:Us7LNNuJ
>LineもCrossも派生用語ではなく基本用語のつもりだったのですか?
そうに決まってる。最初からずーっとその話をしてる。
もし派生用語だったら定義する公理がある(Extension by definitions)から出せっつってんのに出さないし。
人が言っていることを聞かないで、特定のモデルを勝手に前提にして、あなたは何を議論したいの?
0245132人目の素数さん
2021/05/13(木) 11:58:45.30ID:r2BnWzdK
>>244
>そうに決まってる。
少なくとも、そうは決まっていないです。決まっているとするのはあなたの頭の中でだけです。

>もし派生用語だったら定義する公理がある
定義する公理w これもおかしいです。定義は定義であり公理とは違います(わかりますか?)
当たり前だと思っていましたが念のためその定義を前に書きました。あの定義はわかりましたか?
0246132人目の素数さん
2021/05/13(木) 12:04:19.02ID:Us7LNNuJ
定義する公理がおかしいっていうのは、数理論理学の勉強が不足しているからです。
キーワードも書いたでしょう. Extension by Definitions でググってください

派生用語を使って公理を記述するならば、その定義となる公理も一緒に提示しなければいけません
勝手に用語に意味を付与して議論する態度こそ、「あなたの頭の中だけ」ですよ

もうすこし形式的な手法という概念を勉強した方がいいとおもいます
0247132人目の素数さん
2021/05/13(木) 12:12:39.62ID:DLjGGXhe
馬鹿2人がバトルしてる
0248132人目の素数さん
2021/05/13(木) 12:33:10.94ID:r2BnWzdK
>>246
>Extension by Definitions
今の話はそれとは関係ありません。大げさに考えすぎです(初学者にありがちですが)

まずは普通の意味での定義と公理の違いを言ってみてくれますか?
(もしかすると言えないのじゃないかな?)
0249132人目の素数さん
2021/05/13(木) 13:31:28.06ID:Us7LNNuJ
話がかみあわなさすぎてわらった.。まあ終わりにしましょう。
あなたは何らかの公理系における2つの公理の同値性を示したと主張した。
それが無謬であれば価値があることだと思いますよ。
0250132人目の素数さん
2021/05/13(木) 14:24:52.36ID:r2BnWzdK
>>249
>それが無謬であれば価値があることだと思いますよ。

いや、そんなことは初等幾何読本の最初に出てくる事でしょう。
そんなことにとらわれて当初の話が進まなかったですね。
0252132人目の素数さん
2021/05/13(木) 23:30:00.38ID:YQoHLcer
大学数学スレに尋ねたけどこちらが適当か思い直したので来ました
写像は直積の部分集合で特別なものと定義するってことだけど
f(a)って書き方はfとaからf(a)を与える写像なんじゃないの?
mathematicaだとapplyだっけなそういうやつ
任意のX,Yに対してf:X→YがX×Yの部分集合の特別なものなら
そのような写像の集合をF={f⊂X×Y|f:X→Y}⊂X×Yとして
apply:F×X→Yって写像を想定しているってこと?
X,Yについてのapplyは写像でそれはF×X×Yの部分集合?
F⊂X×Yだからapply⊂X×Y×X×Yなの?
X,Yは任意の集合だからapplyってのはクラス関数なの?
0253132人目の素数さん
2021/05/13(木) 23:33:32.51ID:YQoHLcer
何が聞きたいかというと
写像を直積の部分集合と定義するのは変じゃ無いかってこと
直積の部分集合で``表現''されるってなら分からんでもないけど
直積の部分集合だとf(a)って書き方をするための写像applyを考えてそれも写像なら直積の部分集合でそれの適用結果apply(f,a)=f(a)もapplyとfとaからf(a)を対応させている写像なんじゃないかと思えていつまで経っても終わらなく無くない?
0254132人目の素数さん
2021/05/14(金) 00:59:12.06ID:09noit3a
そのapplyはvaluation mapと呼ばれる写像です
0255132人目の素数さん
2021/05/14(金) 01:21:16.36ID:H04HmXci
>>f(a)って書き方はfとaからf(a)を与える写像なんじゃないの?

ZFCには2変数関数記号 ` が導入されており、
f`x = { z | Func(f)∧∃y(z∈y∧<x,y>∈f) }を満たしている。
(これによってクラス関数`が定義されていると考えて良い)
ちなみに、Func(f)は∀z∈f∃x,y <x,y>∈f∧∀x,y1,y2[ <x,y1>,<x,y2>∈f⇒y1=y2]という述語

>>253
その通り
だから、クラス関数というクラスを定義域とする大きな関数を考えている。
だから、クラス関数 ` の引数には任意の集合が入って良い
で、fが写像でxがそのfの定義域に属しているならば、f`xは<x,y>∈fなる一意に存在しているyとなる。

f`x = { z | Func(f)∧∃y(z∈y∧<x,y>∈f) } が本当にyになっていることは、じっくり考えると良い頭の訓練になる。

y={z|z∈y}という表記が出来ることに注意
0256132人目の素数さん
2021/05/14(金) 01:27:10.91ID:H04HmXci
>>252,253
素朴集合論では、集合X,Yを先に考えて、その後にX×Yの部分集合として写像fを考えるという立場をとっている(のが殆ど)が、
ZFCでは、そんな話の前後関係は一切ない。
X,Yを持ち出すとは無関係に、写像fは定義される。それが>>255
0258132人目の素数さん
2021/05/14(金) 02:30:49.44ID:09noit3a
>>254
valuationやない、evaluationだった
0259132人目の素数さん
2021/05/14(金) 07:19:17.70ID:M4irQGMM
>>255
>ZFCには2変数関数記号 ` が導入されており、
>f`x = { z | Func(f)∧∃y(z∈y∧<x,y>∈f) }を満たしている。
スッキリしました
どうもありがとう
0260132人目の素数さん
2021/05/14(金) 07:29:43.00ID:M4irQGMM
>>255
>fが写像でxがそのfの定義域に属しているならば、f`xは<x,y>∈fなる一意に存在しているyとなる。
写像でないか写像でもxが定義域に属してない場合はf`x={}ということですね
うーん分かりやすいなあ
いや?
ホントにf(x)={}である写像と区別が付かないのでは?付かなくても構わない?
0261132人目の素数さん
2021/05/14(金) 10:30:00.12ID:H04HmXci
>>260
いい視点。

ZFCでは(?)使わなくても取り敢えず広く定義しておくってことは多々ある。
関数記号(クラス関数) ` の場合で言えば、fが関数であり、xがfの定義域に含まれる時に、f`xを用いるが、
fが写像かどうか分からず、xがどんな集合かわからない時に、「f`x=yからf、xが何か?」という逆算するような議論は(全く?)しない

他にもZFCでは順序数αを定義して、例えば、順序数に対して定義されたクラス関数F(α)を定義するが、
(本にもよるが)F(α)は一旦全ての集合に対してF(x)を定義しておく。(xが順序数αでないときはF(x)=φ)
Fは順序数αに対してしか用いないから、xが順序数でない時にどんな値を取ろうが構わない。
0262132人目の素数さん
2021/05/14(金) 10:39:13.70ID:DaLqj8l+
>>251
>表現が双対である以上は必然だわな
よくわかりません。どういうことでしょうか?

ところで、無矛盾性の場合は、それがないと致命的ですが、
双対性の場合、それがあるとなにがよいのでしょうか?
それがないとなにが困るのでしょうか?
0264132人目の素数さん
2021/05/14(金) 17:17:33.96ID:b8QDZzwo
下の問題に対する解答は合っていますか?

問題: Let φ(x) be a formula. What does ∀z∀y((φ(x)∧φ(y) )→ z=y) assert?

解答: z ≠ y ならば φ(x) または φ(y) のどちらかは成り立たない。

こういう問題を試験で出題したとき、採点者は間違っていなければすべて正解にするんですかね?

それとも、普通の日常後に直したときに「自然な」解答でないといけないとか言い出すんですかね?

そうすると、主観が入りますよね。

例えば、

What does the formula ∃x∀y(¬(y ∈ x)) say in English?

という問題の解答を、

ある集合 x があって、任意の y に対して、 x は y を含まない

と解答したら正解でしょうか?

それとも、「x は空集合である」と書かないと不正解になりますか?
0265132人目の素数さん
2021/05/14(金) 20:35:52.02ID:9nkd77MP
> What does the formula ∃x∀y(¬(y ∈ x)) say in English?

in English とあるのだから、答えは英語で書かないとバツだろうねw
0266132人目の素数さん
2021/05/14(金) 22:50:20.88ID:DaLqj8l+
>>264
「空集合(なるもの)が存在する」と書かないと不正解でしょう。
0267132人目の素数さん
2021/05/15(土) 04:17:43.61ID:UD8G4c5/
>>261
ありがとうございました
0268132人目の素数さん
2021/05/15(土) 09:46:27.74ID:29Wrdikr
Set Theory: A First Course (Cambridge Mathematical Textbooks) 1st Edition
by Daniel W. Cunningham (Author)

この本は公理に基づく集合論の入門書です。

例えば、 P <-> Q の定義は、 (P, Q) = (T, T) または (P, Q) = (F, F) のとき、かつそのときに限り T になる

というものです。

以下の公理2つを用いて、 A, B を集合とする。 A ∈ B ならば、¬(B ∈ A) が成り立つことを証明せよという問題があります。

Pairing Axiom:
∀u∀v∃A∀x(x∈A <-> (x = u ∨ x = v))

Regularity Axiom:
∀A(A≠Φ → ∃x(x∈A ∧ x ∩ A = Φ)

この問題の解答を以下のように普通の言葉で書いてもいいのでしょうか?

Pairing Axiomにより、 x ∈ C <-> (x = A ∨ x = B) となるような集合 C が存在する。
この C を {A, B} と書くことにする。
{A, B} ≠ Φ だからRegularity Axiomにより、 x ∈ {A, B} ∧ x ∩ {A, B} = Φ を成り立たせるような集合 x が存在する。
{A, B} の定義により、 (x = A ∨ x = B) ∧ x ∩ {A, B} = Φ を成り立たせるような集合 x が存在する。
A ∈ B ∩ {A, B} だから、 B ∩ {A, B} ≠ Φ である。よって、A ∩ {A, B} = Φ でなければならない。
ゆえに、 ¬(B ∈ A) でなければならない。
0269132人目の素数さん
2021/05/15(土) 09:47:53.38ID:29Wrdikr
この問題の後のページをパラパラ見てみると、この本自体、証明は普通の言葉で書いているようです。
0270132人目の素数さん
2021/05/15(土) 12:13:54.46ID:29Wrdikr
集合論の公理のうち、 Union Axiom は認めづらくないですか?

こんなの認めちゃってもいいんですか?
0273132人目の素数さん
2021/05/15(土) 13:07:08.15ID:29Wrdikr
>>272

やはり、 Union Axiom は強すぎるのではないかと思っている人がいるんですね。
0274132人目の素数さん
2021/05/15(土) 13:42:05.38ID:aUxtAVBn
いやいや、既に持っている集合から、より低下位層(?)の集合を作る公理は何の違和感もないっしょ?
むしろ、既に持っている集合から、より大きな集合を作る公理のほうが違和感を感じるべきっしょ?
要するに置換公理のほうが強すぎると感じるべきじゃね?
0275132人目の素数さん
2021/05/15(土) 13:44:29.20ID:y4q0lZ6C
感覚は論理的に整序されているものではない
0277132人目の素数さん
2021/05/15(土) 17:47:26.61ID:29Wrdikr
松坂和夫著『集合・位相入門』の集合のところで、怪しい話だなと思ったのが、集合族の和集合の話です。
0278132人目の素数さん
2021/05/15(土) 17:48:52.51ID:29Wrdikr
この怪しい話が公理的集合論ではどうなっているのかと思い、

Set Theory: A First Course (Cambridge Mathematical Textbooks) 1st Edition
by Daniel W. Cunningham (Author)

の最初のところを読んでみました。

公理になっていて驚きました。
0279132人目の素数さん
2021/05/17(月) 20:56:22.71ID:2qyGz5PO
どうでもいいけど
普通怪しまれてないよ
何の違和感もない
0280132人目の素数さん
2021/05/17(月) 20:58:20.34ID:2qyGz5PO
置換公理も
まあ
集合の像から切り出すみたいなもんだし
違和感ないなあ
0281132人目の素数さん
2021/05/17(月) 22:23:37.64ID:PJndd6iu
松坂和夫著『集合・位相入門』

p.19に

「1つの集合系 A が与えられたとする。」
「A に属するすべての集合に共通な元全体の集合を、 ‘A に属するすべての集合の共通部分’」

などと書かれています。

A = Φ のときには、 A に属するすべての集合に共通な元全体は集合にはならないので、 A には空でないという条件を課さないといけないはずです。
0282132人目の素数さん
2021/05/17(月) 22:26:37.15ID:0kEgd4uv
添字集合が空集合のときにも交わりと合併を定義する流儀もあるね
0283132人目の素数さん
2021/05/17(月) 23:43:32.54ID:PJndd6iu
Set Theory: A First Course (Cambridge Mathematical Textbooks) 1st Edition
by Daniel W. Cunningham (Author)

この本は公理的に集合論を扱っている本ですが、collectionという用語を使っているのに、その定義がありません。

例えば、collection {x : x = x} は集合ではないというような書き方をします。

これはありですか?
0286132人目の素数さん
2021/05/18(火) 02:12:16.39ID:syya1KmR
そのうち定義という用語が定義されてません、とか言い出すんじゃね
0287132人目の素数さん
2021/05/18(火) 09:10:25.83ID:EpgYI3Qu
collection {x : φ(x)} が定義されていません。
0288132人目の素数さん
2021/05/18(火) 17:56:51.58ID:hdSHz5vt
マツっんは数学の本スレで集合論関連で小馬鹿にされ、散々叩かれてたからなあ。
リベンジすべくここで修行中じゃなかろうか。
0290132人目の素数さん
2021/05/18(火) 18:21:03.86ID:zuLzzuBD
松坂くんはメタレベルの概念、用語に形式的かつ厳密な定義を求めるお馬鹿だから。
対象レベルとメタレベルの区別が分からない様子。
0291132人目の素数さん
2021/05/18(火) 21:06:27.51ID:EpgYI3Qu
上江洲忠弘著『述語論理入門』

論理の本なのに、厳密な本じゃないですね。
0292132人目の素数さん
2021/05/18(火) 23:33:18.21ID:qbyxOTG1
今、竹内外史のAxiomatic set theory読んでるんだが、結構いいね、この本。

竹内外史って数学基礎論の数学者だと思ってたんだが、案外色んな分野の本を出してるよな。
物理にも触れてる本あった。
0293132人目の素数さん
2021/05/18(火) 23:59:26.64ID:rjx81/Uc
ブルーバックス「集合とは何か?」で知ったので集合論プロパーの人と思っていた
0294132人目の素数さん
2021/05/19(水) 06:36:57.46ID:M/rwrJcz
Set Theory: A First Course (Cambridge Mathematical Textbooks) 1st Edition
by Daniel W. Cunningham (Author)

Exercise 2.1.33
The collection {x : ∃y(x ∈ y)} is not a set.

{x : ∃y(x ∈ y)} という記号の定義がこの本には書いてありません。

∃A(x ∈ A <-> φ(x)) であるとき、この A を {x : φ(x)} と書くというのがこの記号の定義だと推測します。

¬∃A(x ∈ A <-> ∃y(x ∈ y)) が真であるのに、 {x : ∃y(x ∈ y)} という記号を書くことは許されるのでしょうか?
0295132人目の素数さん
2021/05/19(水) 06:43:57.95ID:M/rwrJcz
訂正します:

Set Theory: A First Course (Cambridge Mathematical Textbooks) 1st Edition
by Daniel W. Cunningham (Author)

Exercise 2.1.33
The collection {x : ∃y(x ∈ y)} is not a set.

{x : ∃y(x ∈ y)} という記号の定義がこの本には書いてありません。

∃A∀x(x ∈ A <-> φ(x)) であるとき、この A を {x : φ(x)} と書くというのがこの記号の定義だと推測します。

¬∃A∀x(x ∈ A <-> ∃y(x ∈ y)) が真であるのに、 {x : ∃y(x ∈ y)} という記号を書くことは許されるのでしょうか?
0296132人目の素数さん
2021/05/19(水) 08:55:08.74ID:1lgrzgic
>>295
君はこの本を読むより前にやるべき事がある。
もっと易しい邦語の本を読みなさい。
0297132人目の素数さん
2021/05/19(水) 17:00:03.52ID:0TeJLiAk
竹内外史のAxiomatic set theoryの53ページの定理5.8の証明

背理法の仮定を使っていないし、
この証明はハウスドルの証明どころか、<F,T>は極大フィルターがただ1つ、つまりFは一点集合の証明をしてるように見えるんだが?
0298132人目の素数さん
2021/05/19(水) 22:57:23.43ID:ymNnVKCx
>>293
彼は敢えて言うなら証明論の人。
専門外の集合論でも一応米国で話題になっている事を
それなりにフォローしてたのがすごいと思う。

ただ彼の頃はそんなに明確にロジックの
分野が分かれていなかったけど。
0299132人目の素数さん
2021/05/19(水) 23:05:37.15ID:ymNnVKCx
>>294
そこら辺について一番論理的に厳密に書いてあるのは
Azriel LevyのBasic Set Theoryの付録の部分。

{z| Φ(z)} という内包的記法を含んだ式が
どういう場合に許されるかは、ある程度まともな
教科書なら一応書いてあると思うけど、
こういう記法を使っても保守拡大になるだけで
決して新しい問題は起きないという事を
きちんと示したのはShoenfieldで、
その彼の証明が付録に載せてある。
ただ結構晦渋な構文論的な証明なので、
なんだか面倒臭いから、問題ないのは認めて先に進もう、
と思ったのならそれはそれで健全な反応だとは思うけど。

折衷的にNBGの公理系を載せてある本も多い。

Cunninghamという人の本は持ってないから
その人がどう書いてるかは知らん。
0300132人目の素数さん
2021/05/20(木) 01:50:44.09ID:OE04UenE
>>299
定義による拡大が保存拡大になっていることの構文論的証明はかなり厄介って山本新の「数学基礎論」で言ってた
だからこそどんな証明なのか気になるんだが、乗ってるサイトor本ある?


クラスAが集合xに対して、A⊂xならばAは集合
ってのはそんなに難しくない問題
0301132人目の素数さん
2021/05/20(木) 06:08:01.78ID:vBmU8DPI
>>300
>クラスAが集合xに対して、A⊂xならばAは集合
こう?

A⊂x⇔A∈2^xよってAは集合
0302132人目の素数さん
2021/05/20(木) 06:19:44.80ID:hWzPi5ul
普通の数学者はそんな証明を一度も読む事なく「数学」を「定義」を多用しながら「実践」。
保存拡大の結論を篤く信用しているw
0303132人目の素数さん
2021/05/20(木) 06:37:05.29ID:4oRXm9CH
自明、当たり前のことの定義、証明を基礎として、
非自明な結論に至るのが数学の理論組み立ての定石だけどね。
0304132人目の素数さん
2021/05/20(木) 09:12:10.25ID:+rlqC+5j
通常数学はもっと超数学の成果を活用すべきと思うの。
0306132人目の素数さん
2021/05/20(木) 17:46:38.76ID:vBmU8DPI
>>305
いやだからさ同値だって言いたかったわけ
で2^xの元だから集合てのでいいの?って聞きたかったわけ
0307132人目の素数さん
2021/05/20(木) 23:04:20.43ID:OE04UenE
>>301
Aがxの冪集合に属するとだけ言っているだけであって、集合であるとの証明になっていないと思う。
むしろ、部分集合公理によって集合とであるといったほうがより分かりやすい(?)
0308132人目の素数さん
2021/05/20(木) 23:40:49.71ID:OE04UenE
>>306
そもそもZFCにおいては「集合である」という述語は存在しない
だから、∀x[∃y(x∈y)⇒xは集合である]という論法も成り立たない
あえて言うならば、ZFCにおいて「Aは集合である」なる主張は、∃x∀z[z∈A⇔z∈x]とか。
0309132人目の素数さん
2021/05/20(木) 23:51:28.51ID:3AUCosgJ
あのなあZFにおいては万物は集合。
集合で無いものは存在し無い。
存在すると言ったら、それは自動的に集合になる。
0310132人目の素数さん
2021/05/21(金) 01:08:31.80ID:i7zfJXgh
質問

竹内外史、Axiomatic set theory 60ページの定理6.4
b_i = Σ_{a∈A_i} b_{i,a}とかあるけど、
A_iは一体何?
Lの閉論理式Φについて、[[Φ]]を計算する過程で∀xψという部分論理式に対して、[[∀xψ]]を計算する時の話のことなんだろうけど、
定義上、この時はΠの添字は既述のB値構造Aだと思うんだが?
なぜ、一々A_iという新たなB値構造(?)が登場しているんだ?

分かる人教えて下さい
0312132人目の素数さん
2021/05/21(金) 01:57:45.98ID:IgIo5kRo
竹内はいいかげんだから間違いだと思っといたらいい
0313132人目の素数さん
2021/05/21(金) 04:17:13.82ID:TRQhctPt
竹内は確かにアバウト
初心者が読むのには全く向かない
0314132人目の素数さん
2021/05/21(金) 11:59:32.84ID:CuseUkNz
>>307,308,309,311
クラスって概念がある話してるんだけど?
BGだと思ったが違うのかな?>>300
BGだと確かすべてはクラスで
集合はクラスの元じゃ無かったかなと思ったけど
うろ覚えり
0315132人目の素数さん
2021/05/21(金) 12:00:47.53ID:CuseUkNz
>>305
他の定義あるの?
0317132人目の素数さん
2021/05/21(金) 12:33:54.88ID:CuseUkNz
>>316
だからZFでの話じゃないんだって
0318132人目の素数さん
2021/05/21(金) 13:33:56.31ID:i7zfJXgh
竹内外史のAxiomatic set theory読んでるんだが、おかしな点は色々あった

冒頭30ページあたりから何の理もなくいきなり「推移的なZFのモデル」が出てきたり、
順序集合のフィルターの定義が後々の議論に矛盾が出てくるようなものであったり、
B値構造の定義に関数記号の解釈が丸ごと抜けてたり、
>>310のように証明に齟齬みたいなのがあったり、断りなく構造と準同型写像の合成を出してきたり、、

とにかく、一旦書いた後の原稿の推敲がされてない感が強い
数学書は誤字・脱字を直せば終わりじゃねぇんだぞと言いたい
0319132人目の素数さん
2021/05/21(金) 14:12:19.16ID:i7zfJXgh
俺壁にぶち当たったら結構行間埋めるのに何日も掛けるタイプで訓練してきたから、
竹内の本もそのペースでやれば行間を埋めるのは多分出来ると思うけど、ほんとこういう労力って無駄なんだよな

長年訓練してきた数学的読解力って著者の書き損じ、著者の説明下手、著者の言葉足らず/説明不足を補うために使う力じゃねぇんだよな
マジでこういう行間にぶち当たった時につくづく思う。
こういうので行間を埋めるのに何時間~何日も足止めを食らうことが如何に無駄であるか。

著者一人が身代わりとなって徹底的に行間を埋めてりゃ、後続の何十/何百/何千もの読者が同じ足止めを喰らわなくて済むんだよ
山登りと一緒。最初に山を登るやつが、きちんと道標作ってりゃ、後から登る奴は如何にやりやすいか考えろよ
0320132人目の素数さん
2021/05/21(金) 14:27:11.42ID:CuseUkNz
それは勿体無い
0324132人目の素数さん
2021/05/21(金) 17:04:10.66ID:i7zfJXgh
>>323
俺は誤解を防ぐために、イコールを含める⊂は常に⊆使ってるわ

>>322
ブール値モデルな。
モデル論だと真と偽の2値だけど、そこをブール値にしてる点が違い。そういう一般化をしてるみたいだが、それが後々無矛盾性証明とか強制法に効くっぽい。まだ知らんが。
0327132人目の素数さん
2021/05/21(金) 19:03:15.82ID:i7zfJXgh
>>318,319,321
https://i.imgur.com/Q4qd3OO.png
↑行間埋めた結果がこれ

行間埋めるのに昨日からダラダラと考えたから8時間以上掛かったかもしれん。
すんげぇ時間の無駄。
時間を掛ければ、まぁ、ある程度、当該分野を学んだ人間なら誰でも出来ると思う。
でも、そのためにたかが1ページの証明の行間を埋めるのに数時間も時間を掛けさせられるこの無駄さ。
マジでいらねぇだろ、こうやって著者の怠惰さが読者にシワ寄せさせられるって事に気づけ、カス
0329132人目の素数さん
2021/05/21(金) 20:04:16.83ID:i7zfJXgh
>>328
俺がこうやって行間を埋めたことによって、また後で学ぶ誰かが無駄な行間埋めに数時間の無駄な時間を割く必要がなくなる
山登りの道を整備していくってこんな感じ
0330132人目の素数さん
2021/05/21(金) 20:10:05.58ID:i7zfJXgh
今回俺が例示したのは1つの本の1つの定理
たったその1つの定理の行間埋めに、俺の能力では8時間ぐらい掛かってしまった
1冊となると、行間埋めに8×数十=数百時間の無駄な足止めを食らうって計算。
先人が山登りの道を整備すれば、後人はその数百時間を省略できる。後人はその数百時間で更にどんどん先にすすめる
でも、だ〜〜〜〜れもそんなことしないんだよな
こうして、「数学は苦労して学ぶものだ」という非合理的な精神論がまかり通ってしまうわけ
0331132人目の素数さん
2021/05/21(金) 20:47:00.50ID:Ebdbgl5J
自己評価高いっすね
0332132人目の素数さん
2021/05/21(金) 21:26:45.88ID:i7zfJXgh
>>331
いや、皆が行間埋めれば、後の人は皆その恩恵を受けれるって話なだけ
輪講だと内輪の数人しか恩恵ないだろ
0333132人目の素数さん
2021/05/21(金) 21:49:02.43ID:n4/Wm7XZ
イラン努力だよそれ
0335132人目の素数さん
2021/05/21(金) 22:07:02.93ID:i7zfJXgh
演習問題って、実際はただ単に著者が行間埋めるの面倒だけど、やること自体は難しくない奴だよな
そんなものを含めて徹底的に行間埋めたものを出して読者にもはや考える必要性無いぐらいにしたら、
論文が読めるようになるまでの時間が年単位で短くなるような気がするよな
0336132人目の素数さん
2021/05/21(金) 22:33:48.50ID:n4/Wm7XZ
>>334
誰も感謝しないから

その努力無くても十分理解できていくのが普通だから
0337132人目の素数さん
2021/05/21(金) 22:34:19.91ID:n4/Wm7XZ
>>335
たぶんならない
0338132人目の素数さん
2021/05/21(金) 22:38:05.94ID:n4/Wm7XZ
早く読めるようになるためには鵜呑みにすることだよ
それが一番
鵜呑みだと理解でないからダメだというのは
数学は理解だと思い込んでいるからだよ
数学は新しいものを作り出すのが目的で
理解はそれに必要なことが表層的でも分かっていれば良いだけ
なんていうか
すべてを定義から持ち上げるのではなくて
どんな性質があるのかをしっかり認識していけばいいよ
0339132人目の素数さん
2021/05/21(金) 22:40:36.09ID:n4/Wm7XZ
もちろん
飛ばしたところは必要になれば埋めることが出来ると
自分で確信が持ててなければ鵜呑みには出来ないよ
その確信は鵜呑みにするには必要なこと
そのためにはたまには掘り下げて理解を深めたら良いけど
四六時中あるいは最初からそれやる意味はほとんど無いよ
0340132人目の素数さん
2021/05/21(金) 23:55:51.66ID:i7zfJXgh
>>339
分かるけど、理解できていない状態ってある日突然訪れるのじゃなくて、
「ここまでは95%理解できた。よし次行こう」
「ここまでは93%ぐらい理解できた。よし次行こう」
「ここまでは90%ぐらい理解できた。よし次行こう」
「ここまでは85%ぐらい理解できた。しっかりやれば99%理解することだって出来るから問題なし。よし次行こう」

こんな感じで自分の中ではしっかり理解できて来たつもりなのに、
「あれ?今まで理解できてたはずなのに、なんかあやふやになってきた。ちょっと戻るか」
「あれ?ちょっとこれって何だったけ?分かってたんだが、今一度よく見てみるとよく分からんな。ちょっと戻るか」
ってな感じでどんどん戻らされる。結局はこれで大分戻ってからしっかり行間を埋め直させられて、やっと再出発できる感じ。

突き詰めれば性格的なところもあるんだろうが、しっかり行間を埋めていないのに、鵜呑みにしてると lim x^n = 0 (x<1)みたいな感じで、
塵が積もって分からなくなるのが自然の摂理。
飛ばしたところがあると、自分が今行っている推論が本当に正しいのか確信が持てなくなるっしょ
0341132人目の素数さん
2021/05/22(土) 01:13:02.33ID:1ezJj2t3
>>340
じゃまガンバってね
0342132人目の素数さん
2021/05/22(土) 06:45:01.31ID:oFLHDVPI
ふと気づいた。
自分の積み上げてきたのは全部ゴミ、間違いだった。
ノートはゴミの山。

↑と、ならんように気をつけてな。
0343132人目の素数さん
2021/05/22(土) 07:17:46.38ID:5OsUuaIc
おめでとうございます。
あなたは任意の命題を演繹出来る、
万有理論を手にされました!
0344132人目の素数さん
2021/05/22(土) 10:55:09.59ID:qvSrt5Ii
>>342
専門数学であっても一応、ブログに纏めて解説動画を作ったら数百再生はいける

むしろ、なんかのテキストを行間埋めまくってるノートあるなら欲しいぐらいかな
俺が知ってる内容なら要らんが
0348132人目の素数さん
2021/05/22(土) 15:33:48.79ID:1ezJj2t3
>>347
逆だって言ってるのが分からないかな
0350132人目の素数さん
2021/05/22(土) 17:05:21.11ID:Ch/bRZTB
行間くんは青チャートとか嫌いだったろ?
てか高校数学嫌いだったろ
行間しかないからな
0351132人目の素数さん
2021/05/22(土) 17:22:31.68ID:qvSrt5Ii
こんなスレにまでゴミが湧いてくんのかよ
ゴキブリってほんとどこにでも湧いてくるな
0352132人目の素数さん
2021/05/22(土) 17:37:25.05ID:d+s/4MJ7
>>342
よほどのウッカリか自分の妄想に溺れるやつしか
そうはならんだろ
自分で書いたのが解読できなくなる事はあるが
思いつきで始めて結構後でダメだから証明やり直し… なんてやらかす先生の時のノートが
いまだに意味不明
0353132人目の素数さん
2021/05/22(土) 18:01:04.83ID:qvSrt5Ii
俺勉強ノートは全部スキャンして捨てたわ

数学のブログ書いてるけど、その時にノートはチラチラ見たな
0355132人目の素数さん
2021/05/22(土) 20:58:27.54ID:1ezJj2t3
>>349
うーん
行間の読めないヤツw
ってことね
0357132人目の素数さん
2021/05/22(土) 22:15:07.19ID:qvSrt5Ii
アホ「僕ちゃんの言ってることに質問してくるやつは行間読めないやつだぁ〜〜」

↑いや、お前、ここは幼稚園じゃないんだからテレパシーを求めんなよw
0360132人目の素数さん
2021/05/23(日) 02:01:41.21ID:LiT6WEAj
>>356
下らないことしか言わないんだなぁw
0361132人目の素数さん
2021/05/23(日) 02:03:36.76ID:LiT6WEAj
>>356
『足りない」と言われたと思っていることを『行間が読めない』と言われたとは気が付かないぐらいに『行間どころか空気も読めない』わけね
0362132人目の素数さん
2021/05/23(日) 02:05:59.61ID:LiT6WEAj
君のあるべきと思う方向性を同じようによろしいと思う人はそうは居ないってことだよ
0363132人目の素数さん
2021/05/23(日) 04:15:17.98ID:I5mXcioG
行間埋める作業が数学だろ
みんなが自明だと思って読み飛ばしてるところで立ち止まって、それが自明でないとわかればそれが一本の論文になる
逆に言えば行間開きまくってんなーっていう箇所は著者があまり突っついてほしくないと思ってる箇所でもある
もしかして行間君に食って掛かかってる奴って天国から書き込んでる竹内じゃないの?
イェーイ、竹内先生見てるぅ?ww
0364132人目の素数さん
2021/05/23(日) 04:18:33.97ID:LiT6WEAj
>>363
>みんなが自明だと思って読み飛ばしてるところで立ち止まって、それが自明でないとわかればそれが一本の論文になる
それって
ほとんどあり得ないことに
ビッドしてるってことだよ
0366132人目の素数さん
2021/05/23(日) 04:58:20.20ID:LiT6WEAj
>>365
みんなががミソ
十分検証はされてることに引っかかってるってことだと思ったが?>みんなが自明だと思って読み飛ばしてるところ
0367132人目の素数さん
2021/05/23(日) 05:12:30.32ID:I5mXcioG
>>366
それが才能でしょ
普通なら気づかないところに気づいてしまう
そういう人が偉い先生になるんだわ
0368132人目の素数さん
2021/05/23(日) 05:19:19.65ID:LiT6WEAj
>>367
そりゃそんなレアなことに気がつけるんならねw
ほぼ無理
0369132人目の素数さん
2021/05/23(日) 05:20:28.20ID:LiT6WEAj
偉い先生はほぼほぼそんなことしてないよ
新しいこと開拓して行くのみ
0370132人目の素数さん
2021/05/23(日) 05:33:22.00ID:I5mXcioG
そうかね?
じゃあスマリヤンは?
当時完全にオワコン扱いされてたゲーデルの不完全性定理をエレガントな形で再構築した
ずっとプー太郎やってて学士取ったのが34歳だぞ...
暗記型の秀才しかいない日本でなら「何いってんだこの落ちこぼれのおっさん」って一蹴されて終わり
0371132人目の素数さん
2021/05/23(日) 05:39:13.28ID:I5mXcioG
行間くんの意見には同意
日本は行間読ませるクソ教授の本多いよ
なぜならクソ狭い分野に齧り付いてる同業者か、自分の講義取ってる学生向けにしか書いてないから
でも「行間は読め、分からないなら直接聞きに来い」って文化だからしょうがないじゃん
日本には啓蒙の文化とか一切ないから
0373132人目の素数さん
2021/05/23(日) 08:52:16.31ID:LiT6WEAj
>>370
だからそんなレアな例出されてもねー
0374132人目の素数さん
2021/05/23(日) 08:55:43.03ID:LiT6WEAj
まあいいや
君も頑張ってナー
0376132人目の素数さん
2021/05/23(日) 11:00:37.21ID:fq3P4R2A
例えば竹内の現代集合論入門だが、
順序集合Xに最大元1を加えた順序集合をX1とした時、
Xの正則開集合全体の成すブール代数B と
X1の正則開集合全体の成すブール代数B1 はブール同型になる
っていう事実を一言触れるだけで証明は読者に丸投げ。
実際の証明は確認作業をするだけの単調でダラダラした証明になるが、証明自体は20行近くかかる。

たとえ方針が簡単であっても20行近くの行間をすっ飛ばしするのはアカンやろ


スマリヤンはゲーデルの不完全性定理が大好きなのか知らんが、一般読者向けに噛み砕いた(と自分で思ってるだけの)本を書いてるけど、
くっそ読みにくい&中身が薄かったのを覚えてる
0377132人目の素数さん
2021/05/23(日) 11:17:32.02ID:f0/KUFEM
簡単だが長引く証明を省略するのはデフォ
自分でやるのは推奨だが他人に提供するのは余計なお世話
0378132人目の素数さん
2021/05/23(日) 15:06:29.47ID:fq3P4R2A
>>377
俺はそういうのには反対やな
省略していいのは、愚直にやるだけの3行程度で終わる短い証明やな


あと、一般的に、本の最初の30ページと最後の30ページでは説明/証明の濃さ・丁寧さが雲泥の差なんだよな
最初の方は「x∈UA_iなのでiが存在してx∈A_iである。従って、このiに対して…」とかバカ丁寧にやるくせに、
最後の方になったら「xに対して自然にA(x)を定めることが出来て、これが○○写像となるので、この写像を使えば定理の主張は明らか」とかの行間だらけの証明が当たり前にある
完全に舐めてる
最後方なんてどっかの論文を簡単にまとめてる程度のパターン多いし
0379132人目の素数さん
2021/05/23(日) 16:01:43.09ID:LiT6WEAj
>>377
ホンソレ
0380132人目の素数さん
2021/05/23(日) 18:30:07.95ID:fq3P4R2A
本を読んで、その本では○○に関する基礎的知識は既知としている時、
その基礎的知識に不安があったら、当該分野の入門書からやり直すべきかと逡巡してしまう。
>>338,339がいう鵜呑みっていうのはこういう時に上手く使っていくべきってことも分かってはいるけど、不安にかられる
0382132人目の素数さん
2021/05/23(日) 20:49:20.22ID:I5mXcioG
一冊で理論が完結してない本はカスだよ
ちゃんと細部まで証明しろ。それを読み飛ばすかは読者が決める。証明できないなら本出すなや
東大うんこ教授はもっと謙虚にならんかい
0383132人目の素数さん
2021/05/23(日) 20:56:32.12ID:I5mXcioG
>>376
スマリヤンは丸善のやつで読んだ?
アホみたいに丁寧だよ
赤本は問題解説ないのが問題だけど、定理の証明ちゃんとしてるしね
日本人の書いた不完全性定理の本はどれも注釈ありきで「※ここの証明は〇〇の論文を参照されたい」とかばっかでふざけてる
日本語で論理学始めるならスマリヤン以外に選択しないわ
0384132人目の素数さん
2021/05/23(日) 22:34:22.04ID:LiT6WEAj
批判だけしてないで自分で書いて出版したら?
まず無料公開で評判が出たら出版しようって所もあろうし
自費で出版するということも十分可能
0385132人目の素数さん
2021/05/23(日) 22:36:30.45ID:LiT6WEAj
いくら批判しても著者が改訂しなければ変わることはない
下らないことでないかどうかは出版という同じ土俵に立って
審判を受けたら良いよ
0386132人目の素数さん
2021/05/23(日) 22:40:59.61ID:LiT6WEAj
著作権には十分配慮せねばならないけど
文章を流用しなければ例や演習問題そのものには
著作権が存在しないのでまあ程度問題だけど使っても構わないよ
定理や書籍の構成も数学の一般的な常識の範囲内なら流用できる
見出しや文章そのものは著作権が適用されるから注意するべき
0387132人目の素数さん
2021/05/23(日) 22:46:22.98ID:LiT6WEAj
自費出版で自分が知っている例は
大学教員が自分のレジュメをそのまま冊子にしているのとか
学生が自分のノートを無料公開している例とかあったよ
後者は君の行為と似てるんじゃないかな
最近だとユーチューブとかもあるし
ブログでいろいろ公開してる人もいるし
敷居はずっと低くなってる
0388132人目の素数さん
2021/05/23(日) 23:47:12.95ID:Klz55ved
>>383
旧版のは酷い本だぞ
0390132人目の素数さん
2021/05/24(月) 02:12:35.61ID:LzDCpLfk
スマリヤンの決定不能の論理パズル ゲーデルの定理と様相理論
は読んだけど、ゴミだったな。
まぁ、これは学術書じゃなくて一般向け読み物だから、これだけを持ってスマリヤンを論評はできんが、この本はゴミだった。

ゲーデルの不完全性定理 Raymond M.Smullyan,訳:高橋昌一郎
も持ってるが、こっちは読んでないな
0392132人目の素数さん
2021/05/24(月) 02:43:46.88ID:LzDCpLfk
詳しい人教えて下さい

構成可能性集合の議論について。
山本新の「数学基礎論」では、空集合から出発して8個の基本的操作を考えています。
{x,y},E(x),x\y,x|y,…とか。
何でこんな意味不明な操作を基本的操作にしたのかについて、キチンとした説明が載ってる本・サイト教えて下さい。
0395132人目の素数さん
2021/05/24(月) 13:00:33.47ID:LzDCpLfk
>>393
田中尚夫の「公理的集合論」とか、他の本の場合、構成可能集合の定義は山本の定義と全く違うし、そもそも「構成可能」っていう直観に合致してる。
0397132人目の素数さん
2021/05/24(月) 15:17:58.93ID:fbDwB+tt
>>395
Gödelの元の論文かモノグラフかが
そういう書き方だったのでそうなってるだけで、
基本的な10個程度の関数はGödel関数とか呼ばれる。
取り方によって8個だったり15,16個だったりするけど
その特定の取り方に大した深い意味は無い。

動機というか理念としては、395みたいに
パラメータなしで定義可能であるような集合だけを
繰り返し超限回取っていくという事なんだけど、
そうして定義したクラス L が、
実際にZFCを満たすようなクラスになっていて、
しかもV= LがLで成り立つと言う事を証明したい場合、
L階層のもっと細かい性質を調べたいので、
L_αの集合からL_ {α+1}を作れるような少数の操作を
(証明の手段として)実際に選んで来て、
L_αの性質を具体的に調べると言う事をやっている。
0398132人目の素数さん
2021/05/24(月) 15:29:02.93ID:fbDwB+tt
高校数学で、初等幾何的な定理を示す為に
具体的に何か一つ恣意的に座標なりベクトルなりを
設定して後はただ式計算をするみたいな事をするけど、
やってる事は要はあれと同じ。
この座標にどういう概念的な意味があるんだろうかと
考えるみたいなもので、技術的な意味以外には
大した意味は無かったりする。
0399132人目の素数さん
2021/05/24(月) 15:38:28.83ID:fbDwB+tt
因みに岩波数学辞典 第3版のω^CKの定義が
こういう技術的な定義で、知らない人には
誰も意味が分からないような恣意的な式が書いてある。
昔、簡単に手に入って読めるような教科書には
こう言う定義しか書いてなかったから、
理解出来ずにトンデモみたいな事を言い出す
某T大の助教授の某先生とかが居たりした。
第4版とか新井先生の『数学基礎論』とかは
概念的な意味が分かりやすい定義になってるけどね。

こういう、同値な中でそのうちどれを定義に
採用するかみたいな話は意外と大事で、
天下り的に技術的な恣意的な定義をするなら、
最低限でも、この定義の意味は後に出てくる
Thm x.xx で明らかになる、くらいは書いてくれないと
初学者が読むのはかなり辛くなる。
ただ実際にはなかなか書いてくれない著者も多いので、
読んでいて今やってる操作の意味が分からなかったら、
前に戻るのではなくて、逆に
敢えて暫く先の定理まで流し読みするみたいな事が
必要になって来たりする。
0402132人目の素数さん
2021/05/24(月) 16:39:08.44ID:Mpdm5C5H
>>396
本でもウザいな
0403132人目の素数さん
2021/05/24(月) 17:22:41.50ID:LzDCpLfk
>>399
分かりました。
プログラムのソースコードだって、手続きの意味というよりも技術的な意味で一瞬分けのわからないようなことすることあるから、
まぁなんとなく分かった。
例えば、P(α)を順序数に関する述語とした時、μαP(α)をP(α)が成り立たないときは0になるように定義するには、
μαP(α) := ∪{α|P(α)∧∀β<α¬P(β)}と定義すればいいわけだが、和集合を取ったのはテクニック上そうすればいいわけであって、こんなのに意味を問うても仕方が無い的な感じだな
0404132人目の素数さん
2021/05/24(月) 17:24:33.00ID:LzDCpLfk
というか、構文論って大体「なんでそんな形の式を持ってきてんの?」っていうテクニック主体だな
0405132人目の素数さん
2021/05/24(月) 18:11:57.66ID:fbDwB+tt
技術的な意味以外に大した意味は無いと書いたけど、
数学って割と、神は細部に宿るみたいな感じで、
理念的な話よりも技術的な部分こそが
大事みたいなところもあって、
Jensenの微細構造の理論とかは、
そう言う技術的なテクニックを駆使すると
もっと詳細な事が分かるみたいな話だったりする。

限定算術とかの弱い算術とかもそうだし、
codingが絡むロジックの分野ってそんなんばっかり……
0406132人目の素数さん
2021/05/25(火) 13:18:27.79ID:5gBvJn+J
以下のどちらが正しいのでしょうか?

松坂和夫著『集合・位相入門』に以下の記述があります:

「これらの集合論の公理系に矛盾がないことを証明するのは、数学基礎論の問題で、今日まだ確定的に解決されてはいないが、
そのように展開された公理的集合論に少くとも経験上矛盾は生じていないのである。」

一方、斎藤毅著『集合と位相』には以下の記述があります:

「これを避けるために、公理的集合論が構築された。しかし、公理的集合論が矛盾を含まないことを証明することはできないことも
証明された。」
0407132人目の素数さん
2021/05/25(火) 13:22:31.87ID:5gBvJn+J
斎藤毅さんの言うように、矛盾を含まないことを証明できないことが証明されているなら、松坂和夫さんが「確定的に解決されてはいない」というのが嘘になります。
0409132人目の素数さん
2021/05/25(火) 14:21:48.53ID:5gBvJn+J
>>408
どう違うのでしょうか?
0411132人目の素数さん
2021/05/25(火) 18:36:58.14ID:PVeQzFMS
どちらの言ってる事もそれなりに正しいけど、
いずれにせよ矛盾した公理系からは何だって
(その公理系が無矛盾である事だって)
“証明”出来ちゃうんだから、
じゃあ数学の公理系の無矛盾性の証明って何だ、
何を前提にして良いんだという話になって来る。
いずれにせよ、普通の数学者が考えている意味での
証明とか、論理学で普通に定義される意味での
純粋に演繹的な証明とは同一視する事が出来なくて、
寧ろ検証とか実証とか説得とか書いた方が実態に近い。
ここら辺を真面目に詰めていくと、数学とか数理論理学の
問題というよりも、数学の哲学の問題になってしまう。
0412132人目の素数さん
2021/05/25(火) 18:39:13.58ID:PVeQzFMS
松坂和夫の記載は、「確定的」って
どういう意味なのかよく分からないのと、
まるで将来的には無矛盾性の証明が「確定的」に
為される前提かのような書き振りが、
竹内外史のドグマ的な主張を真に受け過ぎている。

齋藤毅の記載は「これを避けるために」と言うのが
数学史的にはたぶん嘘なのと
(「これ」が何か分からんのでエスパーだけど)、
あと後段は第二不完全性定理の事を言っているので、
同じ理屈でPeano算術とかそれよりかなり弱い、
帰納法が無くて具体的な数の足し算掛け算しかないような算術の無矛盾性も、
同様に証明出来ないことが証明されてるという話に
なっちゃうけど、本当に著者がそういうで
書いてるのかかなり怪しいと思う。

まあ不完全性定理にとかGentzenとかの無矛盾性証明とか
について勉強すると良いんじゃないかな。
松坂和夫も斎藤毅も、こういう事に関しては
恐らく耳学問で知ってるだけ。
0413132人目の素数さん
2021/05/25(火) 19:14:25.99ID:5gBvJn+J
>>411-412

ありがとうございました。

「これを避けるために」の「これ」はラッセルのパラドクスです。
0414132人目の素数さん
2021/05/25(火) 19:19:56.23ID:5gBvJn+J
正確に引用します:

「これをラッセルの背理(Russell's paradox)という。このように、集合の構成を無制限に行うと、理論が破綻する。これを避けるために、」
0415132人目の素数さん
2021/05/25(火) 19:50:25.07ID:5gBvJn+J
斎藤毅著『集合と位相』

(∀x x ∈ X) ⇒ x ∈ Y

は仮定 ∀x x ∈ X が成り立たないから、

(∀x x ∈ X) ⇒ x ∈ Y

は任意の集合 X と Y について成り立つと書いてあります。

x ∈ Y は命題ではないと思うのですが、この斎藤毅さんの記述は問題ないですか?
0417132人目の素数さん
2021/05/25(火) 20:26:42.17ID:5gBvJn+J
X と Y は与えられた集合ですが、

x ∈ Y と書いただけでは、 x が何なのかが分かりません。
0418132人目の素数さん
2021/05/25(火) 20:28:51.77ID:mzRyTcAR
このゴミ、レスが伸びてるサイトに構わず乗り込んでくるよな
0421132人目の素数さん
2021/05/26(水) 01:59:53.14ID:Vahdg5Vk
帰納的に無限なものを扱おうとするとその体系はω矛盾する
余帰納的に有限なものを扱おうとするとその体系は矛盾する

論理学はオワコンである
よって21世紀にもなって論理学()とかやってる奴はバカ低脳の衒学者である(Q.E.D.)


なにか反論あるぅ?w
0422132人目の素数さん
2021/05/26(水) 02:22:27.22ID:Vahdg5Vk
これだけは覚えておいて
お前らは純粋数学で食っていけない負け組であるどころか、ド文系アホ言語学者からもナンセンスな屁理屈しか捻り出さないうんこ製造機だと見做されてる

悔しかったらω矛盾性を生じない完全で合成性公理を含む体系を作ってみな

でない限り君らは統辞論にも意味論にも関与を許されないよ

論理学というオワった学問の中でそれっぽい数式を操ってオレカッケーやってるところ悪いけど、論理学は数学的に不完全であるどころか、自然言語の解析にも応用が利かないカス分野

お前らは言語学会では「空は青い」と「草は緑だ」を同じ意味としか捉えられない池沼だと見做されている

君らのお仲間の形式言語学者はずーっと学会で冷や飯食ってるどころか、数式なんて一つも分かんないド文系お絵描き認知言語学者に「チョムスキーは死んだ」( ー`дー´)キリッって見下されてるぞwww

それどころか形式的な言語学者までお前らのうんち論理学者を見捨てて機械学習とディープラーニングに奔走してるのが現状


21世紀の論理学者(うんこ製造機)の存在価値ってなんだろうか?...w
0423132人目の素数さん
2021/05/26(水) 02:32:56.47ID:Vahdg5Vk
論理学やってる奴ってさあ
本流の数学できるほど賢いわけでもないのに他分野にちょっかいだして
バカの一つ覚えの数式を振りかざして威圧してくるのなんなん?w
主流数学者、哲学者、言語学者みんなお前らのこと嫌ってるぞ
いつまでオワコン学問に齧りついてんだー
いい年した大人はみんな直観主義、経験主義よりのゆるい論理感覚を身に着けて学会で活躍してるぞ

厳密な俺カッケー(キリッ
↑もはやアカデミズムではこういうのはオナニーだって一番嫌われるって気づけな
0424132人目の素数さん
2021/05/26(水) 03:10:27.42ID:IBAxtGry
ID:Vahdg5Vk←何か詳しそうなやつなんだが、論破できるやつおる?
0425132人目の素数さん
2021/05/26(水) 07:12:20.56ID:tRLCzY5e
>>424
必要あることならともかくそんな面倒くさいことw
0426132人目の素数さん
2021/05/26(水) 08:15:55.68ID:/TLZ/mow
Y が集合であるとき、 x ∈ Y は命題でしょうか?
0427132人目の素数さん
2021/05/26(水) 08:36:02.11ID:/TLZ/mow
Y が集合であるとき、 1 ∈ Y は命題です。
0429132人目の素数さん
2021/05/26(水) 09:54:59.38ID:/TLZ/mow
A -> B

A は命題で偽
B は命題ではない

このとき、

A -> B は真である

と斎藤毅さんは言っています。

これはOKですか?
0430132人目の素数さん
2021/05/26(水) 09:57:34.45ID:/TLZ/mow
B は命題ではないわけですから、

A -> B も命題ではないのではないでしょうか?
0431132人目の素数さん
2021/05/26(水) 10:38:06.12ID:tRLCzY5e
>>400
>基礎論ってアマに大人気の分野なんですね
なんたって
基礎なんだから
とうぜんでしょw
0432132人目の素数さん
2021/05/26(水) 11:28:45.41ID:O0xy/L8Z
数学の出来ないやつが数学が分かるようになりたくて
基礎論に手を出すと更に混乱して分からなくなる罠
0433132人目の素数さん
2021/05/26(水) 12:46:47.97ID:xNC6eauM
数学の基礎ではあっても
数学学習の基礎では全然ないな
0436132人目の素数さん
2021/05/26(水) 14:04:55.12ID:xPVb2lW5
>>434
ん?反論できんの?
そうか。論理学は進展性のないオワコンで言語学にも応用効かないゴミ学問だって認めるんだな
0441132人目の素数さん
2021/05/26(水) 18:27:41.09ID:tRLCzY5e
あんまり触るとドンドン膨れるぞw
0444132人目の素数さん
2021/05/26(水) 21:05:53.72ID:xPVb2lW5
「論理学やってます」←こいつの意識高い系上滑り感
0445132人目の素数さん
2021/05/26(水) 22:01:51.43ID:IBAxtGry
>>444
>>421
帰納的に無限なものを扱おうとするとその体系はω矛盾する
余帰納的に有限なものを扱おうとするとその体系は矛盾する

の出典リンク教えて
0448132人目の素数さん
2021/05/27(木) 00:46:39.36ID:jGXs54qd
>>446
じゃあ数学じゃなくて言語学とか哲学の板に行って管巻いて来なよ
0449132人目の素数さん
2021/05/27(木) 01:50:20.77ID:7jlYOPIy
>>448
じゃあ論理学はオナニーだって認めて
あと他分野に適応できるとかいう翹望を捨ててくれるかな
0450132人目の素数さん
2021/05/27(木) 14:17:35.45ID:jGXs54qd
>>449
それをやりたいのは君だろ?って言ってるんだが
分からないかなあ
関係ない分野のことを評論はしたくないんだけど
0451132人目の素数さん
2021/05/27(木) 14:19:51.50ID:jGXs54qd
昔で言えばポストモダンは下らないと言わせたい
みたいな
0453132人目の素数さん
2021/05/27(木) 15:11:10.48ID:axwi/tGp
>>449
帰納的に無限なものを扱おうとするとその体系はω矛盾する
余帰納的に有限なものを扱おうとするとその体系は矛盾する
から論理学はオワコンだって言ってるけど、
これって、「ゲーデルの不完全性定理があるから、数学は終わってる」って言ってるのと同レベルやろ?

あと、論理学が他分野に応用できるかってのは情報科学とかでもう既に応用されてるやろ。
0454132人目の素数さん
2021/05/27(木) 15:14:26.76ID:axwi/tGp
ID:xPVb2lW5
↑取り敢えずこいつの論点出しておきたいんだが、論理学がオワコンだとする根拠は他にある?
0455132人目の素数さん
2021/05/27(木) 15:41:21.64ID:XwYqASPD
>>451
ポストモダンは下らないよ。
0459132人目の素数さん
2021/05/29(土) 04:03:41.84ID:GOWSGUF+
>>457
前段の、、「ゲーデルの不完全性定理があるから、数学は終わってる」って言ってるのと同レベルやろ に反論は?
0461132人目の素数さん
2021/05/29(土) 13:28:57.00ID:GOWSGUF+
っつーか、>>457こいつみたいなレスしてる時点でググらず反射的にレスしてんのバレバレなんだよww
アホw
0462132人目の素数さん
2021/05/29(土) 14:28:44.51ID:stdHX+GY
他人にやらせて尻馬に乗りたいだけジャね?
0464132人目の素数さん
2021/05/29(土) 17:55:51.75ID:4Vt9FJYC
well-definedを確かめる議論はメタ数学
0465132人目の素数さん
2021/05/29(土) 18:11:14.74ID:h6ezU4gJ
スマリヤンが34歳までプータローとか書かれててワロタ
いや確かなんかしてたと思うんだけどな
0467132人目の素数さん
2021/05/29(土) 19:51:18.63ID:GOWSGUF+
>>466
クソ簡単な詭弁でワロタwww
じゃあお前は論理学が情報科学にどう応用されたか自分でどこまで調べたか言ってみww
0468132人目の素数さん
2021/05/29(土) 19:59:40.62ID:GOWSGUF+
そいつがいいたいのは

He studied mathematics and music at several colleges (including Pacific University and Reed College) before receiving an undergraduate degree from the University of Chicago in 1955 and a Ph.D. in mathematics from Princeton University in 1959.

↑これやろ
0469132人目の素数さん
2021/05/29(土) 20:58:28.46ID:Xj8qiFse
スマリヤンが30まで奇術乞食やってたのって常識だろ
日本では猿回しは乞食みたいなもん
0474132人目の素数さん
2021/05/30(日) 01:45:00.99ID:N8pEMk1c
苦労人から業を成したスマリヤンは偉大だね
受験エリートしか教授になれない日本では考えられない
0475132人目の素数さん
2021/05/30(日) 04:54:25.37ID:SZjhQlmT
結構曲折を経てアカポスを得た人もいるよ。
0476132人目の素数さん
2021/05/30(日) 06:08:48.09ID:Wn/hwjBL
>>475
例えばだれ?
0477132人目の素数さん
2021/05/30(日) 07:12:17.55ID:kbB2TzDc
佐藤幹夫は結構苦労してるが、所詮は上級国民の東大出。
弁護士の父親が早くに病気で片輪になったので金に苦労。
0478132人目の素数さん
2021/05/30(日) 07:15:08.70ID:SZjhQlmT
>>477
品位を欠く言葉は使わない方が良い
0480132人目の素数さん
2021/05/30(日) 13:35:38.53ID:b9+y5HJ4
>>475
20代半ばで学部入学した人とか
アメリカへ飛んで、30過ぎて博士号とった人とか
くらいしか知らない

まあもっといるんだろうけどね
ちなみに上記二人は基礎論の人じゃない
0482132人目の素数さん
2021/06/12(土) 15:17:32.12ID:jwDv+K7b
主張自体はあまり間違ってないね
0483132人目の素数さん
2021/06/12(土) 15:35:56.22ID:jwDv+K7b
2001年にペンシルベニア大のPhDコースに入ったというデータを手がかりにMathematics Genealogy Projectで調べて日本人らしい名前を何個か当たってみた。
https://www.math.missouri.edu/people/takeda
本人画像がYouTubeの人と同じ
0485132人目の素数さん
2021/06/12(土) 17:50:38.30ID:FzAE+QfK
基礎論と呼ぶかどうか別にして
別にこの分や廃れてないように思うけどな
他の数学からドンドン離れて行ってるみたいな気がしなくもないが
そうでないような気もしなくもない
先々に行けばまた関連してくるかもだし
学会で数学史と一緒にされてるのは可哀想だが
まあある意味仕方ないかなとも思わなくはないし
0486132人目の素数さん
2021/06/13(日) 00:31:35.99ID:JuqjLmKr
>>481
どれかの動画で「応用はない。○○では使われてるらしいけど微妙。○○でも使われてるらしいけどなんか違う。」といった感じのことを言っていたけど、
応用はあることを知っていて、何かしらのコンプで受け入れられないんだろうなと思った
0487132人目の素数さん
2021/06/13(日) 00:53:08.65ID:Tqs3G847
モデル理論が代数幾何に使われたのは、確かにあんまり実りある応用ではなかったな。
作用素環の問題が連続体仮説を認めるか否かで答が違ってくるって話もあったよな。
あれも発展性のある話ではないみたい。
0489132人目の素数さん
2021/06/16(水) 07:38:35.99ID:EK0t4r2W
最近面白い結果ある?
0491132人目の素数さん
2021/06/16(水) 10:54:31.29ID:EK0t4r2W
基礎論プロパ−以外にも面白い結果という意味
0492132人目の素数さん
2021/06/16(水) 14:39:58.65ID:+i0kxdZP
巨大基数関連がどんどん荒唐無稽になってってるのが面白い
0493132人目の素数さん
2021/06/16(水) 20:08:20.84ID:a3IfM/w/
>>481
全然似通ってるように見えないんだけど・・・
数学基礎論は現代の数学の基礎付けというよりは
固有の数学的な技術がかなり発達した、数学の一分野になった、的な主張でしょ
何十年も前から言われてることやん
0494132人目の素数さん
2021/06/21(月) 16:35:50.60ID:qR29a8XD
φ は、ベクトル空間の公理のうち、1つを除いてすべて満たす。その1つはどの公理か?

という問題が線形代数の教科書に書いてあります。

(1)がその公理だとは思います。

(2)はvacuously trueということだと思います。
(2)の公理では、その記述に存在しない 0 が使われています。
(2)が真か偽か問う際に、そのことはどう考えればいいのでしょうか?


(1) ∃0 ∈ φ∀v ∈ φ, v + 0 = v
(2) ∀v ∈ φ ∃w ∈ φ, v + w = 0
-----------------------------------------------------------------
(2)
∀v ∈ φ ∃w ∈ φ, v + w = 0

は、

(2')
∃u ∈ φ∀v ∈ φ, v + u = v
この u を 0 と書くと、
∀v ∈ φ ∃w ∈ φ, v + w = 0
が成り立つ。

ということを言っていると考えると、「∃u ∈ φ∀v ∈ φ, v + u = v」は成り立たないので、(2')も成り立たないと考えられるのではないでしょうか?

つまり、

(2)は(1)が成りたつことを前提としているのではないでしょうか?
そして(1)は成り立たないため、(2)も成り立たないということになりませんか?
-----------------------------------------------------------------
それとも、(2)は

「∃0 ∈ φ∀v ∈ φ, v + 0 = v」 ⇒ 「∀v ∈ φ ∃w ∈ φ, v + w = 0」

が成り立つということを言っているのでしょうか?

だとすると「∃0 ∈ φ∀v ∈ φ, v + 0 = v」は成り立たないので、(2)は真ということになります。
0496132人目の素数さん
2021/06/21(月) 19:50:35.59ID:BTwHLzeG
>>494
φって何?
0498132人目の素数さん
2021/06/21(月) 22:12:02.51ID:7VqxVGih
部屋に誰もいないとき
「部屋にいるのは全員 女だ」は真だけど
「部屋に少なくとも1人 女がいる」は偽
という話かな?
0499132人目の素数さん
2021/06/21(月) 22:38:02.68ID:BTwHLzeG
>>494
>ベクトル空間の公理
その本での公理を全部正確に書き出してよ
>>497
サンクス
0501132人目の素数さん
2021/06/21(月) 22:59:32.39ID:qR29a8XD
>>499

普通のよくあるベクトル空間の公理です。(例えば、齋藤正彦さんの本と同じ公理です。)

commutativity
associativity
additive identity
additive inverse
multiplicative identity
distributive properties
0502132人目の素数さん
2021/06/21(月) 23:14:11.83ID:qR29a8XD
>>500

ありがとうございました。

なるほど、それが正しそうですね。

ということは、著者の言う通り、 φ は1つを除いてすべての公理を満たすわけですね。
0503132人目の素数さん
2021/06/22(火) 07:39:20.87ID:V0RrpQTA
>>501
可換体(あるいは斜体もOK?)Kに対し(V,+,・)がベクトル空間であるとは
+:V×V→V
・:K×V→V
であって
>commutativity
∀x,y∈V (x+y=y+x)
>associativity
∀x,y,z∈V ((x+y)+z=x+(y+z))
∀x∈V ∀a,b∈K ((ab)x=a(bx))
>additive identity
∃0∈V∀x∈V (x+0=x)
>additive inverse
∀x,y∈V∃z∈V (x+y+z=x)
>multiplicative identity
∀x∈V (1x=x)
>distributive properties
∀x,y∈V∀a∈K (a(x+y)=ax+ay)
∀x∈V∀a,b∈K ((a+b)x=ax+bx)
ですか?
additive inverseのところは
∀x∈V∃y∈V (x+y=0)
となってますか?
0504132人目の素数さん
2021/06/22(火) 07:54:22.15ID:V0RrpQTA
>>503
>additive inverseのところは
>∀x∈V∃y∈V (x+y=0)
こちらだった場合additive identityが成立していることが前提ですので正確に書けば
∃0∈V∀x∈V∃y∈V (x+0=x → x+y=0)
となるのでしょう
∃0∈V∀x∈V∃y∈V (x+0=x ∧ x+y=0)
かも?
0506132人目の素数さん
2021/06/22(火) 09:15:15.53ID:V0RrpQTA
>>505
P(x),Q(x)で
∃0P(0)を公理
その公理で存在が許される0についてQ(0)を公理という状況
数理論理学的にはQ(0)と書くのは
∃0(P(0)→Q(0))の省略?それとも∃0(P(0)∧Q(0))?
後者かなあ
でも後者なら
∃0P(0)を証明できるからこちらは公理から外すべき?(わかりやすさから残す?)
0507132人目の素数さん
2021/06/22(火) 09:30:51.88ID:cwJCQ+TT
定義による定数記号の導入法、その扱いの仕方が分かっていない様だな。
0508132人目の素数さん
2021/06/22(火) 09:35:44.37ID:cwJCQ+TT
定義による定数記号cの入った論理式、それと同値な定数記号cを除いた論理式への変換手順を考える。
0509132人目の素数さん
2021/06/22(火) 09:38:33.50ID:V0RrpQTA
>>507
分かってないです
変換手順は?
0510132人目の素数さん
2021/06/22(火) 09:45:25.35ID:cwJCQ+TT
この程度は自力で導出できるまでに、知識と理解を積み上げ無いとこの先の独学は不可能と思う。
0511132人目の素数さん
2021/06/22(火) 10:58:52.05ID:V0RrpQTA
変換手順の定義ぐらい教えて欲しいけど?
∃0P(0)を公理
Q(0)を公理としたい場合は
∃0(P(0)→Q(0))
で良いんじゃないの?
0512132人目の素数さん
2021/06/22(火) 11:41:09.23ID:hz0YQ5Zq
詳しい人が居るから、そのうち気が向いたら通りすがりで教えてくれるよw
0513132人目の素数さん
2021/06/22(火) 11:41:46.18ID:V0RrpQTA
>>500
こちらだとすると
これに限らず
∃0P(0)
∀0(P(0)→Q(0))
を2つの公理とするのが正解?という気もするな
正解?>>510
0514132人目の素数さん
2021/06/22(火) 11:46:54.88ID:V0RrpQTA
>>512
まあしかたないねえ
kindな人材求む
0515132人目の素数さん
2021/06/22(火) 12:08:07.85ID:jT69Wgzy
前原昭二さんの記号論理の本ですが、こういう本は厳密でクリアに書かれていないといけないと思うのですが、
前原さんの本はそうじゃないと思います。

何かいい入門書はありませんか?
0516132人目の素数さん
2021/06/22(火) 14:33:36.51ID:jaeA7klo
厳密でもクリアでもないのは松坂くんの頭の方だと思うの
0518◆QZaw55cn4c
2021/06/23(水) 08:43:54.66ID:zPO9gkpt
>>515
前原さん著の「記号論理の入門書」は二冊ありますよ、どっちのことをいっているのですか?
0520132人目の素数さん
2021/06/23(水) 10:31:00.17ID:Ht+mmCAi
>>518

記号論理入門 新装版 (日評数学選書) 単行本 ? 2005/12/10
前原 昭二 (著)

です。
0522◆QZaw55cn4c
2021/06/25(金) 01:12:50.27ID:ForJpWq9
>>520
まず 2005 年というのは嘘でしょう、私の手元にあるのは1967年初版の分ですが?
それに、それは初めて記号論理をやる人のために意識して冗長に書いた(特に序章は漫談風でしょ?)やつだから、それで前原氏を評価するのは早計かと
もう一つの方を読みなさい‥‥
0524◆QZaw55cn4c
2021/06/25(金) 01:17:20.39ID:ForJpWq9
>>521
高卒PGのわたくしめが推薦するのなら戸田山和久氏のがいいとおもいますね、分厚いのが凄くいいですが、なんでこっちをやらないのでしょうか?
0525◆QZaw55cn4c
2021/06/25(金) 01:18:24.72ID:ForJpWq9
>>523
新装版って、初版年をすげ替えるのが慣例なんですか?どこかに初版 1967 年とか書いとかないといけないと思うのですけど‥‥
0526132人目の素数さん
2021/06/25(金) 05:36:59.88ID:mrejkr1j
>>525
出版関係詳しくないから分からないけど
Amazonの新装版の商品ページでは2005年表記
0527132人目の素数さん
2021/06/25(金) 16:22:27.03ID:XflGA9nQ
>>515のやるべき事は、重箱の隅に拘らずに
まず何が大事で何が些事かを
自分なりに分かろうとする事だと思う。

一般論としては、複数の著者が強調している事は
一般的に重要だとみなされている事が多いと思う。

それだけではないけどね。数学の世界には
folkloreと言って、どの本にも書いてないけど
専門家には常識みたいな事項も多いから。
0528132人目の素数さん
2021/06/25(金) 16:27:43.70ID:QqR/ZCDF
重箱の隅で自分を大きく見せようなんて腐った奴に言うだけ無駄
0530◆QZaw55cn4c
2021/06/26(土) 15:39:52.47ID:Q9p0+UPN
>>527
>数学の世界にはfolkloreと言って、どの本にも書いてないけど

「山辺の方法」ですか?
そもそも山辺って誰?
0531132人目の素数さん
2021/06/26(土) 17:05:51.01ID:FOYkOaq1
「山辺の方法」って初耳だったけど
なるほどなー多項式は有限回微分で消えるからか
0532◆QZaw55cn4c
2021/06/26(土) 17:57:02.80ID:Q9p0+UPN
>>531
それどころではないんですよ
無限項の多項式でも成立するんです、例えば
dy/dx + y = e^{ax}

x = e^{ay} / (p + 1), p = dy/dx
と置いて山辺の方法が使えます、お試しあれ
0533132人目の素数さん
2021/06/26(土) 18:51:19.06ID:BlXNtqL5
>>530
そんな名前で呼ばれているとは知らなかったけど
それはたいていの微分方程式の教科書に載っているから
ここでいうfolkloreではないよ
0534132人目の素数さん
2021/06/26(土) 19:00:55.42ID:BlXNtqL5
>>532
y'+y=e^xに使うと
y=(1-p+p^2-p^3+…)e^x=e^x-e^x+e^x-e^x+…
ですか
何とか和で考える?
0535◆QZaw55cn4c
2021/06/26(土) 20:19:35.48ID:Q9p0+UPN
>>534
a = 1 と置いてしまうと残念ながらうまくいかないです‥‥ a を残した状態でやってみてください
0537132人目の素数さん
2021/06/26(土) 21:43:12.64ID:rL58TNW/
科学史の本とか物理学者の伝記とか読んでると
場の量子論関係の計算を半年かけてやったとか三日かけてやったとか
学会から大学に戻るまでの汽車の中で計算したとか
そういう話が出てくるけど
いったいなんの計算をしてるんだ?
数値積分?

場の量子論の勉強、その2 : 大栗博司のブログ
https://planck.exblog.jp/15464130/
最初の論文がすでにノーベル賞クラスだった南部陽一郎
http://jimnishimura.jp/tech_soc/chem_todaynambu/chem_today0903/09_03.html
0542132人目の素数さん
2021/07/12(月) 00:54:56.21ID:4hFd3MT0
そういう意図で付けた名前じゃ無いと思うが・・・・・
0544132人目の素数さん
2021/07/12(月) 14:50:40.67ID:4hFd3MT0
>>543
よく理解して解説してあると思うよ
その取り組み姿勢が素晴らしい
0546132人目の素数さん
2021/07/15(木) 04:17:00.93ID:/Q7LnOe8
質問
田中尚夫の公理的集合論p140のモスウトウスキー同型定理の証明
https://i.imgur.com/8NmMrnH.png

クラス関数Gの定義が不十分だと思う。

クラス関数Pr2を、x=<p,q>ならばPr2(x)=q, そうでないなら Pr2(x)=xと定義することによって、
<u,v>∈G :≡ v=Pr2(u)``Dom(Pr2(u))
と定義するのが正解なのでは?

なぜなら、後に、F`x=G`<x,F|Seg_R(x)>としてFを定める、つまり、Gの引数に2項組が入力されるて、その第2項目が使われるのだから。
0547132人目の素数さん
2021/07/17(土) 20:22:52.84ID:fZREC01i
一階述語論理の教科書の意味論の章で構造、モデルって言葉が出てくるし、
公理的集合論の絶対性の章でも構造、モデルって言葉が出てくる。

これらの相互関係ってなんかありますか、あるいは、それが述べられてる本なり論文なりありますか?
0549132人目の素数さん
2021/07/17(土) 22:55:56.62ID:fZREC01i
>>548
該当章なりページなりキーワードなり教えてもらえると有り難いです
0551132人目の素数さん
2021/07/24(土) 14:11:06.36ID:t5qdrllK
Quora「不完全性定理とはなんですか?」

不完全性定理はヒルベルトの計画を破ったと言われる定理です。

ヒルベルトの時代においては述語論理は第一階述語論理、第二階述語論理と論理関数の階数建てに応じて別れていたのですが、ヒルベルトの計画としては述語論理は無矛盾であれば完全であることが示すことができると予想されていました。つまり第一階述語論理の完全性証明から初めて、第二階、第三階、‥と階数を上げて証明していき、数学的帰納法を使って第n階述語論理も完全という命題を証明して述語論理は完全という証明をしていこうという方針だったようです。

実際1929年にはゲーデルが第一階述語論理の完全性証明を提出して全部の階数の述語論理の完全性証明に第一歩が踏み出されたかと思われました。

ところが1931年に同じくゲーデルが形式体系が自然数論を含んでいると決定不可能性命題を構成できてしまうという不完全性定理を証明してしまいました。実は次に完全性を示すべき第二階述語論理は、第一階述語論理と違って、その自然数論を含んでいる形式体系だったのです。

つまり控えめなゲーデルはヒルベルトに気を使ってまわりくどく、

第二階述語論理は決定不可能性命題が作れる形式体系ですよ=第二階述語論理の完全性証明は不可能ですよ=第二階以上も当然無理=ヒルベルトの計画は破綻しましたよ

ということを不完全性定理を通して間接的に主張したのです。

ヒルベルトもそれを認めてアッケルマンとの共書の「記号論理学の基礎」の後の版で「ゲーデルは第二階述語論理が不完全であることを示した」と書いてヒルベルトの計画は打ち捨てられることになりました。結果、若い数学者のゲーデルが大数学者のヒルベルトを打ち倒すというセンセーショナルな結果をもたらした定理となりました。

つまり質問に回答すると「第二階以上の述語論理は、第一階述語論理と違って、不完全な形式体系であることを控えめに回りくどく主張した定理」です。


こんな話聞いたことない。
0552132人目の素数さん
2021/07/25(日) 02:37:29.76ID:XFSJMVT+
出典のリンク貼ってくれ
0554132人目の素数さん
2021/07/25(日) 02:55:29.50ID:XFSJMVT+
最初から貼っとけ
0555132人目の素数さん
2021/07/25(日) 02:57:01.20ID:myN7dHaI
基礎論とか数理論理、集合論でおすすめの洋書を教えてください
0556132人目の素数さん
2021/07/25(日) 02:59:37.60ID:XFSJMVT+
解答した人のプロフィールから過去の解答をみたら出鱈目ばっかり書いている
無視が正解
0557132人目の素数さん
2021/07/25(日) 03:09:05.66ID:27sSD0Eb
http://alg-d.com/
↑この人は若手の研究者っぽいが入門者でも触れられる話題が非常に詳しい

参考文献でよく見る書籍
Axiom of Choice(Horst Herrlich)
Set Theory -An Introduction to Independence Proofs-(Kenneth Kunen)
Set Theory(Thomas Jech)
The Axiom of Choice(Thomas J.Jech)

General Topology(John L.Kelley)
Topology(James R.Munkres)

[Bibliotheca Mathematica 01] Introduction to Metamathematics(Stephen Cole Kleene)
Introduction to Metamathematics(Stephen Cole Kleene)
Mathematical Logic(H.D.Ebbinghaus,J.Flum,W.Thomas)
Mathematical Logic(Joseph R.Shoenfield)

Theory of Recursive Functions and Effective Computability(Hartley Rogers Jr.)
Recursion Theory(Joseph R.Shoenfield)
0558553
2021/07/25(日) 04:05:12.94ID:8D2BnmTY
>>554
最初からったって俺は>>551じゃないしな
ググればすぐ見つかるんだから貼る「必要」はないだろうし
0559132人目の素数さん
2021/07/25(日) 09:27:48.50ID:X9sEL5Xn
質問する側が出典を明示しないのは駄目だと思う。
リンク先を見たら筆者がおかしな人だってすぐに分かるのに。
0560132人目の素数さん
2021/07/25(日) 09:46:34.61ID:pMx3Ql/8
>>559
別におかしな人には思えないけどなあ
0561132人目の素数さん
2021/07/25(日) 10:11:38.42ID:X9sEL5Xn
https://jp.quora.com/profile/IWAKI-Hidekazu
工学や電磁気の世界ではなぜ複素数を使っているのですか?そのメリットは何ですか?

の解答が、この人変?、と思えた。
0562132人目の素数さん
2021/07/25(日) 10:23:05.09ID:VgVa1qgU
林晋著『ゲーデル不完全性定理』は、長年の謎がつながるようになる著作。ようやく繋がったありがとうって感じになるね。
0563132人目の素数さん
2021/07/25(日) 10:39:24.90ID:X9sEL5Xn
新刊?
林によるゲーデルの原論文の翻訳のこと?
0564132人目の素数さん
2021/07/25(日) 10:46:41.04ID:NH5BCXVt
そうあの読み方が特殊な翻訳本
0565132人目の素数さん
2021/07/25(日) 10:55:45.90ID:NH5BCXVt
「ウソつきのパラドックス」ってそういう意味かよって納得できるようになる。
0566132人目の素数さん
2021/07/25(日) 12:21:32.55ID:W/Husazs
ある試験を受けた学生達は全員不合格だった
の否定は
全ての試験を受けた学生達の中には合格した者もいた
ですか?
0567132人目の素数さん
2021/07/25(日) 12:25:34.37ID:X9sEL5Xn
この場合、「ある試験」を固定して考えるのでは
0568132人目の素数さん
2021/07/25(日) 14:22:07.87ID:pRMst001
>>559
質問はしてないし、1行目の
"Quora「不完全性定理とはなんですか?」"でググればすぐ見つかる
筆者がどんな人か知りたければそれで足りるでしょ
0569132人目の素数さん
2021/07/25(日) 14:43:23.49ID:myN7dHaI
>>566
論理式で書いたらわかりやすいと思うよ
0570132人目の素数さん
2021/07/25(日) 14:56:38.28ID:X9sEL5Xn
>>568
黙ればいいと思うよ
0571132人目の素数さん
2021/07/25(日) 23:35:06.60ID:27sSD0Eb
>>566
exi t in Test, all s in Student [ Take(s,t) -> Fail(s,t) ]
を否定すると
all t in Test, exi s in Student [ Take(s,t) and not Fail(s,t) ]
=どのテストにも合格者は居た
0572132人目の素数さん
2021/07/25(日) 23:40:57.10ID:27sSD0Eb
ある試験を受けた学生達は全員不合格だった
よくよく考えたら2つの解釈がある

1 常識的に解釈したら、「とある試験では、受験者全員が不合格だった」
この解釈の否定が>>571

2 偏屈的に次のようにも解釈できる:「学生が、ある試験を受けてしまうと、全部の試験が不合格になってしまう」
 この解釈は社会通念に照らすとありえないが、文言解釈のみにとどまればこのように理解することも一応可能
この否定は演習問題として考えてみw
0573132人目の素数さん
2021/07/25(日) 23:49:45.60ID:27sSD0Eb
全ての試験を受けた学生達の中には合格した者もいた
は、
exi s in Student [ all t in Test Take(s,t) and exi t in Test not Fail(s,t) ]
となる。これを否定すると二重否定により原文に戻るのでやってみると、
all s in Student [ ( all t in Test Take(s,t) ) -> ( all t in Test Fail(s,t) ) ]
≡学生が全ての試験を受験していたならば、その学生は全てのテストに落ちていた。
≡全科目受験しておきながら1科目でも合格してるような学生は居なかった

まぁおかしな日本語になったが、結局は記号表現のほうが正しくかつ機械的に計算できるから、自然言語で考えるのは辞めたほうがいい
0574132人目の素数さん
2021/07/26(月) 00:29:35.29ID:ZY+g7c/X
>>572
「学生が、ある試験を受けてしまうと、全部の試験が不合格になってしまう」
学生が任意なのか特定なのか曖昧
偏屈じゃなくて知能が足りない
0575132人目の素数さん
2021/07/26(月) 00:50:24.34ID:JnL+zc4n
>>574
粗探しは、粗探ししてる奴の粗探しの仕方に知能のレベルが現れる
多分論理学を学び始めた学生なんだろうけど、今後もそうやって教科書の文章も目を光らせて粗探しのレベルを鍛えていったらいいと思うよ
0576132人目の素数さん
2021/07/26(月) 01:00:36.37ID:ZY+g7c/X
馬鹿が恥の上塗りをしている
0577132人目の素数さん
2021/07/26(月) 01:03:31.10ID:JnL+zc4n
>>576
要するにお前の読解力の問題
>>574で任意か特定かでどういう齟齬、違いが出るのか言ってみ
0578132人目の素数さん
2021/07/26(月) 01:07:52.51ID:XTJ0aNsT
ゲーデルの不完全性定理から数学的資源は無尽蔵であることが保証されている。
「ウソつき」のパラドックスの本質とはそういうことだ
0579132人目の素数さん
2021/07/26(月) 01:14:08.58ID:XTJ0aNsT
世の中にはハイゼンベルグの不確定性原理と不完全性定理を混同するという、もう一回科学史年表を書いて整理してみろと言いたくなるような人が実在するんだぜ。
全く動機が異なる、もっと言えばそもそも分野が違う。
ツイッターや5chでドヤ顔演じている前にそういう基本的な事をだななぞなぞするしかない
0580132人目の素数さん
2021/07/26(月) 01:28:00.76ID:JnL+zc4n
>>576
な?低次元な粗探ししか出来ないからお前は頭悪いんだよ
実際>>577に答えることすらできないのがお前の頭のだろ
くっそアホやわお前

低次元な粗探ししか出来ないくせに、ちょっと聞かれたらなぁ〜んも答えられない
これを世間一般的にアホという
0581132人目の素数さん
2021/07/26(月) 01:35:34.79ID:XTJ0aNsT
だったら、いちいち含みのある話を要所要所に突っ込んでくんなよ!
何も知らんいたいけな素人相手に唐突にSNS上でこんなところあるよ?どう?とかやって落ちたらそら気になるわ!
0582132人目の素数さん
2021/07/26(月) 01:52:14.07ID:XTJ0aNsT
あれは要するにやんわりとT芝に考え直させたかったんだろ。
そんなもん本気じゃないよって。わかるかよそんなもん
0583132人目の素数さん
2021/07/26(月) 08:13:13.19ID:tWdrJyRm
なぞなぞは続ける。文句行っても仕方なかった。スマヌ。
0584132人目の素数さん
2021/07/26(月) 10:17:45.84ID:+i1tNIIC
>>551
意味論的に一階述語論理は完全で
二階以上の高階述語論理は不完全で、
その理由は二階論理では単体でペアノ算術が
解釈できる事なので、一応内容的には正しい。
その旨の言及もあるので解答者がちゃんと理解している
可能性も結構ある。

ただこれ読んだ初学者は結構な確率で誤解しそうなので
その辺はアレではある。
0585132人目の素数さん
2021/07/26(月) 15:05:20.25ID:Bq8hSf20
>>584
551は数学的内容ではなくて
控えめにとかその辺のことを言ってるんだと思った
0586132人目の素数さん
2021/07/26(月) 18:43:17.34ID:+i1tNIIC
ああ、成る程。それは確かに。

真面目に読むとちょこちょこおかしいな。
まあ、物理の人っぽいから仕方ないけど。

ヒルベルトの時代には述語論理が
一階と高階に分かれていたとか(今は違うんかい)
ヒルベルトの計画は述語論理は無矛盾であれば
完全である事を示す事だとか
(ヒルベルト・プログラムは、信頼のおける
実在的な数学(≒算術)のみを用いて、
より広い概念的な数学の信頼性を確立する計画)、
n階述語論理の完全性をnに関する帰納法で
示そうとした、とかはこの人が捏ねたストーリーだね。

ヒルベルトに気を使って、は、
当時ゲーデルが無名の若手で
ヒルベルトは数学界の最高の権威で、
自分と異なる主義の学者を
政治的に排除とかもしてたから、
まあ常識的に考えてそれなりに配慮はするんじゃ
ないかなとは思う。
0587132人目の素数さん
2021/07/26(月) 19:37:37.60ID:Wo6j3jc1
ヒルベルトが果たして本当に大御所だったかなんてわかんなくない?
俺が思うにどうもヒルベルトは当時の時流からワンテンポ遅い。
大御所は大御所だろうけどすごい政治力があったかと言われるとそうは言えないんじゃないかと。
ブラウワーを排除したとも言われるけどあれもブラウワーと示し合わせただけで、黒幕は別にいるだろ。
ホワイトヘッドとか。

今もその系譜が一番政治力があるんだと思うんだが
0588132人目の素数さん
2021/07/26(月) 21:39:38.76ID:ZY+g7c/X
>>587
ヒルベルトは1961年生まれ
ゲーデルの不完全性定理発表が1931年でヒルベルトが70歳のとき
0589132人目の素数さん
2021/07/26(月) 22:38:45.80ID:XTJ0aNsT
ヒルベルト計画ってPM引きずってるよね。
不完全性定理の体系もPMの建て増しだし。

計画の基本コンセプトは広義の述語論理の完全性証明じゃない?
それが無理そうだっただからアッカーマンのイプシロン公理を突っ込んで、
広義の述語論理を狭い意味で完全性証明しようとしたってことじゃない?

ゲーデルはイプシロン関数を入れ子にしたらどうなるだろうって考えて不完全性定理に到達したと思う。
0590132人目の素数さん
2021/07/27(火) 00:11:51.14ID:urOCv5O3
日本の大学で数学基礎論や数理論理学か盛んなところってありますか?
0592132人目の素数さん
2021/07/27(火) 16:17:12.34ID:w01S8XFD
何故、数理論理学や数学基礎論の研究者が所属する院は情報科学みたいな場合が多いのでしょうか?
数学科では無いのでしょうか?
0593132人目の素数さん
2021/07/27(火) 16:28:59.05ID:joo85DAH
ゲーデルに微積や線形代数の授業をさせられる?
0594132人目の素数さん
2021/07/27(火) 17:20:42.81ID:w01S8XFD
>>593
そういう意味もあるんですね
0595132人目の素数さん
2021/07/27(火) 17:23:31.36ID:w01S8XFD
逆に数学科(学部)で数理論理学や数学基礎論の授業は無いのでしょうか?
0597132人目の素数さん
2021/07/27(火) 17:48:57.39ID:joo85DAH
順序数は数学科では教えない
0598132人目の素数さん
2021/07/27(火) 18:53:33.70ID:urOCv5O3
数学基礎論や数理論理学のようなものは大学の学部の授業ではまず無いのでしょうか?
逆に情報学科などはあるのでしょうか?
そもそも学部では習うことがないなら、残りの自分の数学科の時間を割り切って考えることができますが。
0599132人目の素数さん
2021/07/27(火) 19:04:39.20ID:VhYLsD5g
数学科にあってもいいんだろうが
日本では無い、少ないだけかと
上司になる人がそもそもいないから集まらないのかと
0600132人目の素数さん
2021/07/27(火) 19:13:56.12ID:merJkdf+
数理論理学者たちが他の数学者に仲間外れにされてるってことかね
可哀想
0601132人目の素数さん
2021/07/27(火) 19:21:50.19ID:urOCv5O3
>>600
もし数学者が100人の村になったら
数理論理学、数学基礎論の村はどのぐらいの人数になりますか?
解析村が一番多いですかね?
0602132人目の素数さん
2021/07/27(火) 20:08:17.93ID:merJkdf+
>>601
どうかね
数学者じゃないからどの分野がどれくらい人気があるとかは詳しくわからない(推測なら、大きく解析/代数/他、に分けるならたしかに解析の方が代数より人気があるってような感じかな?わかんない)
大学の数学科教授対象のアンケートとかがあるとしたら、専門分野の割合とか載ってそうだね
0603132人目の素数さん
2021/07/27(火) 20:37:42.82ID:VhYLsD5g
東京大学大学院数理科学研究科理学部数学科・理学部数学科
新 井 敏 康 (ARAI Toshiyasu)
講   座 離散数理学大講座 教授
研究分野 数学基礎論
研究テーマ
証明論
研究概要
証明論は数学における証明を対象にしています。その中で主に順序数解析を研究しています。
これは公理系に対して順序数を結びつけてその公理系の内部に潜む構造を解き明かそうとする分野です。
https://www.ms.u-tokyo.ac.jp/teacher/tarai.html
0604ハノン ◆QZaw55cn4c
2021/07/27(火) 20:59:16.18ID:qgZgPEDg
>>601
興味ありますね
古典的分類ではありますが、解析村人、代数村人、幾何村人の割合はどれくらいでしょうか?
0605132人目の素数さん
2021/07/27(火) 21:32:37.09ID:SZhmZpyp
>>593
やろうと思えば、そこらの普通の
数学の先生程度には出来るでしょ。

一般相対論のアインシュタイン・マクスウェル方程式の
ゲーデル解を発見した人だよ?

Bourbaki以前の世代なので
そもそも線形代数はそんなに基礎教養じゃない
(van der Weardenの現代代数学などを参照)けど、
テンソルは使える人なんだから普通の線形代数の
授業はやろうと思えば出来るはず。微積も当然出来る。
0606132人目の素数さん
2021/07/27(火) 21:59:23.49ID:FyQl7GfP
>>605
アインシュタインとゲーデルがやたら仲が良かった理由としては何か近しいものがあったんでしょうか?
0607132人目の素数さん
2021/07/27(火) 22:00:56.57ID:SZhmZpyp
何でだろうね
二人ともかなり孤独な研究者だったから
その点は馬が合うかもね
0608132人目の素数さん
2021/07/27(火) 22:18:34.62ID:icyI2YXr
>>605
下―デルに
そこらの数学教師レベルでしかない授業をさせることに
どんな意味があるかという話
全然わかっていないな
0609132人目の素数さん
2021/07/27(火) 22:19:35.89ID:icyI2YXr
訂正
下―デルーー>ゲーデル
0610132人目の素数さん
2021/07/27(火) 22:25:29.40ID:FyQl7GfP
小泉純一郎と小沢一郎がシンパシー感じて仲いいとかそういう話だろう
0611132人目の素数さん
2021/07/27(火) 22:29:39.48ID:SZhmZpyp
そもそもの元のレスは、なぜロジックの研究者は
情報科学の所属が多いのか、という話だから
ゲーデルが雑用的な仕事を任せておくには勿体ない
偉人だった事とかは取り敢えず関係ないような。

ロジックの研究者はみんな数学科の教員より偉いから
雑用なんかさせるのは勿体ない、とか
言いたい訳じゃないでしょ
0612132人目の素数さん
2021/07/27(火) 22:41:18.47ID:icyI2YXr
ゲーデルなら専門が数理論理学でも
微積や線形代数は普通の数学者以上に
分かっているからその授業をやるのに差支えはない
という話が出たからそういう話にしただけで
本来の意味はむしろ
数理論理と微積や線形代数はそりが合わないという趣旨だった
旧帝大クラスの数学科で順序数を教えなくなっただけでなく
数学と物理が共存するようになった学科などでは
ラッセルのパラドックスも教えられなくなっている
0613132人目の素数さん
2021/07/27(火) 22:44:38.06ID:FyQl7GfP
>>611
横レスだが、
理論物理とかだと学内の雑用に煩わされるのが嫌なんで、
教授会でわざとめちゃくちゃなことを強弁して雑用割り当てられないようにするらしい。
雑用は今後絶対増えていくし、実験とか応用とかの人と縁切ってでも研究するって人は
増えてくるんじゃないかな。多分誰も止めない

僕はホモトピー型理論を研究するんだから雑用はしませんとかそういう風になるように誘導されていくんじゃないの?
雑用する人の評価はひたすら曖昧にしていくと思うね。
0614132人目の素数さん
2021/07/27(火) 22:48:06.84ID:icyI2YXr
純粋数学だけでまとまっていれば
お互い様ということで
仲良く雑用の分担ができるのだろうけれど
0615132人目の素数さん
2021/07/27(火) 23:14:59.20ID:SZhmZpyp
反りが合わないのはまあ割とそうなんだけど、
ただKontsevich-Zagier予想とかの周期の話みたいに
微積とロジックって繋がってたりするし
去年のSLACSでも、与えられた公理系を表示するのに
必要な公理の個数の下界をホモロジー代数的な手法を
使って評価するみたいな発表とかもあったし、
学科でも個人でも、外への意識的な閾は
あまり作らない方が良いと思うなあ。
0616132人目の素数さん
2021/07/27(火) 23:15:12.02ID:FyQl7GfP
真夏の炎天下に午前中グラウンドの草むしりとか入ってきたら流石に揉めるんじゃない。
それなら線形代数なり微積の補習受け持ちますってなるのでは?
0617132人目の素数さん
2021/07/27(火) 23:38:09.35ID:SZhmZpyp
まあでも
ロジックの人
→微積や線形代数の講義するのがそんなに楽しくない
他の数学分野の人
→ 微積や線形代数の講義するのが割と楽しい
って違いくらいはあるかもね

それとモデル検証とか自動定理証明とかと
どっちに興味があるかみたいな話かも
0618132人目の素数さん
2021/07/27(火) 23:51:21.25ID:FyQl7GfP
原発と戦闘機のケースの論文斜め読みしたけど結局人海戦術じゃない?
数学理論から正しいですってよく考えたら稟議通らないし。本気でやってる人もいるだろうけど、本気でやってるとは思えない。
0620132人目の素数さん
2021/07/28(水) 07:04:58.05ID:Px1pzlaM
集合と集合の同値類の違い
0621132人目の素数さん
2021/07/28(水) 08:36:51.89ID:I3NkWihi
>>620
アレフ_1 := μα[αは基数∧アレフ0<α]だが、ω_1の定義ってなんですか?
0622132人目の素数さん
2021/07/28(水) 08:52:13.85ID:I3NkWihi
定義ウィキに載ってましたね

Lim(ω_1) & アレフ0 < ω_1 & ∀α[|α|<=アレフ0 -> α<=ω_1]
0623132人目の素数さん
2021/07/28(水) 09:22:45.67ID:I3NkWihi
ちょっと考えてみた所、
ω_1 := ∪{ α<アレフ1 | |α|<=アレフ0 }
ですね
0624132人目の素数さん
2021/07/28(水) 11:04:31.33ID:edUG4PKe
>>600
学会でも極めて少数派だから仕方なかろ
100人居たら
66人は解析
33人は代数
33人は幾何
1人がその他
0625132人目の素数さん
2021/07/28(水) 13:01:56.59ID:XJ/ne3DD
因みに科研費に応募する際の分類だと
基礎論は解析の仲間だそうだから
あまりその三分類に意味は無い気もするけどね
0626132人目の素数さん
2021/07/28(水) 15:07:35.81ID:mC9FNU7G
>>624
それ足したら100人になります?
0628132人目の素数さん
2021/07/29(木) 01:00:13.58ID:shb14Rxi
数理論理学がやりたいなら数学科には行くなというのは本当ですか?
例えば数理論理学にのみ興味があって解析学などには全く興味がないとか
0629132人目の素数さん
2021/07/29(木) 01:04:23.15ID:7hKXYqER
数学科でもいいが先生がいるところ
先生がいるところなら何学科でもいいかと
0630132人目の素数さん
2021/07/29(木) 01:44:22.46ID:tk4hVKVY
>>629
そういった考え方も有るんですね
数学科のカリキュラムがロジックの人たちは合わないとかはないですか?
微積をやるにしても所詮εδからしかやらないから不満だとか
0631132人目の素数さん
2021/07/29(木) 01:59:51.05ID:ddYgVb0P
寧ろ合うカリキュラムの学科なんてあるの?

論理学は哲学とか計算科学とか数学とか
色々と関係がある領域だけど、数理論理学
mathematical logicと言った場合は、特に数学との
関連が強い部分の事を指すので、
そういう部分が好きなら数学科が良いと思うよ。
0632132人目の素数さん
2021/07/29(木) 02:05:46.95ID:7hKXYqER
グロタンディークやボエボドスキーも数理論理学みたいな感じだろうが
グロタンディークはトポスの推進を弟子たちにやってもらおうとしたら、隠居後無視されたみたいにいってたが
0633132人目の素数さん
2021/07/29(木) 02:13:50.13ID:MGLVb+pU
>>631
数学科から転科しようかと思ってたんですが、転科先も見つからないのでこういう質問をしたのですが、とりあえず数学科に学部の間はいて、院で数理論理学を研究しているところに行くという方がいいですかね?
その方がスタンダードかな?
0634132人目の素数さん
2021/07/29(木) 02:27:28.23ID:azGs4fWk
>>633
研究者レベルの内容は分からんが、学部・院の授業レベルで言うと、
純粋数学の授業内容と数理論理学の授業内容は全く別物というか、内容的にかぶるものがない。
もちろん数学的思考力という意味でいうと似たようなものはあるが。

数理論理学では線形代数もほぼ?出てこないし、むしろ情報科学の方が授業内容に被りがちょっと出てくる。オートマトンとかグラフ理論とか

というか、そもそも、数理論理学は理論上は(?)前提知識に数学的知識は必要とされてないから、頭さえ良ければ高1からでも始められる
0635132人目の素数さん
2021/07/29(木) 02:37:59.99ID:5Huf7L2z
>>634
なるほど
情報科学に転科するのも有りですかね
0636132人目の素数さん
2021/07/29(木) 02:38:13.12ID:ddYgVb0P
そもそもポスドクとかの人にとって
研究職のポストって選り好み出来るほどには無いので、
取り敢えず採用してくれる所に就職していった結果が
数理情報科だったり理工学部だったり
稀には哲学系だったりするんだと思う。

大学教員として採用されるには
その人のプレゼンの上手さや有名さとか、
性格や人当たりの良さ悪さとか、
雇用先が求めている研究分野
(前任者がいる場合、前任者の専攻と近くなる
場合が多いと思うけど)に偶然近いとか、
研究者間での人脈や相互評価とか、色々要素があって
単一の原因で決まる話じゃ無い。

例えば今は東大数学科教授の新井敏康先生だって
工学部の教員だった時期も、
総合科学部の教員だった時期もあって
それは時の流れでたまたまそうなっただけ。
0637132人目の素数さん
2021/07/29(木) 02:38:25.09ID:ddYgVb0P
>>592で書いているような、数理論理学の研究者が
情報系の教員である場合が多いのは、
数学の他の分野は歴史が長いので伝統的に
数学科の多くのポストを占有してきた歴史的経緯があるし
大戦後に誕生した情報科学は実学なので、数学よりも
割と様々な専門の人がそれぞれ評価される文化がある事、
応用志向が純粋数学より強いので、そもそもの
社会的需要が高く、ポストの総数がそもそも数学科より
多い可能性がある事とか、そこらへんが主な要因だと思う
0638132人目の素数さん
2021/07/29(木) 02:43:35.04ID:ddYgVb0P
>>633
そういう事だったら、自分がやりたい事が
特に何なのか考えて、それに近い事をやっている教員が
居る所に行くのがベストかなとは思う。

ラムダ計算やりたいなら情報系だろうし、
モデル理論なら数学科になるだろうし。
やりたい分野の専門家が日本に居ない結果、
海外に行く人も結構居るよね。
圏論をやる為にヨーロッパに行くとか、
内部モデル理論をやりたいから米国に行く、とか。
0639132人目の素数さん
2021/07/29(木) 02:48:51.64ID:5Huf7L2z
>>638
なるほど
沢山の丁寧なご指摘ありがとうございます。
自分はまだ、具体的に何がやりたいというほどのものはありませんし、知識もまだ全然ありません。興味があるものは沢山あります。
ただ、漠然と数理論理学や数学基礎論をやりたいなーと思っています。
数学に興味を持ったきっかけが、ラッセルの『数理哲学序説』だったからです。
数学科に入ってそれなりに数学科的なことに染まってきましたが、どうしても解析学や集合論に納得がいかなくなって転科を考えている次第です。
0640132人目の素数さん
2021/07/29(木) 02:52:49.24ID:ddYgVb0P
同じ大学の情報系のカリキュラムや教員が
自分にとってベストだと思えるなら転科したら良いけど、
そうでない可能性も高いから、院から行き先を
考えるというのも一つの手だと思う。

結構局所的な事情が強く効くので
>>633が名古屋大の人である場合と京大の人である場合と
九大の人である場合とで、転科が良いか悪いかは
全然変わりうる。

個人的に誰かにコンタクトをとって、所属は今のままで
単位とかは関係なくゼミとかに参加させて貰ってる
みたいな場合も多いんじゃないかな

何やりたいか決まってないなら、まずは
いろんな分野を勉強してみてやりたい事を見つけるのが
優先じゃないかな
0641132人目の素数さん
2021/07/29(木) 02:57:54.89ID:cI9l/Z1n
>>640
確かにそうですね。先達からの助言はリアリティがあります。
残念ながら、名古屋大でも京都大でも東北大でも神戸大でも九州大にも所属しておりません。

ご指摘の通り、所属自体は、数学科であとは自由に動く方がいいでしょうね。

数学科=数学ではないので、数学科がつまらないからといって数学がつまらないわけではないですもんね。

とりあえず、院に行くときにまた悩みます。
0642132人目の素数さん
2021/07/29(木) 03:09:43.05ID:7hKXYqER
自分の認識だと形式化、抽象化という面で、数理論理学は数学の王道といった感じだが
いまの数学科が易しすぎて、自分のレベル合ってないと言うなら
無駄な時間を過ごすことになるかと思うから的を絞って進めたほうがいいとおもうぞ
上でグロタンディークとボエボドスキーを上げたが
ボエボドスキーは最後のほうでは証明論してたみたいだが
0644132人目の素数さん
2021/07/29(木) 03:22:23.65ID:4+VK6Ht4
>>642
自分も数理論理学は王道だと思っています。ただ実際の数学界ではアウトロー的な扱いのような気がします。

自分を頭が良いとか、特別だとかは一切思いませんし、数学科が易しすぎるとも思いません。
ただ、見ている視点が違うとは思います。

微積の授業の時は、何故もっと厳密にやらないのか、せめて実数論や集合論、位相をやった後、線形代数で定式化しないのかとか、集合と位相の授業の時は、何故公理的集合論は扱わないのか、数理論理学の主要問題には触れないのか、と思っていたことがあります。

教授に聞いても「そんなことにこだわっていたら前に進まないし、数学者がみんな公理的集合論や数理論理学を理解していない」と言われました。

単純に意識の違いだと思います。
自分は「数学とは何か」という意識を常に持っています。

しかし、具体的に絞ることは今の私の知識だと多分できません。
集合論や数理論理学、圏論には興味があります。ただ他の分野もよく知らないし、これらの分野についても良くはわかっていません。

とにかく悩んでいます笑
0645132人目の素数さん
2021/07/29(木) 03:23:46.04ID:4+VK6Ht4
>>643
別に僕は天才ではありませんし笑、年齢制限とかもありそうですが
0646132人目の素数さん
2021/07/29(木) 03:27:04.59ID:7hKXYqER
トポス、望月新一で検索してでたやつ

“宇宙際”についてのFAQ
Q1. 宇宙を取り替える, って数学基礎論的・論理学的に非自明な操作をしているの?
A1. 望月さんの宇宙際 Teichmuller 理論の論文において数学基礎論的・論理学的に非自明な操作をしているとは山下は (少なくとも現時点では) 思っていません.
“宇宙替え” は大事ですが, 数学基礎論的・論理学的に非自明な操作により証明をしているのではなく, 証明の本質的な数学的道具はやはり遠アーベル幾何的再構成アルゴリズムたちにあります.

Q2. じゃあ, 宇宙を取り替えるってどういう意味?
A2. 宇宙際 Teichmuller 理論では, 環構造そのものを変形します. スキーム論とは環論だと思うと, by definition でスキーム論が通用しない局面がしばしば出てくるということです.
一方のスキーム論での操作や基点などを他方のスキーム論にもちこむことはできません.
一方での恣意的なラベル付けが他方では通用しない, それは “宇宙を取り替える” ということではないか, という意味で使っています.
厳密な意味での Grothendieck 宇宙を取り替えると考えてもいいですし,
数学基礎論的に厳密な観点からはあくまで 1 つの Grothendieck 宇宙の中で考えてその中に別々にスキーム論があって, それを取り替えることを “宇宙を取り替える” という言葉で表現していると考えてもいいです.
“その新しい幾何学” と書きましたが, 従来の幾何学では環構造と整合的な射 (環付きトポスの射) を考えるのが幾何学であるという視点に立つならば, それは幾何学という枠組みすら超えているかもしれません.

Q3. たくさん宇宙を取り替るとしても, もともとそれらをすべて含むような宇宙をとってきてその宇宙で議論をすれば, 宇宙を取り替える必要はないんじゃないの?
A3. 確かに論理的にはその意味では宇宙を取り替える必要はありません.
けれども, 任意のコンパクトな (可微分) 多様体は十分大きな Euclid 空間に埋め込めますが, (可微分) 多様体の定義をEuclid 空間の部分空間として定義するのは不自然です.
それと同じ意味において, 初めから大きい宇宙をとってくるのではなく, 各スキーム論が局所的にあり宇宙を取り替えて別のスキーム論に移ると考える方が自然です.
http://www.kurims.kyoto-u.ac.jp/~motizuki/Uchuusai%20ni%20tsuite%20no%20FAQ.pdf
0647132人目の素数さん
2021/07/29(木) 03:31:05.12ID:azGs4fWk
>>639
なるほど、数理論理学はまだやってないということか
だったら入門書として戸田山和久の「論理学を作る」はぜひおすすめしたい
高校生でも読める一方でゲーデルの完全性定理まで理解できる
学術書なのにアマゾンでもかなりの高評価&多数レビュー
0648132人目の素数さん
2021/07/29(木) 03:39:16.54ID:7hKXYqER
世間が天才を誤解してるとおもうが
たとえばグロタンディークやアインシュタインや岡潔なんかは天才ではないとおもうが
誰にでもできそうな抽象化、形式化へのこだわり、努力が強かったのが大きく努力家だろうと


グロタンディークの数学観
グロタンディークは、フランスで活躍したドイツ出身の数学者です。彼は代数幾何学という分野で革新的な業績をあげました。

彼は、次のように語っています。
「私は学び、熟し、変わります。私は変わっていますが、『同一のまま』 であるなにかもあります。私はこれを 『子ども』 と呼んでいます。
子どもがそこにいないときには、数学もなく恋もなくめい想もありません。それは全くの真の願望、夢中で遊んでいる子どもの願望でした」

数学をするというのは、とても純粋な行為なのかもしれません。
私は、この言葉を見た時、数学者の岡潔が、四歳までの子どもの気持ちで数学をしていると言っていたのを思い出しました。
グロタンディークは、「お母さん、見て、金魚が食べてるよ!」というような小さな子どもの喜びが大事だといいます。
そんなところに数学と向き合う秘訣があったのですね。

グロタンディークは、無邪気さについては、次のようにも述べています。
「私たちの宇宙を閉じ込めている 『目に見えない、絶対的な枠』を乗り越えさせるのは、才能や野心ではありません。この 無邪気さ だけが乗り越えるのです」

グロタンディークは、「ある数学者と他の数学者の相違を生んでいるのは、
あるいはひとりの数学者のある仕事と別の仕事の相違を生んでいるのは、いわゆる 『頭脳の力』 ではない ように思います」と述べています。

「数学上の事柄の美しさに対する開かれた態度、または、感受性のもつ繊細さ、微妙さという質」
が、ある研究者と他の研究者、または、ひとりの研究者のある時期と他の時期の相違を生んでいると言うのです。
https://ameblo.jp/math-field/entry-12338301975.html
0649132人目の素数さん
2021/07/29(木) 03:39:27.22ID:4+VK6Ht4
>>646
用語とかも全然わかりませんが、望月さんの研究は基礎論とも関わりがあるのですね
0650132人目の素数さん
2021/07/29(木) 03:41:40.20ID:4+VK6Ht4
>>647
はい
それこそラッセルの『数理哲学序説』を何回も読む程度です。今は公理的集合論の本に挑戦しようかなと考えています。ロジックの良い題材は見つけられていません。
その本は図書館で借りてみようと思います。
0651132人目の素数さん
2021/07/29(木) 03:42:07.98ID:azGs4fWk
>>644
>>微積の授業の時は、何故もっと厳密にやらないのか〜
これは実際の教育的な事情だと思う。
まず議論のスタート地点としてどこを設定するかでどこまでの内容を扱えるかが全然違う。要は時間の問題。
一階述語論理から初めて実数を定義するだけでも余裕で半期は掛かる。

疑って疑って疑って議論の細部を根幹まで還元していく姿勢って数理論理学を好む人には向いてるだろうけど、
多くの人は「まっ、有理数ぐらい当然だよな。」「素朴に集合を操作しても別に変な集合を作るわけでもないから好きにしていいよな」って思ってるから、議論を根本に還元していこうなんて思わない。
0652132人目の素数さん
2021/07/29(木) 03:45:43.46ID:7hKXYqER
数学は天才では無いほうが向いてるとおもってる

天才とは何だ??ブラームス型凡楽のすすめ
以前、とあるクラシックの音楽番組からブラームスについての取材を受けたことがある。「いや、彼は天才じゃないでしょう」と応えたところ、取材はボツになった。

ブラームスは「保守的で」「新しいことをやらない」「才能のない作曲家」なのに、現代に至る〈クラシック音楽〉の基礎を作った。そこが凄いのだ。
言ってみれば、校長先生の訓辞みたいな…内容があると言えばあるけど、おもしろみがないと言えば言える…という音楽なのだが、なぜかアマチュアでもプロでも安心して演奏できる「クラシック音楽の基本の基本」がそこにはあるのだ。

確かに「天才」の凄さは一目瞭然だが、ブラームスはその逆。全く一目瞭然でない地味なところから、100年間にわたってじわじわ効くボディブロウを叩き出す。

考えてみれば、バッハ・ベートーヴェン・ブラームスの三大Bは3人とも生まれつきの天分に恵まれた…という意味での「天才」とはほど遠い、後天的かつ結果論的な「巨匠」。
むしろ、天分に恵まれなかったことで生涯ジタバタし通しだった印象の方が強い。

早期促成栽培型天才少年は、多くの場合、ピーク時の直後(三十代後半あたり)に大きな壁に直面し、その際に早死にしてしまうか、その後才能を枯渇させてしまうことが実に多い。
「十代で才能を発揮して、二十代でいっぱしのプロ」というタイプの天才型作曲家の系譜は、前期ロマン派を境に途絶えてしまう。

そして、その後といえば、20歳過ぎてもさっぱり才能の片鱗を見せなかったような怪しい経歴の「天才」が俄然多くなる。
確かにスタートは遅いが、いわゆる知能指数は高そうな青年ばかりなので、一旦道を定めるとその吸収力が驚異的なのも共通項。
ほぼ数年で最低限の音楽の基本はマスターしてしまい、基礎に縛られないぶん革命的な理想に燃え、下積みの苦労を経て20代後半か30歳近くなってようやく楽壇に登場。その後は明確な個性を持って音楽界に作品を提供し続ける。

かくして、この種の「戦略家」たちが跋扈し始める時代以降、もはや無垢な「天才」たちに出る幕はなくなったとも言える。
そして、ブラームスはそんな時代を象徴する「天才ではない巨匠」なのだ。
http://yoshim.cocolog-nifty.com/office/2010/06/post-38d4.html
0653132人目の素数さん
2021/07/29(木) 03:47:06.89ID:Qt5HfYef
>>648
とても真に迫った言葉ですね。
グロタンディークの自伝と岡潔の随筆も借ります。

自分が今、もし、現在の数学科に不満で、数理論理学のようなものや、「数学とは何か」を考えるようなことをやりたい、興味がある、ワクワクすると思うなら、それを止めてはならないということですかね。

無理に常識に従わせる必要はない、というような。

スケールを小さく考えてはいけませんね。
数理論理学をしたいなら、そのワクワクに従えばいいということですかね。

転科したところで、やりたいことができるかはわかりませんが。
0654132人目の素数さん
2021/07/29(木) 03:54:10.22ID:7hKXYqER
この戦略家とブラームスの話(>>652)とグロタンディークはつながってるだろう
このタイプは天才とはちがうとおもうが

ジャン・デュドネ - Wikipedia
ジャン・デュドネ(1906年7月1日 ? 1992年11月29日)はフランスの数学者。
抽象代数学、関数解析、の研究で知られる。更に数学史の研究者でもあり、代数的位相幾何学や関数解析学の歴史についての研究がある。

ブルバキの主要人物であり、アレクサンドル・グロタンディークをローラン・シュヴァルツとともに薫陶し、
グロタンディークとともにEGAをブルバキのペンネームで書いた。特にグロタンディークの初期の仕事に非常に大きな影響を与えた。
また、現代数学のほぼすべてに精通していた貴重な数学者の一人としても有名であり、数学全般にわたる著述も多い。
また60年代、フランスの数学教育の現代化を唱え、ユークリッド幾何学の廃止を主張した。

数学者には戦術家と戦略家の2タイプがおり、前者は古典的手法を主に使って、新しいひねりを加える手際の良さがあり、以前試みが回避された解に到達する。
一方で戦略家は得られた解が明らかであるまで徹底的に分析しつくし、とにかくあらゆる概念間の関係が完璧に理解するまで絶対に納得出来ないタイプだという。
特に後者こそが現代数学の発展には必要であるが、一部の勘違いした連中や初等数学においてはそうとも言えず誤解されているとも言っている。
0655132人目の素数さん
2021/07/29(木) 03:55:57.39ID:ULh4or5m
>>651
はい。その通りです。
数学のスタート地点(少なくとも微積において)というのが、確定的なのかは知りませんし、逆にどこまでも辿っていけば、それだけで数学的研究になるのかもしれません。
というか、自分はそもそも微積に対してなんの面白さもありませんでした。興味もあまりありません。

私のイメージは「地球という対象があった時、人はどこを見るのか」だと思います。
私はどんどん内部を掘っていって、いつかマントルを破り、核にたどりつければいいな、なんて思っています。実際にたどり着けるかはどうでもいいのですが。
ですが、数学科に来てある程度時間を過ごすと、多くの人は地球の上に何を立てるのかに興味を持っている人は多いなと、感じました。
地球の外に興味がある人もいますね(これは本当にそういう意味で)。

実際にどんな成果がでるかとか、結果を残したいというようなことには興味がありませんが、とにかく根本を考えたいなぁ、なんて思います。
0656132人目の素数さん
2021/07/29(木) 04:03:15.88ID:ULh4or5m
>>654
ブラームスの話は、まとまってて普通に面白いですね。
デュドネの名前が出たのは驚きました。
僕は一時期何故かデュドネを意識していた時期があったからです(彼の書物のせいでしょうか)。

天才という定義は分かりませんが、デュドネや音楽家たちの例を借りれば、「とことん考えつくす」ということでしょうか。

そういった人間は現代教育では淘汰されているのかも知れませんね。
自由な数学の場でさえ、現在は高校教育のように、コレとコレと枠組みを作って、それからあぶれた人間は落ちこぼれのレッテルを作っているようにも見えますね。
0657132人目の素数さん
2021/07/29(木) 04:04:21.00ID:azGs4fWk
>>655
哲学をしたいのか数学をしたいのか
簡単な目安としては形式化できる議論をしたいのかどうかだな

哲学まで行ってしまうと、「それってあなたの感想ですよね?(ひろゆき風)」みたいになるから
0658132人目の素数さん
2021/07/29(木) 04:15:20.73ID:TX2mErrN
>>657
自分語りで申し訳ないんですが、

自分は高校時代はいわゆる文系で、もっと言えば美術志望で、そこから哲学にたどり着き、『論理哲学論考』を読んで(読破できたわけはありません)、哲学や言語論理学にある程度限界を感じました(そちらの研究者に怒られるかも知れませんが)。
それこそ、言語はコミニケーションにおけるゲームのようなもの、とも思えますね。

そんな時、ラッセルの『数理哲学序説』に出会いました。最初は何もわかりませんでしたが、数学のなんとも言えない面白さを感じました。

なので哲学にいきたいとは思いません。

もっと言えば、哲学を形式化できればもっと簡単なんだろうなとは思います。
ただ哲学的な話題にはあまり興味がありません。
自分の経験に勝るものはないからです。
0659132人目の素数さん
2021/07/29(木) 04:24:33.42ID:7hKXYqER
数学やるなら岡潔を勉強したほうがいいとおもってるが
うまいこと想像や創造や直感や幸福の仕組みが説明されてるかと
自分が知らないだけで、他の人も同じ様なことを言ってるのかもしれないが
自分でホームページ、ブログ作って解説しようとはおもってる
0660132人目の素数さん
2021/07/29(木) 04:26:55.31ID:7hKXYqER
でも岡潔の数学研究ではなく文書ね
自分は岡潔の論文には手を出して無く、そっちはどうなのかわからないが
0661132人目の素数さん
2021/07/29(木) 07:40:35.34ID:NQwrGg8z
B.ラッセルの「幸福論」も参考になるかもしれない
0662132人目の素数さん
2021/07/29(木) 08:58:48.85ID:1g3znWcK
>>651
君みたいな方向は危険だ。
数理論理学でも何も産み出せずに終わる可能性が高い。
0664132人目の素数さん
2021/07/29(木) 09:04:05.79ID:1g3znWcK
ID:7hKXYqERはインチキ臭が強い。
時代の変革者は常に目一杯古臭い、というのは科学史の常識であって卓見でもなんでもない。
0665132人目の素数さん
2021/07/29(木) 09:11:12.83ID:1g3znWcK
>>663
疑ってを先に持ってくるのは厳密化にエネルギーを浪費して実り少ないことになる。
岡もグロタンディークも厳密な推論をしていたわけではない。多分こうだろう、でどんどん前進していった。
0666132人目の素数さん
2021/07/29(木) 09:31:56.04ID:NQwrGg8z
>>662
自分は産み出せたわけね
0667132人目の素数さん
2021/07/29(木) 09:38:20.84ID:7hKXYqER
突拍子もない発想するのを天才として、そうではないのを天才扱いしてるのがある


ニーチェは「日頃から地道な厳しい練習を積み重ねている選手でさえ圧倒され、『天賦の才』のなせる業と思ってしまう。」という問題について、突き詰めて考えぬいていた。
また、偉業を成し遂げた人びとを、「職人」と考えるべきだという。
「天分だの、天賦の才だのと言って片付けないでほしい!才能に恵まれていない人びとも、偉大な達人になるのだから。
達人たちは努力によって偉業を成し遂げ、(世間の言う)“天才”になったのだ。
彼らはみな、腕のたつ熟練工のごとき真剣さで、まずは1つひとつの部品を制かっくに組み立てる技術を身につける。
そのうえでようやく思い切って、最後には壮大なものを創りあげる。
それ以前の段階にじっくりと時間をかけるのは、輝かしい完成の瞬間よりも、むしろ細部をおろそかにせず丁寧な仕事をすることに喜びを覚えるからだ」
http://kyouikukaikaku-2020.com/2019/06/12/27/


野の風に揺れる花々や、高原に流れる小川を見て美しいと思い、夜空に輝く星を見上げたり、山頂から壮大な雲海のながめを見て、大自然の崇高さ、荘厳さに胸を打たれたことのある人は、おそらく少なくないでしょう。
聖書は、次のように述べています。「神の目に見えない性質、すなわち、神の永遠の力と神性とは、天地創造このかた、被造物(造られたもの)において知られていて、明らかに認められる」(ロマ一・二〇)。
ですから、宇宙について深く研究しようとする科学者が、その研究を通して、神の存在についての確信を一層深めたとしても、不思議ではありません。
アインシュタイン博士は、「宇宙の法則は、数学的美しさをもっている」と語りました。
また、イギリスの数学者で物理学者であるディラックは、「神は、非常に優れた数学者であられ、宇宙を造る際に、極めて高度な数学を用いた」と書いています。
宇宙を秩序正しく支配している諸法則は、まことに荘厳なまでの美しさを持っています。
1927年にノーベル賞を受けたアメリカの科学者アーサー・ホリー・コンプトンの言っているように、「秩序正しく広がっている宇宙は、『はじめに神が天と地とを創造した』という、最も荘厳な言葉の真実さを証明するもの」なのです。
http://www2.biglobe.ne.jp/remnant/kagaku02.htm
0668132人目の素数さん
2021/07/29(木) 09:42:11.79ID:vvxAzdos
数学のトウシロウの大方は阿呆な厳密厨。
BAとか典型だよね。
しかも拘ったのに展開する論証は本質的な間違いだらけwww
重箱の隅をつつくだけで大局を見ない池沼は何をやってもあかん。
0669132人目の素数さん
2021/07/29(木) 09:45:23.41ID:ck4rS0By
ID:7hKXYqER

こいつのレスは、典型的に阿呆の長文、修辞と権威づけで誤魔化すだけで無内容。
0670132人目の素数さん
2021/07/29(木) 09:47:02.91ID:azGs4fWk
>>665
>>疑ってを先に持ってくるのは厳密化にエネルギーを浪費して実り少ないことになる。
全くその通り。俺みたいなのは論文読んで分かるのが精一杯。

将棋の羽生善治も「点が線になる」という言葉で例えてるが、レジェンドレベルの天才は細部の筋道をすっ飛ばして合理化は後に持ってくる。
でもそんな事ができるのは天才だけ
0671132人目の素数さん
2021/07/29(木) 09:53:30.99ID:7hKXYqER
羽生も天才ではないとおもうが

羽生善治が教える「直感」の正体
生まれながらに勘に優れ、何かを判断するに当たっては、ピーンとすごいひらめきが降りてくる−。ちょっと陳腐かもしれませんが、直感のイメージといえば大体こんなところでしょう。
羽生さんは、直感は決して先天的なものではなく、日常の訓練で磨くことができると言い切っています。
「直感は、本当に何もないところから湧き出てくるわけではない。考えて考えて、あれこれ模索した経験を前提として蓄積させておかねばならない」
「もがき、努力したすべての経験をいわば土壌として、そこからある瞬間、生み出されるものが直感なのだ」(本文より引用)
そうした蓄積が脳回路を鍛え、ほぼ無意識の状態でパッと答えを導き出せるのが、「直感の正体」なのだそうです。
羽生さんは、そんな直感が生まれるときを「論理的な思考が直感に昇華した瞬間」と表しています。
「なんだ、結局は地道に努力するってことか」とガッカリされる方もいるかもしれません。
ですが、「羽生マジック」とも称されたほど誰も思いつかない一手で鮮やかに勝利してきた、あの羽生さんの直感が、やはり地道な努力の土壌から生まれていたことに、むしろ驚きと安心を感じないでしょうか。
そして、その力は鍛え磨くことができるとまっすぐに説く羽生さんに勇気づけられます。
https://paraft.jp/r000017003127

阿部隆 七段
私が「羽生さんは天才タイプだと思うけど、谷川さんはどうかな。僕にはどちらかというと秀才タイプに見える」と言ったら、
阿部はすかさず「まったく逆ですよ」と異議を唱えた。「それって羽生さんのほうが努力家タイプだという意味?」 「そうですよ。池崎さんは誤解してます」
shogipenclublog.com/blog/2018/06/29/abetakashi-2/

ひふみん「藤井さんの勝負勘は素晴らしかった」 朝日杯 :朝日新聞デジタル
14歳、15歳のころの研究量では1番ではないか。2位は羽生さん。努力して強くなる秀才型。
攻めはさえて、受けは強い。ひとことでいうと欠点がない。どんな棋士にも欠点はあるもので、これはものすごいことなんです。

羽生善治 「天才というのは、村山くんや谷川先生の事だよ。自分はただ、努力し続けてるだけ。」
shogiweblog.net/archives/682
0672132人目の素数さん
2021/07/29(木) 09:54:33.49ID:S8ipbRu5
数学を将棋に喩える…
これも数学の出来ない人に良く見られる
0673132人目の素数さん
2021/07/29(木) 09:55:17.27ID:azGs4fWk
>>それ以前の段階にじっくりと時間をかけるのは、
>>輝かしい完成の瞬間よりも、むしろ細部をおろそかにせず丁寧な仕事をすることに喜びを覚えるからだ
結構この言葉響くな
0675132人目の素数さん
2021/07/29(木) 09:57:09.02ID:1g3znWcK
綺麗なノート作りに情熱を注ぐ学生みたいな話か
0676132人目の素数さん
2021/07/29(木) 09:57:36.77ID:S8ipbRu5
>>670
天才うんぬんは関係ないぞ。
逐次近似法を知らないのか?
数値計算で無い推論でも、予測に修正を重ねていくのはごく普通のことだ。
0678132人目の素数さん
2021/07/29(木) 10:01:13.68ID:1g3znWcK
天才というか既成の教育システムで得られない能力を持つ人がいるのは厳然たる事実。
0679132人目の素数さん
2021/07/29(木) 10:13:11.54ID:7hKXYqER
両極的なtypeがいる

ディドロに演技論を書かせた女優たち
彼が俳優の「感性」について考えてみるに当たって,身近に観察し,また実際に交際していた女優が二人いる。
マリー・デュメニルから見ていこう。ディドロはこの女優が大嫌いだったろうと推測できる。
その理由は,ディドロが俳優に絶対的に欠如しているべきであるとした「感性」を「天性」と呼んで,これに頼って演技し続けた女優だったからだ。
彼女はどうやら技法を身につけ,磨きをかけるべく不断の努力を続けるのではなく,舞台に上る度ひたすら「神様がおいでになる」のを待っていたようである。
しかし,これでは演技にむらが出すぎるのは否めない。
ディドロにとって,最も理想的な資質を有していると思われた女優がクレロンだったのだろう。その相違は,次のようなものだったと言う。
デュメニルは,時々は頂点に達しうる,生まれながらの気まぐれな女優で,一方クレロンは,つねに申し分なく気品にあふれてはいても,恐らく感情の最も深い琴線に触れること決してなかった,意識的な芸術家であった。
https://m-repo.lib.meiji.ac.jp/dspace/handle/10291/13944


天才は、なぜ努力型に勝てないか(天才・升田幸三vs努力の人・大山康晴)
大山名人を何度も泣かせた升田幸三だが、天才特有の弱点があった
升田幸三は100年に一人出るかどうかの天才棋士だ。次々と凄い新手を作り出し、時に手が付けられないほどの強さを見せる。
木村義人名人が将棋界の天才について語る。「(後輩では)塚田、升田、大山らがいるが中でも升田の将棋は最も天才にふさわしい。
ただし天才には出来不出来が多く、努力型はそれが非常に少ない。だから技術が高くなるほど努力型が勝ち続ける」
大山康晴の天才論。
「プロというのはその道で優れた人の集団であり、天才といわれた人でも周りが皆そういう人の中にあってはただの人になってしまう。
だから私はプロの世界には俗に言う天才は居ないと思う。
将棋の世界で強いて天才がいるとすれば、天才とは出来不出来の多い人のことを指すのだと思う。
私が思うに、作家でも画家でも七十、八十になっても衰えを知らず、いい仕事を続ける人こそが真の天才の名に値するのではないか」
http://opu-shogi.chips.jp/www/fudaiOB/essay/essay20041019_tensaihanazedoryokugatanikatenaika.htm
0680132人目の素数さん
2021/07/29(木) 10:32:57.49ID:2UF+EZjZ
こいつは数学好きを自称する数学史雑学好きなんだろうなという気がする
0681132人目の素数さん
2021/07/29(木) 10:39:45.07ID:mbJHSYG5
数学史の「専門家」でも知識の怪しいのがいてね。
某大学の教授でオイラーを「フランス人」と言ってたのが居てまいった。
デイドローがサロンが…と喚いている内に混線したらしいw
揶揄されがちの高瀬正仁だが、流石にこんなアホなことは言わない。
0682132人目の素数さん
2021/07/29(木) 11:31:40.78ID:NQwrGg8z
>>681
フランスでオイラーの子孫に会ったといういう話と
混同しているのではないか
0683132人目の素数さん
2021/07/29(木) 12:20:59.28ID:7hKXYqER
チコちゃん、「ボーっと生きてる」ほうが体に良いんだって
米ワシントン大学医学部のM.E.レイクル教授の研究で、“ボーっとしている時のほうが、集中している状態よりも脳の広範囲が活性化している”と発表されたのです。
レイクル教授によれば、集中して活動している時の脳のエネルギー消費量に比べて、ボーっとしている時は、脳の活動が高まり、約15倍ものエネルギーを消費しているというのです。
それはデフォルト・モード・ネットワークと呼ばれる脳の広い領域が活発に働いているからだと考えられています。
news.livedoor.com/article/detail/16071356/

クリエイティブな仕事のプロは脳の活動が普通の人とは違う
実験の結果、近い未来を想像しているときはプロと対照群の間に脳の活動の差はほとんど見られませんでしたが、
遠い未来のことまで想像しているときには、プロの脳では「背内側デフォルトモードネットワーク」という部分が活性化することが判明したとのこと。
「背内側デフォルトモードネットワークは自分自身の経験とは異なる視点で物事を考えるときに関わってくる部分であるため、
脳のこの部分を活性化できる人は、他人に対する共感性を持ち、政治的な決断を下すときも未来の子どもたちへの影響を考えられるような思慮深い人である可能性があります」
gigazine.net/news/20190622-creatives-using-specific-brain-part/

【アインシュタインの脳】 NHK 〜失われた“天才脳”の秘密に迫る〜
1. 言語、数学、感覚統合に関わる頭頂葉の右上頭頂小葉、左下頭頂小葉が他のヒトより大きい。さらに左頭頂葉におけるグリア細胞が73%も通常ヒトより多いということです。
2. デフォルトモードネットワークに関わる前頭葉と頭頂葉がとくにアインシュタインの脳内では発達していたようです。
3. アインシュタイン20代の脳は70代より計画、判断や推論に関わる前頭葉と頭頂葉は大きく発達していたようです。
eduzenn.muragon.com/entry/159.html

かつて羽生と谷川の対局中の脳波を調べる科学的な実験が行われた。
「お二人を比較すると、谷川さんが集中型なのに対し、羽生さんは瞑想型でした。羽生さんの場合、非常にリラックスされていて、いわば座禅を組んで瞑想している高僧のような脳波が出ている」
taichi-psycho.cocolog-nifty.com/adler/2006/01/post_20b7_2.html
0684132人目の素数さん
2021/07/29(木) 12:33:07.75ID:7hKXYqER
羽生善治やアインシュタインは天才ではないという前提だが
天才はべつにいて、天才との違いはざっくり言えば努力だが
しかし、天才も努力はしているだろうから、決定的な違いと言い難い
それで、脳の働かせ方が違うんだろうと
>>683でも貼ったが天才は瞑想型=起きながら寝てるような半覚醒状態にはなりにくいんだろうと
単純作業っぽいのが苦にならない人が瞑想型になりやすいのかと
0685132人目の素数さん
2021/07/29(木) 12:37:49.88ID:od3vylOI
オカ板でやれクズ
0686132人目の素数さん
2021/07/29(木) 15:36:16.45ID:MbCV6Tf6
実のあることを書けなくても天才を話題にすれば仲間になれた気がするんだろなー
哀れ
0688132人目の素数さん
2021/07/29(木) 16:34:45.79ID:iknLpEtD
意義深い話しろとは言わんから
せめてロジックに関連ある話にして欲しいよね
0689132人目の素数さん
2021/07/29(木) 16:42:05.47ID:pny8Pqxh
超準解析のいま、誰か解説してくんない?
0690132人目の素数さん
2021/07/29(木) 18:04:57.36ID:7hKXYqER
自己組織化とは
非常に抽象化して「自己組織化」を説明すれば、「自己組織化システムとは、経験と環境の関数として基本構造が変化し、合目的的システムが自然にでき上がること」と定義することができるでしょう。
例えば、人間は自己組織化システムです。太陽系も自己組織化システムだと言うことができます。
現代的な意味でのニューラルネットワーク にとっても上記のような意味での「自己組織化」は実現されていません。
外界の情報すなわちデータの相互関係を効率良く表現することは情報科学の分野でも中心的な問題であり、おそらくこのような能力が脳の働きの特徴の1つであるということができるでしょう。
www.cis.twcu.ac.jp/~asakawa/chiba2002/lect7-som-memo/index.html

数学者の孤独な冒険 第1巻
期待される革新(これが再びおこるものとして...)は、物理学者からよりも、むしろ物理学の大問題によく通じている、根っからの数学者からやってくるだろうと私は予測しています。
だがとくに、問題の核心を把握するためには、「哲学的に開かれた心」をもっている人物が必要でしょう。
私たちがよく知っているように、観察者は、「その過去」の影響を受けているだけでなく、自分の自由意志と自分の中にある創造性によって、「その将来」に対して影響を与えます。
したがって、「観察者の前にある」時空のこの部分の構造自体、とくに物質とエネルギーの流れによって叙述さえる構造は、あらかじめ「すべてできあがっている」ものではなく、ある程度は、「自己創造している」のです。
構造の「土台」である、時空自体がそれらにつれて自己創造しているものとしてみるようなモデルを構想する必要がありそうです。
もちろん、私は「観察者」ということで、人間の観察者だけではなく、宇宙のおのおのの「場」で、おのおのの時点で活動しており、宇宙の息吹き、あるいは生命そのものとして存在している創造的な知性をも考えに入れています。
この「息吹き」は、現在までに構想されたモデルに存在しないだけでなく、禁じられているように思われます。
明らかに、この息吹きを含めるには、いままでに提出されているすべてのモデル、つまり「出来合いのモデル」に欠けている柔軟性、内的なダイナミズムを与えるために、時空という基本概念についての深い再考察が必要とされるでしょう。
quasimoto2.exblog.jp/238872006/
0691132人目の素数さん
2021/07/29(木) 18:30:12.30ID:7hKXYqER
量子よりアメーバコンピュータのほうがいいとおもうが、上の自己組織化とも関連して


粘菌 脳のない天才
「粘菌」と呼ばれるアメーバと菌類の特徴を合わせ持つ単細胞生物が、今科学の世界で注目されている。
脳も神経もない粘菌の“知的”な性質を活用する研究の最前線を描く。
粘菌の持つ“知的”な性質の一つが、迷路の出口までのルートを最短距離で探し出す能力。
粘菌を使って東京を中心とした首都圏の効率的な鉄道路線図を作成した科学者は、イグ・ノーベル賞を受賞した。
他にも医療や環境問題、さらにはバイオコンピュータの開発にも粘菌活用の期待が寄せられている。
http://www6.nhk.or.jp/wdoc/backnumber/detail/?pid=191125


「アメーバ」は人間よりも迷路攻略が得意だった!行き止まり回避能力の謎が解明される 2021.01.27
https://nazology.net/archives/68375


1枚のFPGAで最大3万変数の組合せ最適化問題を1秒で解く??単細胞生物アメーバを応用したコンピューター開発
2021/03/05
https://fabcross.jp/news/2021/20210305_ameba_computer.html
0693132人目の素数さん
2021/07/29(木) 19:43:44.32ID:1g3znWcK
オリンピック金メダリスト アナ・キーセンホーファーの言葉

若くて何も知らない選手には "これをすれば上手くいく” とコーチや周りの人間に言われる危険がつきまとう。私も一時それを信じ、そして被害者の1人だった。だがいま30歳になり、何かを知っている人なんていないことを学んだ。なぜなら本当に何かを知っている人は「知らない」と言うからだ。
0694ハノン ◆QZaw55cn4c
2021/07/29(木) 20:17:58.37ID:xRSR7u3p
>>635
数学科では絶対にやらないラプラス変換で転向組は墜落するときいています
0696ハノン ◆QZaw55cn4c
2021/07/29(木) 20:33:11.80ID:xRSR7u3p
>>652
http://yoshim.cocolog-nifty.com/office/2010/06/post-38d4.html のコピペですねえ‥‥

>「保守的で」「新しいことをやらない」「才能のない作曲家」

×保守的で
あんがいそうでもないようですよ。伝統的な弦楽四重奏はベートーベンほどには書かず、むしろ弦楽六重奏が絶品だったりするんです https://www.youtube.com/watch?v=jHSgWAN_t1k

×新しいことをやらない
ピアノコンチェルトの癖にチェロのソロ、バイオリンコンチェルトの癖にオーボエソロ、という風に目立たないところでバンバン新しいことをやっていますよ
ピアノコンチェルトをシンフォニーにしてしまったのも目新しい

×才能のない作曲家
あれでブラームスはムンムンです、「ブラームスから色気と才能を引いたのがシベリウス」という言葉を思い出しました
0697132人目の素数さん
2021/07/29(木) 20:47:15.84ID:7hKXYqER
宇宙を形成する銀河同士のネットワーク「コズミック・ウェブ」を粘菌で再現  2020年3月21日
粘菌。このアメーバ様単細胞生物が、宇宙最大の謎の1つの解明を手助けしてくれるのだそうだ。
カリフォルニア大学サンタクルーズ校の研究グループは、3万7000以上の銀河のアーカイブデータを選別し、高度なアルゴリズムで目に見えないフィラメントをマップ化することにした。
開発のヒントになったのは、なんと「モジホコリ」という粘菌の仲間だ。
www.excite.co.jp/news/article/Karapaia_52288922/


「宇宙と脳は構造が似ている」と数学的に証明される! 宇宙は脳で脳は宇宙だった? 2021.01.27
11月16日に『Fronties in Physics』に掲載された論文によれば、脳内のニューロンと宇宙の大規模構造の類似性を数学的に証明することに成功したとのこと。
以前から両者の構造が似ている理由は、構成割合の類似性が原因ではないかといわれています。
しかし脳と宇宙の類似性は、真面目な科学者にとっては、常に一歩引いて接する問題でした。
なぜならば両者の構造は一見して似通ってみえるものの、一方は生命活動の結果であり、他方は重力を主とした物理学的な力によって引き起こされた結果だからです。
ネットワークの類似性を数学的に解明
ニューロン(特に小脳)と銀河の類似性を調べるにあたり、2人の研究者たちはまず最初に、双方の密度のゆらぎ幅に着目しました。
驚くべきことに、密度のゆらぎ幅は、宇宙でも同じでした。自然界には網目状をとる多くの構造が存在しますが、密度のゆらぎ幅まで一致することは多くありません。
次に研究者たちは脳のニューロンネットワークと銀河の網の結節点の性質について調べました。
分析の結果、ニューロンの場合は1つの結節点に4〜5本の軸索が伸びている一方で、銀河の網の結節点にはおおよそ4本の銀河の連なりが伸びていました。
さらにニューロンネットも銀河の網も、中央の結節点の周りに接続のクラスターを作りやすいという同じ傾向を示しました。
そして最も興味深い点として、ネットワークが記憶できる情報容量を比較した結果、両者はほぼ同じスケールの量を保持できることが判明したのです。
この事実は、ネットワークの接続性に限れば、同様の法則に従って進化していることを示します。
nazology.net/archives/73939
0698132人目の素数さん
2021/07/29(木) 20:48:39.97ID:1g3znWcK
>>696
君が吉松さんを誤読しているように思える
0699132人目の素数さん
2021/07/29(木) 21:01:59.09ID:7hKXYqER
これとも近いとおもうが

米津玄師「BOOTLEG」インタビュー|オリジナルってなんだ? “海賊盤”に詰め込んだ美と本質

“過剰なオリジナリティへの信仰”に対する答え
今回のアルバムは、自分の中ではオマージュのアルバムだと思っているんです。
それは自覚的にやっているんです。それをやることによって過剰なオリジナリティへの信仰みたいなものに対して自分なりに解答を出したかったっていうのがあります。

これは昔からふつふつと思っていたことではあって、ここ最近に始まったことではないんですけど……1つのきっかけになったのは「砂の惑星」ですね。
あの曲には、歌詞の中に昔の……金字塔的な、ボカロ界隈の曲のタイトルとか歌詞をオマージュとして入れ込むパートがあるんです。
面と向かって「もし自分の曲があんなふうに誰かの歌詞の中に使われたらイヤだろう?」みたいなことも言われたりして。
でも僕は「イヤなわけないじゃん」って思うんですよ。
何かと何かが似ているということだけを指摘して、作り手でもない第三者がそれを「パクリだ」とこき下ろすような風潮がある。
そういう考え方は、僕は端的に文化への冒涜だと思います。

皮肉めいた言葉だけれど、米津玄師の作品というものは今語ったような“過剰なオリジナリティへの信仰”が定義する“オリジナル”ではない。
いろんなものを摂取して吸収して、その土台の上にできたオリジナルであると。そこにこそ美しさが宿ると言うか。

誰も見たことのないもの、聴いたことのないもの、という意味でのオリジナルなものをどんどんを突き詰めていけば、最終的に残るのはノイズみたいな異物にしかならないと思うんです。
それはそれで美しいと思うけど、自分がやりたいのはそういうものではない。
自分が作りたいものは普遍的なものであって、それはいろんな人間の根本に流れている何かだと思うんです。
そう考えると、やっぱりどこかで聴いたことがあるものなんですよね。
だから、そういう過剰な意味を込めて“オリジナリティ”という言葉をあえて使うのであれば、俺はオリジナリティなんてどうでもいいと思っている。
そういう気持ちがすごく強いです。
https://natalie.mu/music/pp/yonezukenshi11
0700132人目の素数さん
2021/07/29(木) 21:43:56.28ID:7hKXYqER
岡潔講演録

自我を生かすと、個というものが生きてしまう。
自我を忘れる、自我を離れることを無我と云う。無私とも云う。
これが全とのつながりを持つということです。自我が個ですね。
特に生きる喜びは全から来るんです。自我から生きる喜びの来るわけなんかない。

晴れた日の自然の美しさはよくわかるが、雨の日の自然の趣きの深さはわからない、と云うのが今の若い人達ですね。
西洋人は絶えずそうだったし、日本人も今ではそうなって来てる。
雨の日の趣きがわからなくて、晴れた日の美しさだけがわかる。こういうのを刺激を求めると云う。
今世界の人々は、刺激から刺激と追い求めようとしてるんです。

しみじみとした深いものが良いのです、本当はね。が、それはわからない。
おれがおれがにいろんな種類があって、その各々を指して個性と云ってるらしい。
ところが個性というものを自然に求めてみますと、
どういうものが個性かと云いますと、松の含水炭素はどこへどう使われでも滴々みな松になる。
竹の含水炭素はどこへどう使われでも滴々みな竹になるでしょう。これが個性です。

だから個性を一段低く見てしまってる。
おれがおれがというのは穢れです。普通個性と云ってるものは、その穢れの色どりの種類です。
頭頂葉にある個性と云うのは、松が松であり竹が竹であるのと同じ。

http://www.okakiyoshi-ken.jp/oka-19-16.html
http://www.okakiyoshi-ken.jp/oka-22-08.html
0701132人目の素数さん
2021/07/29(木) 22:56:54.24ID:z5FLWbEa
>>686
なんかそんな感じにも見えるね
0703132人目の素数さん
2021/07/30(金) 13:26:43.56ID:N8gLnP70
NGしとけや
0704132人目の素数さん
2021/07/31(土) 15:16:34.30ID:BH051squ
含意命題について
ある論理式Qに対してその存在が保証されているとき

P→Q

となる任意の論理式Pがある.

これをたとえば述語論理で読み換えると

∀x∃yFxy すべての○○はある△△が関係する

とは

@∃yFxy   xはある△△と関係する
A∀x∃Fxy  すべての○○はある△△と関係する

とわかる.つまり関数f:X→Yについて

∀x[x∈X→∃x[x∈Y∧f(x)]


@∃x[x∈Y∧f(x)]
A∀x[x∈X]
の順序で読むことになる.
0706132人目の素数さん
2021/07/31(土) 15:21:21.92ID:BH051squ
自然演繹で書くと

|- P→Q

1 (1) Q     仮定
1 (2) P→Q   1.→-導入
0708ハノン ◆QZaw55cn4c
2021/07/31(土) 15:34:25.87
>>704-706
なんだかまどろっこしい説明ですね
普通に「任意の x に対して、任意の x に依存してもいい y がそれぞれの x について存在する」とかなんとかでいい気がします
0711ハノン ◆QZaw55cn4c
2021/07/31(土) 15:43:40.99
>>710
違いますね、x は自由に選んでもよく、どんな x についても成り立つ、のだから
∀x□
という形式でしょう?
つまり依存しているのは y で、 x の値が変わると y も変わるが、それでも必ず Fxy を満たす y が存在する、ってことでしょう?
0712132人目の素数さん
2021/07/31(土) 15:56:34.45ID:BH051squ
卵が先か鶏が先かの議論になることはわかっているので

P→QをQから読むというソースは

金子洋之『記号論理入門』 p.94-97 第6章6.3節

です.もし,これに異論や批判があったら
ここに書いて頂けるとありがたいです.
0713132人目の素数さん
2021/07/31(土) 15:59:15.95ID:DvvuMYvf
漢文じゃねぇし
0715ハノン ◆QZaw55cn4c
2021/07/31(土) 16:22:28.61
>>712
無論、Px→Qy を Q から先に読むことはできますよ

「Px → Qy を満たすような Px が存在する」
∀y∃x(Px → Qy)
であれば Q から先に読むしかありません

∀y∀x(Px → Qy)
∃y∃x(Px → Qy)
であれば、どっちから読んでも大差はないと思います

>金子洋之『記号論理入門』 p.94-97 第6章6.3節

その本持っていないので BTC で買う資金を送ってくれませんか?
BTC アドレスは 3HHTgVjACmY4f53Z5RVBqeiD7aZtHj9Ach
0716132人目の素数さん
2021/07/31(土) 16:28:44.98ID:BH051squ
匿名だと強気なゴミどもがw
0717132人目の素数さん
2021/07/31(土) 16:29:55.79ID:BH051squ
伝統的論理学や命題論理でいえることが
一階述語論理では失われるという理解でいいのかな?
ゴミはゴミ箱へ
0718132人目の素数さん
2021/07/31(土) 16:35:13.34ID:rV+RaEGj
本代だけで教えてもらえるなんて破格だぞBA
0720132人目の素数さん
2021/07/31(土) 17:04:44.18ID:tBNvG/C/
ひょっとして理科大君?
0721132人目の素数さん
2021/07/31(土) 19:59:01.76ID:i8E34p+N
>>704
>ある論理式Qに対してその存在が保証されているとき
その存在とは?
∃xQ(x)のこと?
>P→Q
>となる任意の論理式Pがある.
これ見ると違うみたいだけど・・
0722132人目の素数さん
2021/07/31(土) 20:27:17.14ID:Jth2+LgP
>>717
何を言っているのか分からないんだけど、とりあえず命題論理⊂述語論理だよ
0723132人目の素数さん
2021/07/31(土) 21:11:08.35ID:BH051squ
>>721
>その存在とは
>>706の仮定レベルのものです
命題を量化する前の命題論理の話です
0724132人目の素数さん
2021/07/31(土) 21:57:45.11ID:BH051squ
ああわかったわ
仮定を落とすという作業を忘れていた
参考文献も(1)と(2)だけではなかった
失礼しました

教えてくれてありがとね!
0725132人目の素数さん
2021/07/31(土) 22:33:22.14ID:BH051squ
いや仮定を落とすことは必ずしも条件ではないと記述されているw
やっぱり未解決だったw
あーわからん
0727132人目の素数さん
2021/08/01(日) 00:12:46.78ID:fEcpSWLO
>>723
意味不明よ
>ある論理式Qに対してその存在が保証されているとき
たとえば
¬A∨B
という論理式をQとしたとき
この論理式Qに対して保証される「存在」とは何?
0728132人目の素数さん
2021/08/01(日) 00:15:43.08ID:fEcpSWLO
>>708
君は ID:BH051squ の>>704の意味が分かっているようだから
説明してもらえるかな
0729132人目の素数さん
2021/08/01(日) 00:29:53.21ID:KoBKtwBR
>>727
それは論理式の定義の問題じゃないかな
Aが論理式のとき¬Aも論理式
というような感じの
ただし存在することを定義できないとかいう哲学はなしでw
0730132人目の素数さん
2021/08/01(日) 00:33:19.01ID:KoBKtwBR
つまり

AとBが論理式のときA∨Bも論理式

と先の¬Aを組合わせれば

¬A∨Bは論理式である

この論理式が存在性を保証する
と解する他ない
俺自体から出てきた哲学ではないので棒読みしかできない
0731132人目の素数さん
2021/08/01(日) 00:34:22.50ID:KoBKtwBR
論理式を定義するということが存在を担保し
その存在性は自然演繹において仮定するというレベルでよい
0732132人目の素数さん
2021/08/01(日) 00:35:39.98ID:KoBKtwBR
論理式の存在性が仮定レベルでよいというのは含意命題に限るけどね
0733132人目の素数さん
2021/08/01(日) 03:07:03.43ID:fEcpSWLO
>>729
論理式の定義は明確
命題変数は論理式から始まって帰納的に定義できるものが論理式
>>704
>ある論理式Qに対してその存在が保証されているとき
というのは君の解釈では
「Qが論理式であるとき」
という意味だと?それで良い?> ID:BH051squ

(そうだとしてそういう意図を
「ある論理式Qに対してその存在が保証されているとき」
と書くのはにいかがなものかと思うけどね)
0734132人目の素数さん
2021/08/01(日) 03:10:24.91ID:fEcpSWLO
>>732
>論理式の存在性が仮定レベルでよいというのは含意命題に限るけどね
何を言っているのか意味不明
0735132人目の素数さん
2021/08/01(日) 03:12:37.86ID:fEcpSWLO
ID:KoBKtwBR = ID:BH051squ ですか?
なんか酷くこじらせちゃってる人みたいだけど
0736ハノン ◆QZaw55cn4c
2021/08/01(日) 03:45:49.61
>>728
>>708,711 をみていただければわかるように私は >>704,710 に真っ向から反逆していますから >>704>>704 で主張したかった意図は当然のように理解していませんよね
0737132人目の素数さん
2021/08/01(日) 09:04:11.99ID:fEcpSWLO
>>736
そう?
じゃあ意図を理解していなくてそれに反論したということか
それもいかがなものかなあ
0739ハノン ◆QZaw55cn4c
2021/08/01(日) 16:41:09.46
>>737
意図が理解不能だから反論する、というのは十分に成立すると思いますよ
0740132人目の素数さん
2021/08/01(日) 19:43:56.15ID:hYKILjpl
>>739
理解不能だってことを言ってないでしょ?
「まだるっこしい説明」というのは内容は理解できた上でそれが長すぎるという表明だと思ったけど?
理解した上で反論しているのと
理解せずに反論するのとは雲泥の違いだよ
0741132人目の素数さん
2021/08/01(日) 19:45:03.04ID:hYKILjpl
>>738
そうかもね
彼は「何かそれっぽいことを言いたいだけ」のような気もする
0742ハノン ◆QZaw55cn4c
2021/08/01(日) 19:50:32.30
>>740
それは >>708 の時点での私の理解ですね。 >>708 の時点で >>704>>708 の理解でよいのだと私は考えたのですが、 >>708 を受けたレンポンスは

>>710 いやxがyの選び方に依存しているのだが

これは私の理解と完全に真逆ですね、それを私は >>711 で明示しておきました
したがって >>728 の時点から現在まで >>704,710 を理解不能としています
0743132人目の素数さん
2021/08/01(日) 19:51:05.78ID:hYKILjpl
>>715
>>>712
>無論、Px→Qy を Q から先に読むことはできますよ
こういうこと書いてるんだから
彼が何を言いたいかは分かってると表明してるんだと思ったので
>>728
では
>>>708
>君は ID:BH051squ の>>704の意味が分かっているようだから
>説明してもらえるかな
と聞いたのでした
分かってなかったのならまあいいや
0744132人目の素数さん
2021/08/01(日) 19:54:12.19ID:hYKILjpl
>>742
>>>708 の時点で >>704>>708 の理解でよいのだと私は考えたのですが
なるほど
それはそれで先走りしすぎで慎重さに欠けた反応をしたってことだと思うので
それはそれでいかがなものかと思うけどね
まあいいや
いずれにしても
>>704
>ある論理式Qに対してその存在が保証されているとき
は無意味な言葉だったってことなら
もうどうでもよさそう
0745132人目の素数さん
2021/08/01(日) 19:55:54.31ID:hYKILjpl
ID:BH051squ
ID:KoBKtwBR
それでいい?それともちゃんと説明できるならして
0746ハノン ◆QZaw55cn4c
2021/08/01(日) 20:18:33.53
>>743
>>715・私
>無論、Px→Qy を Q から先に読むことはできますよ
>∀y∃x(Px → Qy)
>であれば Q から先に読むしかありません

と私が書いたからといって、それ以前にあった >>704 やそのの続きである >>710 を理解不能とすることに、私は矛盾を感じません
なぜならば私の持論の表明である >>711>>710 を真っ向から否定する、>>710 とは真逆の意見ですからね

あなたは字面ばかりみていて、論理学的文脈を理解しようとしていないようですね
0747ハノン ◆QZaw55cn4c
2021/08/01(日) 20:25:21.86
>>744
>それはそれで先走りしすぎで慎重さに欠けた反応をしたってことだと思うので

相手のいうことを自分は理解できているかどうかを確認を取るために、論理的につながりはあるが、敢えてそのつながりを明示せずに結果だけ明示した内容で問い合わせるのはよく行われることなのでは?
「つまり、あなたはこういうことがいいたいのですか?」っていう言い方をしませんか?
私はそれをしたまでのこと、そしてその結果、彼は私と真逆の思考だったことがわかったので、それはそれで一つの成果でしょう

それを「慎重さに欠いた」と評価するようでは、あなたの聞き手としての技量を私は疑いますね
0748132人目の素数さん
2021/08/01(日) 20:27:48.90ID:eX88Mz3m
論理式の存在性が仮定レベルでよいことの例

P→S,Q→(S→R)|- P→(Q→R)

1 (1) Q→R      仮定
1 (2) P→(Q→R)   1.→-導入

なお教科書によれば同様にして
¬-導入と∨-除去にも仮定落としが不要だといえるのだそうだ
0749132人目の素数さん
2021/08/01(日) 20:38:26.86ID:eX88Mz3m
もしこの法則が認められるなら数学がチョー簡単になるかもw
0750132人目の素数さん
2021/08/01(日) 20:41:28.00ID:eX88Mz3m
演繹の中で帰納的に証明される
これが本当の数学的帰納法か?w
0751132人目の素数さん
2021/08/01(日) 20:46:38.25ID:2kV/UVPv
ケリーの位相空間論(General Topology)って面白い?
0752132人目の素数さん
2021/08/01(日) 20:50:13.14ID:hYKILjpl
>>746
>あなたは字面ばかりみていて、論理学的文脈を理解しようとしていないようですね
あー
それでいいよ
0754132人目の素数さん
2021/08/01(日) 21:03:30.48ID:hYKILjpl
>>753
別にイイヨそれで
どうも意味ないコト言う人だったみたいだから(君じゃ無いよ)
もう興味なくなってるし
0755132人目の素数さん
2021/08/01(日) 21:12:21.59ID:IKQJ9hLo
数学基礎論あるある
文系が「論理式の存在性」や「仮定レベル」など定義のない自作用語を使ってまくし立ててくる
そして定義を聞いても答えてくれない
0756132人目の素数さん
2021/08/02(月) 02:47:10.68ID:K0uRt4F8
理論S'が理論Sの保存拡大ならば、
S'の任意の論理式Aに対してSの論理式A^*に変形する操作が存在してS'|-A⇔A^*が成り立つ


は成り立ちますか?
参考文献教えて下さい
0757132人目の素数さん
2021/08/02(月) 03:29:18.18ID:K0uRt4F8
>>756
理論Sに、定義による(関数or述語)記号の導入を有限回繰り返して得られる保存拡大たる理論S'に対しては結論が成り立つことは分かったけど
そうとは限らない保存拡大に対しては成り立つのかと気になったが、
そもそも無限個の記号を導入された保存拡大って考える必要がないから、気にしなくていいかな?
0758132人目の素数さん
2021/08/02(月) 10:37:26.56ID:hXcilsOO
成り立たないんじゃない?

xが自然数である事をN(x)、
実閉体Rの要素である事をR(x)とでも表現する事にして、

自然数論と実閉体の理論を
ただ形式的に合併して、
N(x)が成り立つ要素とR(x)が成り立つ要素に
関する理論にしたら、自然数論の部分については
保存拡大だけど、成り立たない気がする。
0760132人目の素数さん
2021/08/02(月) 12:05:40.23ID:K0uRt4F8
一応補足しておくと、理論Sに定義による(関数・述語)記号の導入&公理の追加をしてえられる理論S'に対しては、
>>756の主文は成り立つということが山本新の数学基礎論で証明されています
0762132人目の素数さん
2021/08/02(月) 23:11:05.50ID:DaDPbdBA
昔の学者はなんでも一通り噛ったものだ。
とくに橋爪は京大理学部数学科卒の小室直樹の指導を受けていた。
0763132人目の素数さん
2021/08/02(月) 23:16:47.26ID:ae206rHd
>>761
どれ読んでもある程度の所まではどれも同じよね
最初っからモナドで定義したヤツとかあるかな?
0765132人目の素数さん
2021/08/02(月) 23:30:26.62ID:ae206rHd
>>762
>京大理学部数学科卒の小室直樹
へぇ〜と思って見たら
ホントにそうじゃんw
トポロジーやってたのか
でも院は経済学なんだな
なんかあれこれ手を出してコジレた典型のような
0766132人目の素数さん
2021/08/02(月) 23:33:22.24ID:ae206rHd
>>764

最初から超準解析で書いてある微積の本は見たときあるけど
位相空間をモナドから始めてる本とかあるかな
0767132人目の素数さん
2021/08/02(月) 23:45:27.82ID:Df+le9y8
KdV系になるんじゃね?知らんけど
だからはっきりは書いてくれないんじゃない?
0770132人目の素数さん
2021/08/03(火) 02:51:11.70ID:TW9rMP0j
>>765
こじれたから全共闘か笑
0771132人目の素数さん
2021/08/03(火) 10:07:54.15ID:EKwPtSOf
小室直樹先生は亡くなる少し前
日本人がこれから目指すべき方向について問われ
「キリスト教をもっと勉強したらよい」
と答えられていました。
0772132人目の素数さん
2021/08/03(火) 10:37:38.38ID:JR1nXnyQ
>>761
ケリー以外に引用してあるのは
ブルバキと松坂和夫とBH Arnoldのトポロジー入門とか
だから、割と参考文献の選び方としては順当な気はする。

たぶん誰かもう少し数学が分かっている人に
教えてもらったんだろうけど。
0773132人目の素数さん
2021/08/03(火) 11:00:52.53ID:HE9n2APK
小室直樹 → トポロジーに挑戦して玉砕
高橋洋一 → 整数論に挑戦して玉砕

経済学って数学ができない馬鹿がやるトンデモ学問だったんだな
0774132人目の素数さん
2021/08/03(火) 11:02:03.44ID:HE9n2APK
ま、小室程度じゃ、トムやミルナーやスメイルにはなれんよな
0775132人目の素数さん
2021/08/03(火) 11:14:07.35ID:ZIt5dGpr
ケリーは爺さん世代のスタンダード
0776132人目の素数さん
2021/08/03(火) 11:16:58.78ID:ZIt5dGpr
今もケリーは需要があるらしく、新品が岩波オンデマンドで買える。
0777132人目の素数さん
2021/08/03(火) 12:05:09.31ID:VJM+dwYQ
>>771
世界宗教ならなんでもいいと思う。仏教でも。
直にあたってもよくわからないからオウムから
ワード拾うのが順当か
0778132人目の素数さん
2021/08/03(火) 12:14:55.70ID:VJM+dwYQ
>>773
経済学者のサムエルソンはジョージ・バーコフの定理を使ったというが、経済学者がどこからバーコフの情報を得たんだろうか?

まだわからんが、小泉構造改革はおぼろげに見えてきた
アベはいろいろまざりすぎてて主題がわからん。ほかもさっぱり
0780132人目の素数さん
2021/08/03(火) 14:22:29.50ID:TNGXKiaO
>>776
ケリーは今の学生が読むのか?
俺の時は教授が絶対読めって言ってた
しかも原著で
0782132人目の素数さん
2021/08/03(火) 15:07:21.90ID:IAB08IND
>>781
妙な事かどうか
リーマン予想を解くために聖書を読めと言ったのならともかく
0783132人目の素数さん
2021/08/03(火) 15:34:49.68ID:IAB08IND
歎異抄を理解できた頭で聖書を深読みしてみろ
といったような意味とも思える
0784132人目の素数さん
2021/08/03(火) 15:40:41.69ID:+cIcf4sZ
>>783
聖書だ宗教だなんてどうでもいいから
0786132人目の素数さん
2021/08/03(火) 16:35:05.38ID:fxXuYEAj
基礎論と宗教は無関係と言っていいのか?
0787132人目の素数さん
2021/08/03(火) 16:38:41.89ID:TOoJ5avY
おちんちんまつりとの関係ならある
0788132人目の素数さん
2021/08/03(火) 16:44:09.75ID:Xm2mQrYb
>>786
分析哲学と大陸哲学の違いもご存知ない?
0789132人目の素数さん
2021/08/03(火) 18:06:41.55ID:S0HwPSGS
>>780
ケリーはネットの収束のところがおかしいらしいとか
グロタンディークのネタ本だとか
評価写像evのところが核心だろうとか思うけど
途中まで読んでほったらかしにしてる
0790132人目の素数さん
2021/08/03(火) 18:10:06.90ID:S0HwPSGS
>>781
素人だが見習ってなんでも宗教に絡めたトンチキ話するおじさんに切り替えていくかな
0791132人目の素数さん
2021/08/03(火) 18:13:49.92ID:x55Nb0B2
一般位相は位相群知ってると反例、実例作りに重宝するぞ。
0792132人目の素数さん
2021/08/03(火) 18:20:06.57ID:IAB08IND
>>788
ラッセルは亡くなる直前「自分はヘーゲルは嫌いだ」と言った
ということを知っている程度には知っている。
0793132人目の素数さん
2021/08/03(火) 18:24:26.99ID:S0HwPSGS
ブレンターノではないんか。マーチンレーフは暗にブレンターノ読めだが追いきれない
0794ハノン ◆QZaw55cn4c
2021/08/03(火) 18:36:57.42
>>773
高橋洋一って数論の人だったんですか?
数論、位相幾何学、群論、とか当時の流行ってよくわかんないですね‥‥
0796132人目の素数さん
2021/08/03(火) 18:54:48.66ID:rIv9jF12
学部しか出てない人に専攻分野も何も無いと思う

>>776
何かと勘違いしてない?
ケリーの邦訳が出てたのは吉岡書店で、
岩波から出てるのは日本人が書いた本しかないと思うけど
0798132人目の素数さん
2021/08/03(火) 19:44:10.33ID:HQv5Z/ou
>>796
アマゾンの書誌の間違いやね

オンデマンドのはケリーの翻訳者の書いた本
リンク貼れない
0799132人目の素数さん
2021/08/03(火) 22:39:09.02ID:Ny+L90zP
ケリーの焼き直し本なら野口宏の位相空間がある
なんでかアマゾンに見当たらなかったけど
中身はほぼケリーだった記憶。ただわかりやすいかと言われると省略が多いから結局ケリーのほうがいい
0800132人目の素数さん
2021/08/03(火) 22:57:23.43ID:Ny+L90zP
年代的に橋爪大三郎のメンターは野口宏というのは有り得そう。
0801132人目の素数さん
2021/08/03(火) 23:04:16.44ID:FXrewn3U
数学者の話好きね君たち
結局はどうでもいいのに
0802132人目の素数さん
2021/08/04(水) 01:27:46.93ID:eQDgNMvs
ここにいるやつってみんな専門は何や?
証明論?モデル理論?
0803132人目の素数さん
2021/08/04(水) 07:14:11.39ID:zntJXgxu
P.Cohenの専門は実解析だった
0804132人目の素数さん
2021/08/04(水) 09:34:48.91ID:7Yppv9rp
山本新の数学基礎論を読んでるんだが、V_G(x)の定義の意味が全く分からん
関連でx^\barとかx^\lorみたいなのも定義されてるが、なぜそんな概念を定義しようとしているのか、っていう説明が皆無だから何がしたいのか全く分からん
どうようにp||-Aの定義(強制法)の動機も分からん

定義の動機や背景を説明してもらわないとホント「なにがしたいの?」って感じ
0805132人目の素数さん
2021/08/04(水) 12:27:13.80ID:IYSEH6yW
>>804
フーリエ解析?みたいにはっきりした応用がないから動機がよくわからないんじゃない?
0807132人目の素数さん
2021/08/04(水) 14:57:02.87ID:FSJRf93S
児強制法の動機説明してる本はキューネンしかないから
理解したかったら、そういう無名の教科書よりは
有名な本を読んだ方が近道だと思う
0808132人目の素数さん
2021/08/05(木) 00:13:55.51ID:/ZyxqiWp
金森の巨大基数の集合論はレベルが高すぎるな
先端の研究者しか読めないでしょ、これ
0809132人目の素数さん
2021/08/05(木) 00:42:30.32ID:zgko0I2H
ここにいる奴らってみんなちゃんとした数学機関に属してんのか?
宗教やら何やらのやつは公民館っぽいが
0812132人目の素数さん
2021/08/05(木) 04:18:06.64ID:mqwMCb9K
>>809
数学者やその書籍を話題にするヤツは何か違うキガス
0813132人目の素数さん
2021/08/05(木) 08:51:17.06ID:/ZyxqiWp
>>810
サンクス
とりあえず今読んでます

数理論理学を産まれて初めて触れたときの「は?何やってんの?」感って昔に味わったけど、
強制法辺りの各種記号の導入や議論の入り方はその当時と似た感情を思い起こさせるわ
0814132人目の素数さん
2021/08/05(木) 10:44:37.55ID:/ZyxqiWp
愛媛大学の藤田博司って94年に講師になってるのに27年たった今でも講師止まりなんだが何で?
本人の能力不足なのか?
0816132人目の素数さん
2021/08/05(木) 12:08:06.99ID:/ZyxqiWp
無限基数κに対してκより大きい最小の基数をκ^+と表記する
という約束や表記はよく見るけど、
κはアレフ(α)と一意に表せるので、κ^+の定義なんて持ち込まなくても、アレフ(α+1)を表せばいいじゃん、
って前から思ってたんだが、何でそうしないの?
0817132人目の素数さん
2021/08/05(木) 13:31:52.90ID:zGC1GRIl
基数と順序数は区別した方がよい
0820132人目の素数さん
2021/08/05(木) 14:23:21.09ID:6EkLrMf1
>>818訂正
どっちでも良いから
>>819訂正
こっちのが正論
0821132人目の素数さん
2021/08/05(木) 14:35:54.13ID:/ZyxqiWp
>>817
Card(κ) :≡ μα(∃可逆写像f:α→κ) ぐらいは知ってる
っていうか、ACを仮定するかどうかで議論が変わる可能性があるからκ^+とアレフ(α+1)を使い分けてるのかなとは思ってるが違うのかな
0822132人目の素数さん
2021/08/05(木) 16:55:58.67ID:zGC1GRIl
ACは問題が多いとして避けられる傾向にあるようだから
あるいはそうかもしれない
0823132人目の素数さん
2021/08/05(木) 17:02:23.79ID:2KQ8NsR5
ACを否定した数学なんて不自由極まりない
0824132人目の素数さん
2021/08/06(金) 00:31:00.88ID:zW0yYebI
別に選択公理が避けられる傾向なんかないよ
ただ決定性公理の研究なんかをする時には
ZFで考えないと仕方ないから選択公理なしで
どこまで出来るのかと言う事も、集合論プロパーとしては
大事ではあるけど

>>816
単純に両方の書き方があった方が便利じゃん
0825132人目の素数さん
2021/08/06(金) 05:01:55.23ID:XKWfpwfN
一応数理論理学関連の動画をアップしてる日本人Youtuber貼っておく

Maruyama Lectures
https://www.youtube.com/c/MaruyamaLectures/videos

川井新:論計舎主催
https://www.youtube.com/channel/UCDlRxYI4V7hxmaDEft5XMIQ/videos

Kyoto Logic←普通に哲学の授業
https://www.youtube.com/channel/UCwXt5me9JwL1N7gkXIDLYXQ/videos

Takayuki Kihara Lab
https://www.youtube.com/channel/UCwsTJy8qRylVe5xT5DQtpEQ/videos
0826132人目の素数さん
2021/08/06(金) 09:12:17.58ID:7I4XiH80
独 立 性 証 明 と そ の展 望
難 波 完 爾
0827132人目の素数さん
2021/08/06(金) 11:59:46.60ID:XKWfpwfN
>>826
科学基礎論研究に無料配布されてる数学基礎論関係の論文が大量にあるな
今後時間をかけて落としていくわ
0828132人目の素数さん
2021/08/09(月) 04:35:25.79ID:4MqPbXQd
強制法がやっぱわからん
自分の無能さを痛感するわ
0829132人目の素数さん
2021/08/09(月) 11:38:20.40ID:4MqPbXQd
>>810
chapter6まで読んだけど、これでForcingの概念をまだ5%ぐらいしか理解できないんだが俺ってよっぽど向いてないんか?
0830132人目の素数さん
2021/08/09(月) 12:10:54.84ID:VjqiAWcX
Quora 「計算機科学者のダナ・スコットは、
当初数理論理学の強制法の分野で顕著な業績を
出したらしいのに、なぜわざわざそれを全部捨
てて計算機科学の分野に移ったのですか?その
理由としてはどういうものがあったのですか?」

ここ見てんじゃないか?
0833ハノン ◆QZaw55cn4c
2021/08/09(月) 13:37:18.33
>>830
数理論理学と計算機科学とは、もとから親和性が高いのですよ
数論と計算機科学とはどうでしょうか?
0834132人目の素数さん
2021/08/09(月) 15:02:11.15ID:4MqPbXQd
証明はどんなものであっても式変形追えばいいだけだから、まだいいけど
わけの分からん定義はほんとに困るな
その定義を持ち出そうとする思想的背景が分からんとずっと気持ち悪い
0835132人目の素数さん
2021/08/09(月) 15:05:59.72ID:4MqPbXQd
>>834
一文目はちょっと言い過ぎたな
証明だってややこしいのは普通にあるから
0836132人目の素数さん
2021/08/09(月) 15:29:45.95ID:RR0M77Lu
>>834
>その定義を持ち出そうとする思想的背景が分からんとずっと気持ち悪い
分かるまで勉強する他有るまい?
0837132人目の素数さん
2021/08/09(月) 15:31:29.82ID:RR0M77Lu
分かりにくい書き方をするとかで
この本が良い悪いとかいう奴たまに出てくるが
ホント努力しないなとしか思わないな
数学から最も遠い態度じゃないか?
0838132人目の素数さん
2021/08/09(月) 15:44:24.19ID:4MqPbXQd
>>831
サンクス。見てみます。

>>837
読者の理解力が劣っていて、その読者にとっては良くないように思えているのか、
本当にその著者の説明の仕方が悪くて良くなく見えるのか
初見の分野を学ぶ時は知識がないから見分けがめっちゃつきにくい
0839132人目の素数さん
2021/08/09(月) 19:14:00.47ID:GrIG3IAP
>>834
定義を使う時に分かるんじゃねーの?
準備じゃなく本格的に使う時
0841132人目の素数さん
2021/08/09(月) 19:28:10.99ID:RR0M77Lu
>>840
そういうのは有るかも知れないが
特にどうでもよさげなことが多いよ
数学を指向するんだから
自分の頭で考えるがよからむ
0843132人目の素数さん
2021/08/10(火) 15:33:34.50ID:0X3Tr6oz
盲信というか結局はそれしか無い
教えてもらえるのは
何を考えるべきかということぐらい
0845132人目の素数さん
2021/08/10(火) 17:24:17.77ID:npsQ/26O
諦めて大学に入れ
0846132人目の素数さん
2021/08/10(火) 17:48:58.64ID:WtU2O+tt
>>843
盲信も何も、数学をちゃんと理解したかったから迷い込みましたって。
何を考えるべきか、ってそれは小学校でみっちり教えていることの延長線じゃないのか、またはその裏か
よくできている
0847132人目の素数さん
2021/08/10(火) 20:40:59.60ID:lf9lEBl0
どんなに頭よくても誤読とか勘違いは生まれうるから他人の頭を借りることは大事よね
0848132人目の素数さん
2021/08/10(火) 20:50:46.51ID:OypVO28U
対話が出来ない人はダメ
0849132人目の素数さん
2021/08/10(火) 20:51:46.86ID:OypVO28U
つまり貸すことも出来ないとダメ
0850132人目の素数さん
2021/08/10(火) 21:21:29.54ID:5ahfUm8x
貸しを作るのは苦手だな。進歩がないあるといったらそれまでだが
確かに対話は重要。数学だけの世界に限らない話か。
0851132人目の素数さん
2021/08/10(火) 22:13:57.80ID:OypVO28U
>>850
勉強は宗教の修行じゃないから手を借りたり貸したりしてもなんの問題もない。
0852132人目の素数さん
2021/08/10(火) 22:37:03.83ID:X8nsiI//
ゲーデルの哲学を理解するにあたっては、当初ゲーデルはウィーン学団に出入りしていたというのはでかい
つまりウィトゲンシュタインの業績(『論理哲学論考』)を結構確かなルートで聞いていたと考えるのが普通
つまり不完全性定理には論考の哲学が入っていてウィトゲンシュタインを理解しないことには
その哲学的意味を理解できないんじゃないかと思う。

後になってゲーデルはウィーン学団に出入りしていたことを否定しだしたり、ウィトゲンシュタイン自身もウィーン学団との繋がりを否定しだしたりするのも
論考の哲学が影響していると考えるべき。
正直、ウィトゲンシュタインの哲学がこんなに重要だとは思ってなかった。
0853132人目の素数さん
2021/08/10(火) 23:11:38.62ID:X8nsiI//
マーチン-レーフの型理論は果たして本当にまともな理論であるか。
単にホーア論理とカリー=ハワード同型対応を組み合わせておまけにカテゴリ論で味付けしただけなのではないか。
後付される判断理論。初期はそんな事言ってなかったのに。
そしてなんで誰もそういうツッコミを入れないのか。
0854132人目の素数さん
2021/08/10(火) 23:13:33.35ID:OypVO28U
>>852
ヴィトゲンシュタインはゲーデルの業績にはそんな重要な存在じゃないと認識している
あなたの主張は何に基づいているの?
0855132人目の素数さん
2021/08/10(火) 23:14:49.98ID:OypVO28U
>>853
ああ、ハッタリだったか
0856132人目の素数さん
2021/08/10(火) 23:24:03.72ID:X8nsiI//
>>854
ウィトゲンシュタインがいないとゲーデルが不完全性定理をヒルベルトにおそるおそる提示したら
ヒルベルトがなんやかんや即座に不完全性定理を認める、というくだりが理解できない。
0858132人目の素数さん
2021/08/11(水) 01:14:35.00ID:6oXTjifJ
超準解析のいい本ってあります?
0859132人目の素数さん
2021/08/11(水) 05:32:25.38ID:VIPPB4Lj
>>851
そりゃ当然
ただね
「手を貸してくれなくて糞」
とか言うのには呆れる
0860132人目の素数さん
2021/08/11(水) 16:59:09.14ID:JSdwf1xz
二項関係について、
等しいことを含まない大小関係(狭義順序)"<,>"や
等しい集合を含まない包含関係である真部分集合"⊂,⊃"は、
推移関係は成立するが、非反射関係である。
ゆえに前順序(preorder)にもならない。
0861132人目の素数さん
2021/08/11(水) 17:37:15.72ID:s7XzqIsq
最後の行がおかしい
0863132人目の素数さん
2021/08/11(水) 18:01:22.50ID:VIPPB4Lj
>>860
順序の定義は2つあって
反射律の成立を条件とするものと
反射律が成立しないことを条件とするもの
0864132人目の素数さん
2021/08/11(水) 18:25:59.35ID:JSdwf1xz
>>861
前順序関係とは、反射関係と推移関係が成立する二項関係では?
0866132人目の素数さん
2021/08/11(水) 19:08:08.47ID:JSdwf1xz
>>863
反射律の成立しない順序が狭義の順序
0867132人目の素数さん
2021/08/11(水) 19:10:06.94ID:JSdwf1xz
>>865
同値関係には、反射律・推移律の他に対称律が必要
0868132人目の素数さん
2021/08/11(水) 19:30:46.75ID:VIPPB4Lj
>>865
本質は同じ
順序の本質は推移律だから
0870132人目の素数さん
2021/08/11(水) 19:58:51.11ID:muvTl4J1
役に立つかどうかはともかく、等号(同値関係)の部分を除いた不等号<を一般化すること自体は自然だと思うの

>>868
同値関係との違いという点では反対称律が本質と言える
反対称な同値関係(または対称な順序関係)は等号に限られる
0871132人目の素数さん
2021/08/11(水) 20:27:49.93ID:BlPzYbQk
延々分かり切ったことを書き綴るスレ
0872132人目の素数さん
2021/08/11(水) 20:42:25.87ID:s7XzqIsq
>>860が馬鹿すぎてオモチャにされている
0873132人目の素数さん
2021/08/11(水) 20:43:51.01ID:HCiCPuy7
5chってそんな得ることの大きい確かな情報得るところじゃないよ。
ノイズ大きいし、真偽不明の話するにしても結局は実地で確認するのが一番だろうし
0874132人目の素数さん
2021/08/11(水) 20:47:03.88ID:HCiCPuy7
>>872
狭義順序というものの話をしているだけでは?
どこで使うかわからないが
0875132人目の素数さん
2021/08/11(水) 22:23:32.09ID:s7XzqIsq
>>874
「ゆえに前順序(preorder)にもならない。」
こう言ったのがまずかったね。
>>863さんの言うように順序の定義は<でやる流儀も、≦でやる流儀もあるのに。
0877132人目の素数さん
2021/08/11(水) 22:50:23.87ID:s7XzqIsq
<を使う流儀
https://en.wikipedia.org/wiki/Preorder
のstrictly preorderと呼ばれるものになるね。
要は定義次第だ。
0878132人目の素数さん
2021/08/11(水) 22:50:53.52ID:s7XzqIsq
流儀→流儀だと
0879132人目の素数さん
2021/08/12(木) 01:03:21.12ID:JdN7kKyO
社会学者の今田高俊という人の『自己組織性』ってソーカル本(こう呼ぶことに間違いはないはず)見てたら
あとがきに前原昭二、吉田夏彦哲学教授から教示を受けたとサラッと書いてあった。

読んでも意味のわからないソーカル本の片棒活がされてるみたいで気の毒
ちょっと出入りしていただけだろうに

それになんで分野が「哲学」だとしているんだろうか
0881132人目の素数さん
2021/08/12(木) 19:17:24.74ID:JdN7kKyO
>>880
それらしい数理的な文言、この場合リフレクションとかゲーデルの不完全性定理とかを散りばめてあるものの
何が書いてあるか内容がわからない(=中身がないと考えるのが当然)本や論文
という意味つまり否定的意味以外にあるか?
0882132人目の素数さん
2021/08/12(木) 19:35:44.63ID:NdzfJ5Wy
インチキを告発する本をソーカル本というのでは?
0883132人目の素数さん
2021/08/12(木) 19:44:53.92ID:aPv9BUXR
ポストモダンの欺瞞、似非科学であることを告発

ソーカル事件
https://ja.m.wikipedia.org/wiki/ソーカル事件
0884132人目の素数さん
2021/08/12(木) 19:48:25.50ID:JdN7kKyO
そんな意味あったっけ?あれマッチポンプじゃないの?
うまく読み取れないな。
0885132人目の素数さん
2021/08/12(木) 19:50:58.97ID:VaiKH8kF
浅田彰とかはおまい等の大先輩、仲間w
インチキ同士仲良くな
0886132人目の素数さん
2021/08/12(木) 19:56:17.37ID:JdN7kKyO
>>885
カタストロフ理論とかフラクタルとかの数学書で対談とかしてんのな

結構俺は好きなんだけど東浩紀とかは?インチキですらない?
0887132人目の素数さん
2021/08/12(木) 22:43:47.12ID:SjgHPfaV
何か
誰かの批判したいだけの人が居るようね
どっかそれ用のスレないかな
0888132人目の素数さん
2021/08/12(木) 23:06:01.49ID:JdN7kKyO
ゲーデルの不完全性定理を巧みに自説に盛り込んでわけのわからないような解釈を開陳する人文系に対しては反対なのではなのか?

マッチポンプじゃないガチの批判はお断り?
0889132人目の素数さん
2021/08/12(木) 23:35:23.21ID:FxSbLfca
879 名前:132人目の素数さん[sage] 投稿日:2021/08/12(木) 01:03:21.12 ID:JdN7kKyO [1/5]
社会学者の今田高俊という人の『自己組織性』ってソーカル本(こう呼ぶことに間違いはないはず)見てたら

882 名前:132人目の素数さん[] 投稿日:2021/08/12(木) 19:35:44.63 ID:NdzfJ5Wy
インチキを告発する本をソーカル本というのでは?

884 名前:132人目の素数さん[sage] 投稿日:2021/08/12(木) 19:48:25.50 ID:JdN7kKyO [3/5]
そんな意味あったっけ?あれマッチポンプじゃないの?
うまく読み取れないな。


ID:JdN7kKyOってわかっててわざと紛糾するような間違い書いてるだろ
>>887も指摘してるけど、今までもこういう変な書き込みしては
俺がスレを盛り上げてやったって一人でほくそえんでたのかね
きもすぎ
0890132人目の素数さん
2021/08/12(木) 23:52:51.08ID:SjgHPfaV
>>888
要らない
0891132人目の素数さん
2021/08/12(木) 23:53:18.55ID:SjgHPfaV
どっか別でやって
それと
人文系のスレでやって
0892132人目の素数さん
2021/08/12(木) 23:53:47.82ID:JdN7kKyO
そうだよ
今までゲーデルの不完全性定理は生半可な知識の文系が語ると誤用するから
そういう生半可な文系の定理の解釈は徹底して正ていこうというのが共通認識としてあった
だからそういう路線で書いた

ただ、疑念がある。それを確認もしたかった。
0893132人目の素数さん
2021/08/13(金) 00:03:05.12ID:tnFMAK/+
脊髄反射で反論してたわけじゃなかったんだな
わかったよ。人文系で手がかりを探すよ
0894132人目の素数さん
2021/08/13(金) 03:50:08.43ID:Lo0YbqQL
>>892
利用と誤用の不完全ガイドというものがあってだな…
0895132人目の素数さん
2021/08/13(金) 08:01:51.09ID:BXmMpLnH
>>892
>共通認識としてあった
どこの?ここにはないよ
0896132人目の素数さん
2021/08/13(金) 08:09:59.25ID:nCXCDdpU
>>894
フランセーンの本ね

ソーカル本じゃなくトンデモ本と書けばよかったね

ソーカルとブリクモンの「知の欺瞞」は
ポストモダンにおける数学の誤用(つまりトンデモ)
を批判する本だから
0897132人目の素数さん
2021/08/13(金) 14:04:57.99ID:2PNHCMxt
「ソーカル本」を
トンデモ本の意味で使う人と告発本の意味で使う人は
どちらが多いんだ?
0898132人目の素数さん
2021/08/13(金) 15:35:07.23ID:r3Lrmbox
しかしソーカルなんて記号論理と意味論をよくわかってない人間に対して鬼の首を取ったかのように『知の欺瞞』なんて言ってるが、問題はポストモダン自体じゃなくて、その当事者だっただけだろう。
大体哲学系の論文なんて査読もクソもないし、ちゃんとした数学系の論文と比べてもしょうもないよな。
数学者以外は数学用語を誤用しているなんて当たり前じゃないか。
0899132人目の素数さん
2021/08/13(金) 15:47:30.98ID:nCXCDdpU
>>898
ポストモダンで、数学を「ハッタリかまし」に使う事例は顕著だよ

ドゥルーズはその最たるものだね
フーコーはそういうことはしないけど
0900132人目の素数さん
2021/08/13(金) 16:06:18.78ID:uG9VOe/F
ソーカルに向けられる非難が八百長を告発した内部関係者に向けられた非難と全く同質で笑った覚えがある
0901132人目の素数さん
2021/08/13(金) 16:08:24.19ID:inz5S3Dg
>>899
哲学はよく知らないが、分析哲学以外が数学を使うのがおかしいんだろ。
それこそ大陸哲学なんてウィトゲンシュタインによって死んだんだから
0902132人目の素数さん
2021/08/13(金) 18:13:47.40ID:uG9VOe/F
「大陸哲学なんてウィトゲンシュタインによって死んだんだから」
こんなハッタリ誰も信じないよ
0903132人目の素数さん
2021/08/13(金) 18:49:27.50ID:xFTEdzv8
黒崎 宏
悪の起源: ライプニッツ哲学へのウィトゲンシュタイン的理解

ライプニッツがモナド論とともに哲学は死んだと言っていたらどうなっていたか
読んでないけど
0904132人目の素数さん
2021/08/13(金) 18:58:58.54ID:BXmMpLnH
哲学の話は哲学板でやって
そんな板あるか知らんけど
0906132人目の素数さん
2021/08/13(金) 19:50:24.90ID:+IHboGp5
屑哲
0908132人目の素数さん
2021/08/13(金) 22:42:03.05ID:Lo0YbqQL
気を抜いたらゴミみたいな哲学の話題になるな笑
0910132人目の素数さん
2021/08/16(月) 00:01:03.96ID:I/GAonL7
XからYへの配置集合"Map(X,Y)=Y^X={f|f:X→Y}"は、
XからYへのあらゆる写像の集合となる。

一般にある一つの写像を定義しようとする時、そのXとYの其々の要素の対応
について、何らかの形で規則性を説明できる形のものを定義しようとする。

各要素の対応について、一見、その規則性を見出すことができない写像を
扱うことは余り無い。
例えば定義域X={1,2,……9,10}から値域Y={1,2,3}への以下の様な写像があ
ったとする。
1→1,2→1,3→1,4→2,5→1
6→3,7→2,8→2,9→3,10→3
これはXからYへの配置集合Map(X,Y)に含まれる3^10=59049通りの写像の中の
一つである。

上記の配置集合には、そういう対応の論拠を説明しにくい写像も含む。
0911132人目の素数さん
2021/08/16(月) 02:31:32.26ID:5l+ZBffD
自明な事を嬉しがって長々書くのやめてほしいです
0912132人目の素数さん
2021/08/16(月) 04:08:19.78ID:5zJAuxz/
>>910
↑俺こういうふうな発展性のない議論(?)っていうか、ただ単に思索にふけってるだけの議論嫌いだわ
まさしく「だから何?」

そういうのが好きなやつは哲学科行け
0913132人目の素数さん
2021/08/16(月) 13:14:56.58ID:aVQsGNJt
哲学好きってやつと、晦渋になぞなぞしてくるやつの2パターンが少なくともある。
0914132人目の素数さん
2021/08/16(月) 14:00:54.28ID:gRd59Vjf
数理論理学って哲学かと思ったわw
0916132人目の素数さん
2021/08/17(火) 04:48:23.33ID:r+D2qFJk
このYouTubeの動画どうよ

謎の数学者【アメリカ大学准教授の数学者】
数学基礎論が衰退した理由は何故か?理由を考察
https://youtu.be/1X8npQqoGkQ

同じ投稿者
数学基礎論、発展と衰退の歴史。約100年の系譜。
https://youtu.be/jIPjkGPecnE

アメリカ大学っていう大学があるのか知らないが笑
0918132人目の素数さん
2021/08/17(火) 12:55:26.90ID:MEBmuRb0
間違っちゃいないわな
こういう見識もある
0919132人目の素数さん
2021/08/17(火) 12:55:49.68ID:528Uw3fh
>>917
特定されてるんだね
やたらと基礎論を貶してるし、酷いのはコメント欄で「今から基礎論を志すのは良くないんですか?」という質問に「はい。やめたほうがいいです」と返信している。

たけだしゅういちろうと言うんだな笑
0921132人目の素数さん
2021/08/17(火) 14:09:19.31ID:kPf/Sxz1
>>919
>「はい。やめたほうがいいです」
これは実感
職がない
情報系と張り合うのも不利
0922132人目の素数さん
2021/08/17(火) 14:16:43.26ID:lJwyCdka
>>921
好きでやりたいと言っている人間に「もう終わった分野」だとイデオロギー的なレッテル貼りで「やめたほうがいい」ということが、どこが正しいんだ?
0923132人目の素数さん
2021/08/18(水) 02:05:11.03ID:B6pHFd6O
死にたいんですと言って今にも崖から飛び降りようとしてる人に自殺なんて馬鹿のやることだやめた方が良いと言い聞かせるのは間違ってるとは思えないけどね
0925132人目の素数さん
2021/08/18(水) 19:32:50.04ID:n/bq52Go
>>923
上手い言い回しね
0926132人目の素数さん
2021/08/19(木) 00:29:05.03ID:cKYpNcOD
>>923
論理学できなさそう
0927132人目の素数さん
2021/08/22(日) 12:22:25.13ID:K424t7Qg
数論幾何からみたらつまらないように見えてもしょうがない
でもバカにして新しい体系を創設しようとしたら、ABC予想巡ってのお粗末な展開みたいになるだけだけどね
0928132人目の素数さん
2021/08/22(日) 13:36:19.02ID:X6ixO12a
女体の神秘に比べたら詰まらないのは仕方ない
0929132人目の素数さん
2021/08/23(月) 16:39:17.72ID:CGiJtKwu
強制法が未だに分からん前にレスしたやつです

強制法はZFCのモデルMに都合よくジェネリックフィルターGを添加して拡大したモデルM[G]を作る話ってことまでは理解できた

p||-a∈b, p||-a=bの定義の思想的背景

P-名前の定義の思想的背景

を知りたい
0930132人目の素数さん
2021/08/23(月) 19:43:39.94ID:CGiJtKwu
A Beginner's Guide to Forcing Timothy Y.Chowの黙読が一応終わった
正直強制法の5%ぐらいしかまだ理解できてない

ブール値モデルによる強制法は直観的に理解しやすいけど応用が効かないと巷で言われてるから、俺はこっちではやりたくない
0931132人目の素数さん
2021/08/24(火) 03:07:14.72ID:Bsh1Qh3T
まず直観的に大体何をやってるのか理解しないと
応用も何も無いと思う。

ZFの公理から、universeは累積階層
V = ∪ V_α の形になっていることが分かる。
だからM = ∪ M_α にある要素 x を付け加えたときに
Mとxを含む最小のモデルM[x]がどうなるかを
考えたいなら、どっかの層にxを付け加えて
次の層、次の次の層、…がどうなるかを考えれば
良さそうだけどそうすると新しい要素が
付け加わる過程と順序数に沿って合併を取る過程が
混在することになる。

その二つを分離する為にあるのがnameだと
言っていいんじゃないかな。
いずれにせよ全体像が分かってなきゃ
分からないと思うけど。
0933132人目の素数さん
2021/08/24(火) 19:51:12.94ID:Bsh1Qh3T
お勧めは証明飛ばしていいから先に
Jechとかネットの資料とかで
Boole値モデルの勉強する事。
一通りやったら全体像が大体分かるので。

定理の主張を読んで証明を流し読みして
うんうん、そう言うことが成り立つんやな、
そんな気もするわな、そう言うことにしとくわ、
くらいのノリで。
細かい論理的な検算は後回しにするのがお勧め。
0935132人目の素数さん
2021/08/26(木) 15:26:00.81ID:i2/RXlO6
山本新、「数学基礎論」238ページ
SはZFの推移的モデル。
Pは有基底的述語でSに関して絶対的かつ、y∈SならばPに関するy切片Seg(y)⊆Sが成り立っていると仮定する。
この時、y=Seg(x)はSに関して絶対的であることはわかっている。

xに関する推移閉包y=C(x)がSに関して絶対的であることの証明だが、
y,x,f,n∈Sの時
∃m<ω(n=m+1∧y=U{Seg(v)|v∈f`m})
はSに関して絶対的
という議論をしているが、その証明がわからない。
0936132人目の素数さん
2021/08/26(木) 18:25:08.96ID:i2/RXlO6
v∈SならばSeg(v)∈Sが成り立っていないと証明できないような気がするんだが
0938132人目の素数さん
2021/08/26(木) 18:35:47.18ID:fZyan1kG
この分野って結果は面白いけどついてくのは面倒すぎるな
0939132人目の素数さん
2021/08/26(木) 18:39:13.88ID:i2/RXlO6
やっぱり丁寧に黙読してても、手を動かして勉強してる時に見つけれる小さなロジックの飛躍までは補足しきれないな
読むだけだと、ホント、なんとなく「確かに言われてみたらそんな気がする」程度の認識までしか到達できない
0940132人目の素数さん
2021/08/26(木) 21:14:48.21ID:i2/RXlO6
SをZFのモデルとする.
x=t(y1,…,yn)がSに関して絶対的であるならば,
ZFL^M|-y1,…,yn∈S⇒t(y1,…,yn)∈S
が成り立つ


↑この証明分かる人いる?
ZFLはZF+ALに更にMが加算推移モデルであることの公理を付け加えたもの
0941132人目の素数さん
2021/08/26(木) 22:05:08.99ID:i2/RXlO6
>>935-940
自己解決しました
っていうか、俺の過去の勉強ノートを見たらまとめられてた
数年ぶりに、ノートを見ずに教科書だけを見て行間を埋められなかった俺…確実に数学力が落ちてる
0943132人目の素数さん
2021/08/27(金) 16:04:05.23ID:IqQVqbpq
だいたい30歳がピーク
0944132人目の素数さん
2021/08/28(土) 10:45:39.85ID:DWrFkVt6
Provability logicってわかる人いる?
0947132人目の素数さん
2021/08/28(土) 15:36:34.84ID:j6A6Uinw
>>941-943
まあ、そう言わずに

https://style.nikkei.com/article/DGXMZO79123750R31C14A0000000/
大人になっても頭は良くなるの? 年とともに低下する知能と上昇する知能がある 2014/11/3 日経Gooday 30+

脳科学が専門の諏訪東京理科大学・篠原菊紀教授にズバリ聞いてみた。大人になっても、頭は良くなりますか?

「もちろん、イエス! 年をとったほうが断然、頭は良くなる」。うれしいことに、篠原教授の答えはため息も吹き飛ぶほどに明快だった。

篠原教授によると、頭の良さには大きく「流動性知能」と「結晶性知能」があるという。流動性知能とは、計算力や暗記力、集中力、IQ(知能指数)など、いわゆる受験テクニックに反映されるような知能のこと。この知能は18〜25歳くらいがピークで、その後は徐々に落ちていき、40代以降になるとガクンと低下する。一方、結晶性知能は知識や知恵、経験知、判断力など、経験とともに蓄積される知能のこと。こちらは年齢とともにどんどん伸びて、60代頃にピークを迎える(図1)。

大人になると10代の頃のように数字や英単語を丸暗記するのが難しくなるのは、流動性知能が低下するため、致し方ないこと。そもそも、子供の脳と大人の脳では記憶の仕方が異なっており、その変化は思春期の頃に起こるという。「子供の脳は“単純記憶型”で、言葉や数字の並びをそのまま覚えようと思えば、割と簡単に覚えられる。ところが、思春期以降、記憶の仕方は“自我密接型”ヘと変わっていく」と篠原教授。つまり、自分が納得できること、役に立つこと、意味のあることが優先的に頭に入ってくるようになり、丸暗記自体が難しくなる。ただし、筋道だって理解する力はグンと伸びる。

流動性知能もトレーニングをすれば、年をとっても伸ばすことは可能だ。しかも、「やればできる!」と思っている人ほど伸びるという。これとは反対に「できない」と思っていると、本当にできなくなる。例えば、「年とともに記憶力は落ちる」という理詰めの講義を受けてから記憶力テストを受けると、本来、年齢の影響を受けないはずのテスト内容でさえ、成績が悪くなるという実験結果も。できないという思い込みは、能力を低下させる。「大人になってもやればできる、能力は伸びるとポジティブに思っていたほうが、絶対にお得」と篠原教授は強調する。
0948132人目の素数さん
2021/08/28(土) 16:05:54.45ID:62LtynbH
>>946
なるほど読んでみます
0950132人目の素数さん
2021/08/28(土) 21:45:44.03ID:l0CsZu+O
数学書の読書で使う頭は、それまでの定理を駆使して目の前の証明の行間を補完する知能。
何度も何度も行間を埋める作業を繰り返すことにより、埋めるべき行間のパターンなるものが見えてくるので、それをも活用して行間の補完をする。
従って、行間埋めに使う頭は、単なる暗記でも、閃きでも無いのが特殊な所。

埋めるべき行間のパターンを多く経験すると、「パット見た感じ○○から攻めるのが筋」っていう将棋に似た感性が育つ
脳内の神経細胞を刺激し続けて新しい回路ができるのってまさに行間埋め(や将棋)がピッタリな感じする。

これは、あえて言うならば>>947の結晶性知能に該当するかな
0951132人目の素数さん
2021/08/28(土) 21:48:30.71ID:l0CsZu+O
>>949
人には個性がある
俺の場合は10代は体力しかない頭だったから、写経戦法で頑張ったな
正しいことを正しいと理解して正しさに辿り着くのではなく、正しいことを理屈抜きで正しいと叩き込ませる戦法もあるんだよ
0954132人目の素数さん
2021/08/29(日) 19:20:39.23ID:YnvXbLQ+
ここ1ヶ月ぐらい絶対性に関してウダウダと読書してきて
絶対性の考え方が30~40%ぐらい見えてきた
絶対性がわかってくるとp||-Aの手法による議論が追跡しやすい
未だに強制述語の定義の動機づけは全く意味不明だけど

議論自体は追跡できるけど、その背景思想がわからない時の気持ち悪さってマジで気持ち悪いな
0955132人目の素数さん
2021/08/30(月) 08:56:37.40ID:vbQcxHeL
野良の自称数学研究者の多くは統合失調(精神分裂)もしくはその予備軍である。
0956132人目の素数さん
2021/08/30(月) 10:27:48.20ID:GN+eaRSh
細かい論理的なチェックと、全体の流れの把握は
片方が出来てももう一方が出来ない事はあって
どちらかと言うと大事なのは全体の把握の方なんだよね

これが無いといくら細かい行間を埋めても
理解した気分にならないはず
0957132人目の素数さん
2021/08/30(月) 10:33:40.57ID:B5e1Su+g
>>956
>どちらかと言うと大事なのは全体の把握の方なんだよね
実感
後で埋めれるだけの実力は必要だが
証明は読み飛ばして行くべき
0958132人目の素数さん
2021/08/30(月) 11:39:54.99ID:8qLxmle2
それって守破離の破レベルの話であって、守のレベルでそれやるとおかしなことになる
0959132人目の素数さん
2021/08/30(月) 12:15:08.04ID:mBgKmkDE
>>958
957が守のレベルかどうかは
その話と関係がないものと受け止めておこう
0961132人目の素数さん
2021/08/30(月) 16:00:01.00ID:GN+eaRSh
>>958
そうなんだけど、
端的に言って、958の「守」レベルの人には
強制法を細部まで理解するのは無理
0962132人目の素数さん
2021/08/30(月) 16:34:28.41ID:mBgKmkDE
天性の資質にもよるわけか
0963132人目の素数さん
2021/08/30(月) 16:47:11.46ID:DUycNIHW
俺今ナウで強制法の勉強中だが、これを理解するのって行間を埋めれるのと、強制法の定義の背景思想を理解してるのが全く別物だと感じてる
0964132人目の素数さん
2021/08/30(月) 16:57:28.72ID:GN+eaRSh
何やってるか基本的なアイデアが何かが
一見分かりやすいのはBoole値モデルの方だけどね

>>963
Cohenは2007年に亡くなったけど、
2002年に招待講演として
『強制法の発見』という講演をしてて、
どう言う事を考えながら強制法を作っていったのかを
述べてるから参考にしたら良いよ。
0966132人目の素数さん
2021/08/30(月) 17:29:19.90ID:6gKsD7wv
>>955
そういう奴らは研究者とはいわんわな
0967132人目の素数さん
2021/08/30(月) 17:45:53.05ID:mBgKmkDE
>>965
感謝
0969132人目の素数さん
2021/08/30(月) 21:34:29.42ID:DUycNIHW
思うんだが、理論は時間が後になるほど発展し、整備されるんだから、誰かが○○した前と後とか比較する必要って無いよな
シンプルに後の方がいいやん
0970132人目の素数さん
2021/08/30(月) 22:22:57.79ID:B5e1Su+g
>>969
けどさ
微積はライプニッツの無限小の方が分かりやすくない?
厳密性はその当時は無かったし
εδによる厳密化はむしろ理解しにくかったり
0971132人目の素数さん
2021/08/31(火) 00:02:35.16ID:Ih+3yc5h
εδの方が証明し易いぞ
0973132人目の素数さん
2021/08/31(火) 17:58:45.39ID:qeyKf7qS
>>970
世間的にはεδの方が見通しがいいから超準解析は風当たり強め
俺は超準解析の方が好きだし
ロジシャンならみんな好きだろう
0974132人目の素数さん
2021/08/31(火) 19:58:31.25ID:VP5W8zLZ
εδ分からん奴が超準解析できるんかな?
両方できるなら高度な方が面白いだろ
0975132人目の素数さん
2021/08/31(火) 22:22:01.48ID:QvDMCTk0
>>974
超準解析をどこから始めるのかは知らんが
数理論理やモデル理論なんかもやるなら学部生じゃ無理 
εδなんて簡単なんだから学部卒なら誰でもわかってる
0976132人目の素数さん
2021/08/31(火) 23:26:52.53ID:SRTKJIEo
超準解析で書かれてる微積の入門書もあるし学部生に無理ってことは無いんでは
モデル理論とかの話になるならそもそもε-Nの時点でなぜ定義にε∈?という式が含まれないのにn∈?が含まれるのか?みたいなのを説明するのにストラクチャーの定義し始めることになる
0977132人目の素数さん
2021/08/31(火) 23:29:55.83ID:SRTKJIEo
×ε∈?
○ε∈R (R:実数全体の集合)
×n∈?
○n∈N (N:自然数全体の集合)
0978132人目の素数さん
2021/08/31(火) 23:29:57.15ID:QvDMCTk0
>>976
超準解析で書かれてる微積の入門書って何?
齋藤正彦?
0981132人目の素数さん
2021/09/01(水) 00:04:22.09ID:aSJuls56
>>979
俺は齋藤正彦の『超積と超準解析』から入ったけど
確かに学部3年の時に読んだわ
でもそもそも論だが、学部生で超準解析に興味ある奴なんて100人に1人ぐらいじゃないか?
名前すら知らない奴が大半だろ
0982132人目の素数さん
2021/09/01(水) 00:14:07.33ID:lg8QHoSd
無限小って何?っていうのは割と誰でも疑問に思うだろうけどそこから超準解析にたどり着くのは確かに限られてると思う
0983132人目の素数さん
2021/09/01(水) 01:26:51.22ID:S55+8A+7
別にたどり着かなくていい
矛盾しないことが補償されてる
て程度で十分
0985132人目の素数さん
2021/09/01(水) 13:25:04.43ID:aSJuls56
>>979
邦訳は『無限小解析の基礎ー微積分の新手法』(齋藤正彦訳)で合ってる?
0986132人目の素数さん
2021/09/01(水) 15:05:02.73ID:ztlAzOa/
知らん
もう忘れた
ファインマン経路積分とか勉強しろ
0987132人目の素数さん
2021/09/01(水) 15:41:38.02ID:L+5g5kKC
>>986
それって物理?くだらないからやらないよ
0990132人目の素数さん
2021/09/01(水) 21:12:07.98ID:aSJuls56
>>989
モデル理論で良い入門書ってある?
0991132人目の素数さん
2021/09/01(水) 23:22:59.67ID:lg8QHoSd
横からだけどキューネンの基礎論の本でモデル理論が分かり易く解説されてる
一応そこだと参考文献として
C. C. Chang and H. J. Keisler, Model Theory
D. Marker, Model Theory, An Introduction
があげられてた
0994132人目の素数さん
2021/09/02(木) 14:10:41.66ID:GB3f6WVF
D. Marker, Model Theory, An Introduction がおすすめされてるが amazon のレビューだと一人じゃ読めない位には難しいみたいなこと書かれてるな
0995132人目の素数さん
2021/09/02(木) 14:46:05.02ID:obollUaR
ここの人って数論には興味持てる?
自分は解析や数に全く興味が湧かないんだ
数理論理に行きたいんだけど一般的な数学も出来ておくべきかな?
0996132人目の素数さん
2021/09/02(木) 15:48:08.69ID:38cRHrSl
解析や数論に全く興味が持てなくても
数理論理で大成した人は知っている。
その人が一度、専門が解析である私に向かって
「P. Cohenは解析の専門家だったんだよ」と言ったが、
それは私にとって
数理論理への誘いにも聞こえた。
0998132人目の素数さん
2021/09/02(木) 17:31:26.08ID:taJVzx6A
>>995
純粋数学の知識はほぼなくても数理論理学は何ら問題なく学べる
必要な数学的知識は初等整数論と位相空間論の初歩ぐらいか
でも数理論理学はやってることは数学とほぼ同じだから、抽象思考力を養うという意味では純粋数学は避けて通れないんじゃね?

群の準同型定理やテーラー展開すら知らない数理論理学の研究者っているんかな?w
0999132人目の素数さん
2021/09/02(木) 18:37:10.94ID:c/mmP2tw
>>996
>>997
自信が無かったけど
自分がやりたいことをやるよ
ありがとう
1000132人目の素数さん
2021/09/03(金) 10:48:02.89ID:1ccWtNOZ
次スレはまともになりますように
10011001
Over 1000Thread
このスレッドは1000を超えました。
新しいスレッドを立ててください。
life time: 895日 22時間 52分 16秒
10021002
Over 1000Thread
5ちゃんねるの運営はプレミアム会員の皆さまに支えられています。
運営にご協力お願いいたします。


───────────────────
《プレミアム会員の主な特典》
★ 5ちゃんねる専用ブラウザからの広告除去
★ 5ちゃんねるの過去ログを取得
★ 書き込み規制の緩和
───────────────────

会員登録には個人情報は一切必要ありません。
月300円から匿名でご購入いただけます。

▼ プレミアム会員登録はこちら ▼
https://premium.5ch.net/

▼ 浪人ログインはこちら ▼
https://login.5ch.net/login.php
レス数が1000を超えています。これ以上書き込みはできません。

ニューススポーツなんでも実況